SlideShare a Scribd company logo
1 of 65
Tải tài liệu tại sividoc.com
Viết đề tài giá sinh viên – ZALO:0973.287.149-TEAMLUANVAN.COM
ĐẠI HỌC THÁI NGUYÊN
TRƯỜNG ĐẠI HỌC KHOA HỌC
PHẠM THỊ THANH THỦY
PHƯƠNG PHÁP DIỆN TÍCH VÀ THỂ TÍCH
TRONG HÌNH HỌC SƠ CẤP
LUẬN VĂN THẠC SĨ TOÁN HỌC
THÁI NGUYÊN - 2017
Tải tài liệu tại sividoc.com
Viết đề tài giá sinh viên – ZALO:0973.287.149-TEAMLUANVAN.COM
ĐẠI HỌC THÁI NGUYÊN
TRƯỜNG ĐẠI HỌC KHOA HỌC
PHẠM THỊ THANH THỦY
PHƯƠNG PHÁP DIỆN TÍCH VÀ THỂ TÍCH
TRONG HÌNH HỌC SƠ CẤP
LUẬN VĂN THẠC SĨ TOÁN HỌC
Chuyên ngành:Phương pháp Toán sơ cấp
Mã số: 60 46 01 13
NGƯỜI HƯỚNG DẪN KHOA HỌC
PGS.TS. ĐÀM VĂN NHỈ
THÁI NGUYÊN - 2017
Tải tài liệu tại sividoc.com
Viết đề tài giá sinh viên – ZALO:0973.287.149-TEAMLUANVAN.COM
iii
Mục lục
Mở đầu iv
Chương 1. Phương pháp diện tích 2
1.1 Định lý Pythagore . . . . . . . . . . . . . . . . . . . . . . . . . . 2
1.1.1 Tam giác vuông . . . . . . . . . . . . . . . . . . . . . . . 2
1.1.2 Hệ tọa độ Descarte vuông góc . . . . . . . . . . . . . . . 6
1.2 Định lý Stewart . . . . . . . . . . . . . . . . . . . . . . . . . . . 8
1.3 Phương pháp diện tích . . . . . . . . . . . . . . . . . . . . . . . 13
1.3.1 Phương pháp diện tích . . . . . . . . . . . . . . . . . . . 13
1.3.2 Định lý Ptolemy và mở rộng . . . . . . . . . . . . . . . . 15
1.3.3 Đường thẳng Simson, đường thẳng Steiner . . . . . . . . 22
1.4 Định lý Ceva và Định lý Menelaus . . . . . . . . . . . . . . . . . 24
1.5 Bất đẳng thức Erdos¨-Mordell cho đa giác . . . . . . . . . . . . . 29
Chương 2. Phương pháp thể tích 35
2.1 Phương pháp thể tích . . . . . . . . . . . . . . . . . . . . . . . . 35
2.1.1 Phương pháp thể tích . . . . . . . . . . . . . . . . . . . . 35
2.1.2 Thể tích qua định thức . . . . . . . . . . . . . . . . . . . 36
2.2 Quan hệ bán kính mặt cầu ngoại-nội tiếp . . . . . . . . . . . . . . 42
Chương 3. Vận dụng giải bài thi học sinh giỏi 46
Kết luận 56
Tài liệu tham khảo 57
Tải tài liệu tại sividoc.com
Viết đề tài giá sinh viên – ZALO:0973.287.149-TEAMLUANVAN.COM
iv
Mở đầu
Hình học là một trong những phân nhánh Toán học xuất hiện sớm
nhất của nhân loại. Nhiệm vụ của hình học có thể được mô tả ngắn gọn
là trả lời cho các câu hỏi về hình dạng, kích thước, vị trí tương đối của
các hình khối, và các tính chất của không gian.
Các phương pháp giải toán trong hình học sơ cấp vốn vô cùng phong phú
và đa dạng. Điều đó hoàn toàn dễ hiểu vì hình học là một môn học truyền
thống trong nhà trường phổ thông và các trường đại học sư phạm. Dưới sự
hướng dẫn của PGS.TS. Đàm Văn Nhỉ, tác giả luận văn này có mục đích trình
bày về các phương pháp diện tích và thể tích trong hình học và những thảo
luận về các bài thi học sinh giỏi, nhằm làm phong phú lý thuyết vừa trình bày
và tạo cái nhìn đa chiều nhiều khía cạnh hơn cho giải toán hình học.
Ngoài các phần Mở đầu, Kết luận, Tài liệu tham khảo, Chỉ mục, nội
dung của luận văn được trình bày trong ba chương:
• Chương 1. Phương pháp diện tích. Chương này sẽ trình bày các kết quả
về phương pháp diện tích và ứng dụng vào giải toán hình học sơ cấp.
Các chủ đề sẽ được thảo luận là các Định lý Pythagore, Định lý Stewart,
Ceva, Menelaus và Bất đẳng thức Erdos¨-Mordell cho đa giác.
• Chương 2. Phương pháp thể tích. Chương này dành để trình bày về
phương pháp thể tích trong hình học, đặc biệt lưu ý đến thể tích qua định
thức và một quan hệ liên quan đến bán kính của mặt cầu nội và ngoại tiếp.
• Chương 3. Vận dụng giải bài thi học sinh giỏi. Chương này sẽ trình
bày lời giải của một số bài thi học sinh giỏi điển hình liên quan đến
các phương pháp diện tích và thể tích của Chương 1 và Chương 2.
Tải tài liệu tại sividoc.com
Viết đề tài giá sinh viên – ZALO:0973.287.149-TEAMLUANVAN.COM
v
Tác giả hi vọng rằng, bản luận văn này có thể làm tài liệu tham khảo
hữu ích cho những ai quan tâm đến Hình học sơ cấp và ứng dụng. Nó
sẽ có ích trong việc bồi dưỡng giáo viên, các học sinh khá giỏi, và những
ai quan tâm đến toán sơ cấp và muốn mở rộng nhãn quan nói chung.
Luận văn này đã được tác giả đầu tư nghiên cứu dưới sự hướng dẫn của
PGS.TS. Đàm Văn Nhỉ nhưng do nhiều lí do, luận văn chắc chắn sẽ không tránh
khỏi những thiếu sót. Tác giả mong muốn sẽ nhận được nhiều đóng góp của các
quý Thầy Cô, các anh chị em đồng nghiệp để luận văn này hoàn chỉnh hơn.
Thái Nguyên, ngày 20 tháng 5 năm 2017
Tác giả
Phạm Thị Thanh Thủy
Tải tài liệu tại sividoc.com
Viết đề tài giá sinh viên – ZALO:0973.287.149-TEAMLUANVAN.COM
2
Chương 1
Phương pháp diện tích
Hình học sơ cấp phát triển được là dựa trên nhiều kết quả của toán học cao cấp.
Ví dụ đơn giản là để có thể đo độ dài một đoạn thẳng hay diện tích một hình vuông
theo một đoạn thẳng được chọn làm đơn vị đo ta đã phải sử dụng kết quả về giới hạn,
liên tục và tích phân xác định. Vấn đề lý giải quá trình hình thành kết quả nào đấy qua
toán cao cấp là cần thiết và sẽ thường sử dụng tỷ số các đoạn thẳng hoặc diện tích
trong chứng minh. Từ đó ta có thể phát hiện ra nhiều kết quả mới nữa.
1.1 Định lý Pythagore
1.1.1 Tam giác vuông
Dựa vào tiên đề số đo độ dài một đoạn thẳng và nhiều kết quả trong
lý thuyết về giới hạn ta sẽ sử dụng mệnh đề dưới đây để tính diện tích
một hình vuông cạnh a.
Mệnh đề 1.1.1. Diện tích hình vuông ABCD với độ dài cạnh AB = a (đơn
vị dài) đúng bằng a
2
đơn vị diện tích.
Chứng minh. Dựng hệ tọa độ Axy : A(0, 0), B(a, 0), C(a, a), D(0, a). Khi đó
a
a
SABCD = a DX = ax = a
2
.
0
0
Như vậy, diện tích hình vuông ABCD cạnh a đúng bằng a2 đơn vị diện tích.
Mệnh đề 1.1.2. Tam giác vuông ABC có độ dài cạnh a = BC, b = CA, c =
AB và ∠BAC = 900
. Hạ đường cao AH ⊥ BC. Đặt h = AH và diện tích
tam giác qua S. Khi đó ta có các đồng nhất thức
Tải tài liệu tại sividoc.com
Viết đề tài giá sinh viên – ZALO:0973.287.149-TEAMLUANVAN.COM
3
(1) a
2
= b
2
+ c
2
[Pythagore].
(2) b2
= a.BH và c2
= a.CH .
(3) a.h = b.c, h2
= BH .CH và
1
=
1
+
1
.
h2 b2 c2
(4) 2S = a.h = b.c.
Chứng minh. Dựng hình vuông ABCD với cạnh AB = a. Dựng vào bên trong
hình vuông ABCD bốn tam giác vuông bằng nhau ABA1, BCB1, CDC1 và
DAD1 bằng tam giác vuông ABA1. Khi đó ta có hình vuông A1B1C1D1 với
A1B1 = |b − c|. Ta có SABCD bằng tổng diện tích bốn tam giác vuông ABA1,
BCB1, CDC1, DAD1 và hình vuông A1B1C1D1. Vậy, ta có hệ thức
a2
= 4.
b
2
.c
+ (b − c)2
= b2
+ c2
.
Các kết quả còn lại là hiển nhiên.
Hệ quả 1.1.1. Với biểu diễn b = a SIN B, c = a COS B trong tam giác vuông
ABC ta có
SIN
2
B + COS
2
B = 1.
Chứng minh. Từ a
2
= b
2
+ c
2
= a
2
(SIN
2
B + COS
2
B) theo Định lý 1.1.2 ta
nhận được hệ thức SIN
2
B + COS
2
B = 1.
Hệ quả 1.1.2. Trong mặt phẳng cho hai đường thẳng d và d′
. Lấy A, B thuộc
d và C, D thuôc d′. Khi đó d ⊥ d′ nếu và chỉ nếu AC2 + BD2 = AD2 + BC2.
Chứng minh. Kết quả được suy ra từ Định lý Pythagore.
Tải tài liệu tại sividoc.com
Viết đề tài giá sinh viên – ZALO:0973.287.149-TEAMLUANVAN.COM
4
Hệ quả 1.1.3 (Steiner). Cho tam giác ABC. Lấy M, N, P thuộc đường thẳng
BC, CA, AB, tương ứng. Dựng đường thẳng mM ⊥ BC, nN ⊥ CA và pP ⊥
AB. Ba đường thẳng mM, nN và pP đồng quy tại một điểm khi và chỉ khi
MC
2
+ NA
2
+ PB
2
= MB
2
+ PA
2
+ NC
2
.
Chứng minh. Kết quả được suy ra trực tiếp từ Định lý Pythagore.
Ví dụ 1.1.1. Cho tam giác ABC. Gọi M, N, P là trung điểm các cạnh BC,CA,
AB, tương ứng. Dựng ra phía ngoài tam giác ABC các hình vuông BCA1A2 ,
CAB1B2, ABC1C2. Gọi A0 , B0,C0 là trung điểm A1A2, B1B2,C1C2, tương ứng
và dựng các đường thẳng mM ⊥ B0C0, nN ⊥ C0A0 và pP ⊥ A0B0. Chứng
minh rằng, ba đường thẳng mM, nN, pP đồng quy tại một điểm.
Bài giải. Đặt α = ∠ABC0 và a = BC, b = CA, c = AB, S = SABC . Theo Định lý
Côsin ta có MC
2
=4c
2
+ b
2
+ 2S và MB
2
=4b
2
+ c
2
+ 2S. Ta nhận được
0 4 0 4
MB0
2
− MC0
2
=
3
(b
2
− c
2
).
4
Hoàn toàn tương tự, ta cũng có hai kết quả
NC0
2
− NA0
2
=
3
(c
2
− a
2
) và PA0
2
− PB0
2
=
3
(a
2
− b
2
).
4 4
Ta có
MB
2
0 − MC0
2
+ NC0
2
− NA
2
0 + PA
2
0 − PB
2
0 = 0.
Vậy, ba đường thẳng mM, nN, pP đồng quy tại một điểm theo hệ quả trên.
Ví dụ 1.1.2. Cho tam giác ABC và các đường cao AA1, BB1,CC1. Dựng các đường
thẳng AM ⊥ B1C1, BN ⊥ C1A1 và CP ⊥ A1B1. Chứng minh AM, BN,CP đồng quy.
Tải tài liệu tại sividoc.com
Viết đề tài giá sinh viên – ZALO:0973.287.149-TEAMLUANVAN.COM
5
Bài giải. Ta có MB
2
1 − MC1
2
= AB
2
1 − AC1
2
= (AB
2
− BB
2
1) − (AC
2
−
CC1
2
). Hoàn toàn tương tự ta cũng có NC1
2
− NA
2
1 và PA
2
1 − PB
2
1.
Từ đó suy ra AM, BN,CP đồng quy theo ví dụ trên.
Ví dụ 1.1.3. Tam giác ABC có ∠A = 900 và đường cao AH = h. Giả sử M
là một điểm tùy ý trong tam giác và khoảng cách từ M đến BC,CA, AB là
x, y, z. Xác định giá trị nhỏ nhất của tổng T = x
2
+ y
2
+ z
2
.
Hạ MI ⊥ AH , I ∈ AH . Vì y
2
+ z
2
= MA
2
nên ta có thể đánh giá như sau:
T = MA
2
+ x
2
AI
2
+ x
2
= (h − x)
2
+ x
2
= 2x
2
− 2hx + h
2
.
Vậy
T = 2 x −
h 2
+
h2 h2
2 2 2
và Tnn =h
2
khi x = h hay M là trung điểm đường cao AH .
2 2
Ví dụ 1.1.4 (Bulgarian MO and TST 2013). Cho tam giác nhọn ABC. Lấy các điểm M,
N, P thuộc các cạnh BC,CA, AB, tương ứng, sao cho tam giác APN, BMP, CN M nhọn
và ký hiệu Ha, Hb, Hc là trực tâm của chúng. Chứng minh rằng, nếu ba đường thẳng
AHa, BHb,CHc đồng quy thì ba đường thẳng MHa, N Hb , PHc cũng đồng quy.
Bài giải. Ký hiệu A1, B1,C1 là hình chiếu vuông góc của A, B,C lên N P,
PM, MN, tương ứng. Vì đường thẳng AHa, BHb,CHc đồng quy tại một
điểm nên ta có hệ thức
(NC1
2
− MC1
2
) + (MB
2
1 − PB
2
1) + (PA
2
1 − NA
2
1) = 0.
Vậy
0 = CN
2
−CM
2
+ BM
2
− BP
2
+ AP
2
− AN
2
Bài giải.
Tải tài liệu tại sividoc.com
Viết đề tài giá sinh viên – ZALO:0973.287.149-TEAMLUANVAN.COM
6
= CN
2
− AN
2
+ AP
2
− BP
2
+ BM
2
−CM
2
Ta suy ra ba đường thẳng vuông góc hạ từ M, N, P xuống cạnh BC,CA, AB, tương
ứng, đồng quy tại điểm K. Vì các tứ giác K MHcN và K MHbP là những hình bình
hành nên tứ giác PHb HcN cũng là một hình bình hành. Khi đó hai đường chéo
PHc, N Hb cắt nhau tại điểm T, trung điểm mỗi đường. Tương tự, MHa cũng nhận
T làm trung điểm hay ba đường thẳng MHa, N Hb , PHc đồng quy tại điểm T.
Ví dụ 1.1.5. Với hai số dương a, b xét ABCD độ dài cạnh AB = a + b. Lấy M
thuộc cạnh AB với AM = a, BM = b. Giả sử N chạy trên cạnh BC và P thuộc
cạnh AD sao cho N M ⊥ MP. Xác định giá trị nhỏ nhất của tam giác MN P.
Bài giải. và
suy ra của
SMN P
Đặt x = BN, y = AP. Từ điều kiện MN ⊥ MP ta suy ra MN
2
+MP
2
= N P
2
xy = ab. Khi đó SMN P = ax + by √ = ab. Vậy, giá trị nhỏ nhất
abxy
2
bằng ab khi x = b, y = a.
Ví dụ 1.1.6. Coi bốn xã như bốn đỉnh hình vuông ABCD với độ dài cạnh
AB = 10 km. Chứng minh rằng, có thể xây mạng đường nối bốn xã với
tổng độ dài nhỏ hơn 28 km.
Bài giải. Gọi O là tâm hình vuông. Lấy I, J là trung điểm đoạn AO, BO, tương ứng.
Mạng đường nối bốn xã là AI, IJ, JB, ID, JC. Tổng độ dài mạng đường này
√ √
bằng d = 5 + 5 2 + 5 10 < 28, (đúng).
1.1.2 Hệ tọa độ Descarte vuông góc
Trước khi trình bày hệ tọa độ Carte vuông góc chúng ta sẽ chứng
minh lại Định lý Thales và nhắc lại khái niệm số đo đoạn thẳng. Giả sử ta
chọn đoạn AB làm đơn vị đo. Theo tiên đề về số đo, ứng với mỗi đoạn
thẳng MN luôn có một số thực x để MN = |x|.AB và MN = x.AB. Từ đó ta
định nghĩa tỷ số giữa hai đoạn thẳng: Giả sử MN = |x|.AB, PQ = |y|.AB
với y = 0. Tỷ số giữa hai đoạn được định nghĩa như sau:
MN := |x| và MN := x .
PQ |y| PQ y
Định lí 1.1.1 (Thales’s Theorem). Với hai đường thẳng d, d′
và ba đường thẳng
song song a, b, c cắt d, d
′
tại A, B,C và A
′
, B
′
,C
′
, tương ứng, ta luôn có
AB
=
A′B′
.
BC B′
C′
Tải tài liệu tại sividoc.com
Viết đề tài giá sinh viên – ZALO:0973.287.149-TEAMLUANVAN.COM
7
Chứng minh. Dựng Aa1 d
′
và B1
AB1 = A
′
B
′
, B1C1 = B
′
C
′
. Như vậy
= Aa1 × b,C1 = Aa1 × c. Khi đó ta nhận được
AB
=
AB
1 =
A′B′
.
BCB1C1B
′
C
′
Ví dụ 1.1.7. Cho hình vuông ABCD. Lấy điểm K thuộc cạnh AB và N thuộc cạnh
AD thỏa mãn AK.AN = 2BK.DN. Giả sử các đoạn thẳng CK,CN cắt BD tại điểm
L, M. Chứng minh rằng, năm điểm A, K, L, M, N cùng nằm trên một đường tròn.
Bài giải. Ta chỉ cần xét hình vuông với độ dài cạnh 1. Đặt a = AK, b =
AN. Từ AK.AN = 2BK.DN ta suy ra ab = 2(1 − a)(1 − b) và ta nhận được
2 − 2a − 2b + ab = 0.
Dựa vào Định lý Thales ta dễ dàng tính được
BL = √ (1 − a) , √ .
2 BM = 2
2 − a 2 − b
Vậy
BL.BM =
(2
2(1 − a)
b)
= 1
−
a = BA.BK
−
a)(2
−
và suy ra tứ giác AK LM nội tiếp trong một đường tròn. Tương tự, tứ giác
ALMN cũng nội tiếp trong một đường tròn. Từ đó suy ra năm điểm A, K,
L, M, N cùng nằm trên một đường tròn.
Trong mặt phẳng (P), ta xét hai đưòng thẳng x
′
x, y
′
y với x
′
x ⊥ y
′
y và O =
x
′
x × y
′
y. Với điểm gốc O và định hướng đường thẳng x
′
x và y
′
y để biến
mỗi đường thành một trục. Khi đó ta đã có một Hệ tọa độ Carte vuông
góc Oxy với gốc tọa độ O và trục hoành x
′
x, trục tung y
′
y. Mặt phẳng (P)
với Hệ tọa độ Carte vuông góc Oxy được gọi đơn giản là hệ tọa độ Oxy.
Mỗi điểm M thuộc trục x
′
x được gắn với đúng một số thực a ∈ R và viết
OM = a. Mỗi điểm N thuộc trục y
′
y được gắn với đúng một số thực b ∈ R
và viết ON = b. Đây là những số đo đại số.
Giả sử điểm A tùy ý thuộc mặt phẳng tọa độ Oxy. Hạ AM ⊥ x′
x, M ∈ x′
x
và hạ AN ⊥ y
′
y, N ∈ y
′
y. Dễ dàng thấy rằng, mỗi điểm A thuộc mặt phẳng
(P) tương ứng một-một với một cặp số thực (a, b), trong đó OM = a, ON
= b. Cặp số thực (a, b) được gọi là tọa độ điểm A và viết A(a, b).
Tải tài liệu tại sividoc.com
Viết đề tài giá sinh viên – ZALO:0973.287.149-TEAMLUANVAN.COM
8
Khoảng cách: Giả sử điểm A(x1, y1) và điểm B(x2, y2). Theo Định lý
Pythagore, khoảng cách giữa hai điểm A và B hay độ dài đoạn AB bằng
AB = (x2 − x1)
2
+ (y2 − y1)
2
.
Chia đoạn theo tỷ số: Giả sử điểm A(x1, y1) và điểm B(x2, y2). Ta xác định điểm I
thuộc đoạn AB sao cho
IA
=
γ2
. Ta chỉ cần xét trường hợp AB không song song
BI γ1
với trục tọa độ nào. Hạ AA1, IM, BB1 ⊥ x
′
x. Gọi I(x, y). Khi đó
γ2
γ1
Giải ra được
Tương tự, ta cũng có
=
IA
=
MA
1 =
x
1 −
x
.
− x2BIB1Mx
x =γ1
x
1 + γ2
x
2 .
γ1+γ2
y =γ1
y
1 + γ2
y
2 .
γ1+γ2
Như vậy, tọa độ điểm chia
I γ1 x1 + γ2x2 ,γ1y1 + γ2 y2 .
γ1+γ2 γ1+γ2
1.2 Định lý Stewart
Tiếp theo, ta chứng minh Định lý Stewart sau đây.
Định lí 1.2.1 (Stewart). Với ba điểm M, N, P thẳng hàng và mọi điểm I có hệ thức
T = IM2
.NP + IN2
.PM + IP2
.MN = −MN.NP.PM.
Chứng minh. Dựng hệ tọa độ Oxy sao cho M(0, 0), N(b, 0), P(c, 0). Giả
sử I(x, y). Khi đó
T = (x
2
+ y
2
)(c − b) + [(x − b)
2
+ y
2
](−c) + [(x − c)
2
+ y
2
]b.
Dễ dàng suy ra hệ thức T = bc(c − b) = −MN.N P.PM. Hệ quả 1.2.1.
Cho tam giác ABC với BC = a,CA = b, AB = c. Gọi M là trung điểm cạnh
BC và đặt ma = AM. Gọi N là chân đường phân giác trong AN của góc
∠A và đặt ℓ = AN; Gọi L là chân đường phân giác ngoài AL của góc ∠A
khi b = c và đặt la = AL. Khi đó ta có các kết quả dưới đây:
Tải tài liệu tại sividoc.com
Viết đề tài giá sinh viên – ZALO:0973.287.149-TEAMLUANVAN.COM
9
(1) m2 =
2b2 + 2c2 − a2
.
a 4
(2)ℓ2 =
bc[(b + c)2 − a2]
.
a
(b + c)
2
(3)l2 =
bc[a2 − (c − b)2]
.
a
(c − b)
2
Chứng minh. (1) Với trung điểm M của cạnh BC ta luôn có
m2 = 2b
2
+ 2c
2
− a
2
a 4
theo Định lý 1.2.1, ở đó AM = ma.
(2) Khi AN = ℓa là phân giác trong của góc ∠A thì
BN = ac và CN = ab .
b + c b + c
Theo Định lý 1.2.1 có
ℓ2a =
bc[(b + c)2 − a2]
.
(3) Khi b = c và AL = la là phân giác ngoài của góc ∠A thì
BL = ac và CL = ab .
c − b c − b
Theo Định lý 1.2.1 có l2 = bc[a2
− (c − b)2
] .
a
(c − b)
2
Hệ quả 1.2.2 (Steiner-Lehmus). Cho tam giác ABC với BC = a, CA = b,
AB = c. Giả sử ℓa, ℓb, ℓc là độ dài các đường phân giác trong của ΔABC.
Nếu ℓa = ℓb thì tam giác ABC cân.
Chứng minh. Ta biết
ℓ
2
= bc[(b + c)
2
− a
2
] và ℓ
2
=ca[(a + c)
2
− b
2
] .
a
(b + c)2 b
(a + c)2
Nếu ℓa = ℓb thì
bc[(b + c)
2
− a
2
] ca[(a + c)
2
− b
2
]
= .
(b + c)2
(a + c)2
Ta có a = b hay tam giác ABC cân với AB = AC.
(b + c)2
Tải tài liệu tại sividoc.com
Viết đề tài giá sinh viên – ZALO:0973.287.149-TEAMLUANVAN.COM
10
Ví dụ 1.2.1. Cho tam giác ABC với BC = a,CA = b, AB = c. Giả sử ha, hb,
hc, ℓa, ℓb, ℓc và ma, mb, mc là độ dài các đường cao, các đường phân giác
trong và các đường trung tuyến của ΔABC. Ta có các bất đẳng thức
(1) ha ℓa ma. Dấu = xảy ra khi b = c.
ℓa ℓb ℓc
√
+ √ +
(2) + + 2 bc ca
ma mb mc b + cc + a
a = b = c.
√
ab
.Dấu = xảy ra khi và chỉ khi a
+ b
Bài giải. (1) Hiển nhiên ha ℓa. Vì
ℓa
2
=
4bc (b + c)
2
− a
2
4bc
1
ma
2 (b + c)
2
2(b
2
+ c
2
) − a
2
(b + c)
2
theo Chú ý 1.2.1 nên ℓa ma. Hiển nhiên, dấu bằng xảy ra khi và chỉ khi b = c.
(2) Bởi có
√ √ √
ℓa
ℓ
b 2 , ℓc 2 ab
2 bc , ca
ma b + cmb c + amc a + b
nên
ℓb ℓc √ √ √
ℓa + + ab .
+ + 2 bc ca
ma mb mc b + cc + aa + b
Dấu = xảy ra khi và chỉ khi a = b = c.
Ví dụ 1.2.2. Cho ΔABC với BC = a,CA = b, AB = c, 2 p = a + b + c, bán kính các
đường tròn bàng tiếp là ra, rb, rc. Giả sử ℓa, ℓb, ℓc và ma, mb, mc là độ dài các đường
phân giác trong và các đường trung tuyến của ΔABC. Khi đó ta có các kết quả sau:
(1) m
2
+ m
2
+ m
2
= 3 a2
+ b2
+ c2
, và
a b c 4
b + c
2
ℓa
2
c + a
2
ℓb
2
a + b
2
ℓc
2
2
+ + = 4 p.
bc ca ab
(2) ℓ
2
+ ℓ
2
+ ℓ
2
p
2
.
a b c
(3) r2
+ r2
+ r2
m2
+ m2
+ m2
+(a − b)2
+ (b − c)2
+ (c − a)2
.
a b c a b c 4
(4) r2 + r
2
+ r
2
+ ℓ
2
+ℓ
2
+ℓ
2
m
2
+ m
2
+ m
2
+ p
2
.
a b c a b c a b c
Chú ý rằng các bất đẳng thức này sẽ trở thành đẳng thức khi và chỉ khi a = b = c.
Tải tài liệu tại sividoc.com
Viết đề tài giá sinh viên – ZALO:0973.287.149-TEAMLUANVAN.COM
11
Bài giải. (1) suy ra từ Hệ quả 1.2.1 và (2) được suy ra từ (1).
(3) Đặt x = p − a, y = p − b, z = p − c với p =
a + b + c
. Biến đổi được
2
ra
2
+ rb
2
+ rc
2
= x + y + z
yz
+
zx
+
xy
y
x z
= xy + yz + zx + x 2 y
+z
+ y 2 z
+
x
+ z 2 x
+y
z y x z y x
xy + yz + zx + 2x
2
+ 2y
2
+ 2z
2
= 3 a
2
+ b
2
+ c
2
+ a − b
2
+ b − c
2
+ c − a
2
.
4
Do vậy có được
r
2
+ r
2
+ r
2
m
2
+ m
2
+ m
2
+(a − b)
2
+ (b − c)
2
+ (c − a)
2
.
a b c a b c 4
(4) Ta có
ℓ
2
= bc[(b + c)2
− a2
] = bc a
2 bc bc a2 .
− b + c 2 − 4
a (b + c)
2
Như vậy
ℓa
2
+ ℓb
2
+ ℓc
2
bc + ca + ab −
a2
+ b2
+ c2
.
4
Kết hợp với bất đẳng thức (2) được
ra
2
+ rb
2
+ rc
2
+ ℓa
2
+ ℓb
2
+ ℓc
2
ma
2
+ mb
2
+ mc
2
+ p2
.
Ví dụ 1.2.3. Cho ΔABC với diện tích S và BC = a,CA = b, AB = c. Giả sử ha, hb, hc và
ma, mb, mc là độ dài các đường cao và các đường trung tuyến của ΔABC. Khi đó
ta có bất đẳng thức (m2 + m
2
+ m
2
)(h
2
+ h
2
+ h
2
) 27S
2
.
a b ca b c
Bài giải. Vì
ma
2
+ mb
2
+ mc
2
=
3
(a
2
+ b
2
+ c
2
)
4
nên
(ma
2
+ mb
2
+ mc
2
)(ha
2
+ hb
2
+ hc
2
)
3
3 3 (abc)
2
3 3
(hahbhc)
2
= 27S
2
.
4
Tải tài liệu tại sividoc.com
Viết đề tài giá sinh viên – ZALO:0973.287.149-TEAMLUANVAN.COM
12
Ví dụ 1.2.4 (Garfunkel). Tam giác ABC có độ dài ba đường trung tuyến ma, mb,
mc. Gọi G là trọng tâm tam giác ABC và các đường thẳng AG, BG,CG cắt đường
tròn ngoại tiếp tam giác tại A′, B′,C′, tương ứng. Khi đó ta có bất đẳng thức
A′
A +B′
B +C′
C 4.
m
a
m
b
m
c
Bài giải. Ký hiệu a, b, c cho độ dài ba cạnh ΔABC. Khi đó ta dễ dàng có
các hệ thức
GA
=
2b
2
+ 2c
2
− a
2
GA′
a2
+ b2
+ c2
GB
=
2c2
+ 2a2
− b2
GB′
a2
+ b2
+ c2
GC
=
2a2
+ 2b2
− c2
.
GC
′
a2
+ b2
+ c2
Cộng ba hệ thức, ta sẽ nhận được hệ thức
GA
+
GB
+
GC
= 3
GA
′
GB
′
GC
′
và suy ra bất đẳng thức
GB′
GC′
GA′
+ + 3.
GA GB GC
Vì
AA′
= 1 +
GA′
GA GA
nên
3.AA′
= 1+ GA
′
.
2 maGA
Tương tự, ta cũng có
3 .BB
′
= 1 +GB
′
và 3. CC
′
= 1 +GC
′
.
2 mb GB 2
m
c GC
Cộng ba hệ thức này ta nhận được bất đẳng thức
A
′
A + B
′
B +C
′
C 4.
m
a mb
m
c
Ví dụ 1.2.5. Tam giác ABC có độ dài ba đường trung tuyến ma, mb, mc
và bán kính đường tròn ngoại tiếp R. Khi đó ta có bất đẳng thức
1 + 1 + 1 2 .
m
a
m
b
m
c R
Tải tài liệu tại sividoc.com
Viết đề tài giá sinh viên – ZALO:0973.287.149-TEAMLUANVAN.COM
13
Bài giải. Vì AA
′
, BB
′
,CC
′
2R nên
2R +2R +2R A′
A +B′
B + C′
C 4.
ma
m
b
m
c ma
m
b
m
c
Từ đó suy ra bất đẳng thức
1 + 1 + 1 2 .
m
a
m
b
m
c R
Ta có điều phải chứng minh.
1.3 Phương pháp diện tích
1.3.1 Phương pháp diện tích
Phương pháp diện tích là phương pháp giải bài toán hình trong mặt
phẳng qua diện tích của một miền phẳng nào đó. Để giải bài toán đã cho
ta chọn một miền phẳng (D) với diện tích S. Sau đó đem chia miền (D) ra
thành nhiều miền nhỏ một cách thích hợp và tính diện tích S1, . . . , Sr
của các miền nhỏ ấy. Từ S = S1 + · · · + Sr suy ra lời giải bài toán.
Mệnh đề 1.3.1. Cho tam giác ABC với độ dài ba cạnh a, b, c với độ dài các
đường cao ha, hb, hc; độ dài các bán kính đường tròn ngoại, nội và bàng tiếp
là R, r, r1, r2, r3. Khi đó diện tích S của tam giác ABC được xác định qua
(1) S =
ah
2
a
=
bh
2
b
=
ch
2
c
.
(2) S = bcSIN A = ca SIN B = ab SIN C .
222
(3) S =
abc
.
4R
(4) S = 2R
2
SIN A SIN B SIN C.
(5) S =
(a + b + c)r
=
pr. 2
(6) S = (p − a)r1 = (p − b)r2 = (p − c)r3.
(7) S =p(p − a)(p − b)(p − c).
Mệnh đề 1.3.2. Hình tròn bán kính R có diện tích S = π .R
2
đơn vị diện tích.
Tải tài liệu tại sividoc.com
Viết đề tài giá sinh viên – ZALO:0973.287.149-TEAMLUANVAN.COM
14
Chứng minh. Giả sử đường tròn x2
+ y2
= R2
trong mặt phẳng Oxy. Ta có diện tích
R √ 2
π /2
2 2
(đơn vị diện tích).
2 2
t DT = π .R
S = 4 0 R − x DX=4R 0 SIN
Mệnh đề 1.3.3. Cho tứ giác lồi ABCD với độ dài bốn cạnh AB = a, BC = b,
CD = c và DA = d. Khi đó ta có công thức tính diện tích S của tứ giác ABCD :
4(ab − cd)2
− (a2
+ b2
− c2
− d2
)2
+ 16abcd SIN
2 ∠B + ∠D
2
S = .
4
Khi tứ giác ABCD ngoại tiếp đường tròn thì
S =
√
abcd SIN
∠B + ∠D
.
2
Chứng minh. Ta có 2S = 2SABC + 2SADC = ab SIN ∠B + cd SIN ∠D.
Từ a2
+ b2
− 2ab COS B = AC2
= c2
+ d2
− 2cd COS D ta suy ra
2ab COS B − 2cd COS D = a2
+ b2
− c2
− d2
. Biến đổi biểu thức sau:
(a
2
+ b
2
− c
2
− d
2
)
2
+ 16S
2
= 4(ab COS B − cd COS D)
2
+4(ab SIN ∠B + cd SIN ∠D)2
= 4(a2
b2
+ c2
d2
) − 8abcd COS(∠B + ∠D)
= 4(ab − cd)
2
+ 16abcd SIN
∠B+∠D
2
và
4(ab − cd)
2
− (a
2
+ b
2
− c
2
− d
2
)
2
+ 16abcd SIN
2
∠B+∠D
S = 2 .
4
Khi tứ giác ABCD ngoại tiếp đường tròn thì 4(ab − cd)
2
− (a
2
+ b
2
− c
2
− d
2
)
2
= 0 và
ta nhận được S =
√
abcd SIN
∠B + ∠D
.
2
Mệnh đề 1.3.4 (Bretschneider). Cho tứ giác lồi ABCD với độ dài bốn cạnh AB = a,
BC = b, CD = c, DA = d và AC = x, BD = y. Khi đó ta có công thức tính diện tích
S của tứ giác ABCD :
S = 4x2
y2
− (a2
− b2
+ c2
− d2
)2
.
4
Tải tài liệu tại sividoc.com
Viết đề tài giá sinh viên – ZALO:0973.287.149-TEAMLUANVAN.COM
15
Chứng minh. Ta có 2S = xy SIN α , ở đó α là góc giữa AC và BD. Dễ thấy
2xy COS α = |a2
− b2
+ c2
− d2
|.
Vậy
S = 4x
2
y
2
− (a
2
− b
2
+ c
2
− d
2
)
2
.
4
Mệnh đề 1.3.5. Cho tứ giác lồi ABCD nội tiếp trong một đường tròn với độ dài
bốn cạnh AB = a, BC = b, CD = c và DA = d. Khi đó, với p =
a + b + c + d
ta
có 2
công thức tính diện tích tứ giác:
S= (p − a)(p − b)(p − c)(p − d).
Chứng minh. Vì 2S = (ab + cd) SIN B và
COS B = a
2
+ b
2
− c
2
− d
2
2(ab + cd)
nên ta có biến đổi sau đây để tính SIN B qua độ dài các cạnh và chu vi:
2 2 2 2 2
SIN
2
B= 1 COS
2
B= 1 a + b − c − d
− − 2(ab + cd)
= [(a + b)
2
− (c − d)
2
][(c + d)
2
− (a − b)
2
]
4(ab + cd)
2
4(p − a)(p − b)(p − c)(p − d)
=
và nhận được
(ab + cd)
2
(ab + cd) SIN B = 2 (p − a)(p − b)(p − c)(p − d).
Từ đây ta có được công thức tính diện tích S = (p − a)(p − b)(p − c)(p
− d).
1.3.2 Định lý Ptolemy và mở rộng
Định lí 1.3.1. [Ptolemy] Cho tứ giác lồi ABCD nội tiếp trong một đường tròn. Khi
đó ta luôn có AC.DB = AB.DC + BC.DA hay AB + BC = AC .
DA.DC DB.DC DA.DC
Tải tài liệu tại sividoc.com
Viết đề tài giá sinh viên – ZALO:0973.287.149-TEAMLUANVAN.COM
16
Chứng minh. Đặt a = AB, b = BC, c = CD, d = DA và x = AC, y = BD. Tứ
giác lồi ABCD nội tiếp trong một đường tròn (T ). Kẻ BE AC với E thuộc
(T ). Khi đó 2SABCD = 2SAECD = (ac + bd) SIN ∠E AD. Vì 2SABCD = xy SIN ∠E AD,
ở đó α là góc giữa AC và BD. Vậy ac + bd = xy. Chia hai vế cho DA.DB.DC ta
nhận được hệ thức dạng phân thức AB + BC = AC .
DA.DC DB.DC DA.DC
Nhận xét 1.3.1. (i) Nếu tứ giác ABCD lõm và nội tiếp trong một đường
tròn với độ dài bốn cạnh AB = a, BC = b, CD = c, DA = d và độ dài hai
đường chéo AC = x, BD = y thì luôn có ac − bd = xy khi tứ giác ACBD lồi
hoặc bd − ac = xy khi tứ giác ABDC lồi. Do vậy, với việc chọn dấu ± một
cách thích hợp ta luôn có ac ± bd ± xy = 0.
(ii) Biểu diễn Đồng nhất thức Ptolemy dạng phân thức là dựa vào phương
pháp phân tích phân thức thành tổng đại số các phân thức đơn giản.
Định lí 1.3.2. Với số nguyên n 3, đa giác lồi A1 . . . AnM nội tiếp trong
một đường tròn có đồng nhất thức
A1A2
+
A
2
A
3
+···+
A
n−1
A
n
=
A
n
A
1
.
MA1.MA2 MA2.MA3 MAn−1.MAn MAn.MA1
Chứng minh. Quy nạp theo n. Với n = 3, kết luận đúng theo Định lý 1.3.1.
Giả sử kết luận đúng cho n. Khi đó có đồng nhất thức
A
1
A
2
+
A2A3
+···+
A
n−1
A
n
=
AnA1
.
MA1.MA2 MA2.MA3 MAn−1.MAn MAn.MA1
Với đa giác lồi A1 . . . AnAn+1M nội tiếp trong một đường tròn, có
A1A2
+
A
2
A
3
+···+
A
n−1
A
n
=
A
n
A
1
MA1.MA2 MA2.MA3 MAn−1.MAn MAn.MA1
Tải tài liệu tại sividoc.com
Viết đề tài giá sinh viên – ZALO:0973.287.149-TEAMLUANVAN.COM
17
cho đa giác lồi A1 . . . AnM và cho tứ giác A1AnAn+1M ta có hệ thức
A
1
A
n
+
A
n
A
n+1
=
A
1
A
n+1
.
MA1.MAn MAn.MAn+1 MA1.MAn+1
Cộng hai hệ thức này lại, ta nhận được Đồng nhất thức Ptolemy tổng quát cho đa
giác lồi
A1A2
+
A2A3
+···+
A
n
A
n+1
=
A
n+1
A
1
.
MA1.MA2 MA2.MA3 MAn.MAn+1 MAn+1.MA1
Ví dụ 1.3.1. Tam giác ABC có bán kính đường tròn ngoại tiếp và nội tiếp R, r. Giả sử
ba phân giác trong các góc của tam giác cắt đường tròn ngoại tiếp ở A1, B1,C1.
Chứng minh rằng AA1.BB1.CC1 16R2
r.
Bài giải. Theo Bổ đề 1.3.1 ta có
AA1 =
A1B(b + c)
=b + c.2R SIN
A
2
a a
vì A1B = A1C và bằng 2R SIN
A . Ta cũng có hệ thức tương tự cho BB1,CC1. Vậy
2
AA1.BB1.CC1 = (b + c)(c + a)(a + b) 8R
3
SIN A .SIN
B .SIN
C
2
abc 2 2
8abc .2R
2
.4R SIN A r. SIN B .SIN C = 16R
2
r.
abc 2 2 2
Tóm lại, ta nhận được AA1.BB1.CC1 16R
2
r.
Ví dụ 1.3.2. Tam giác ABC có a = BC, b = CA, c = AB và bán kính
đường tròn ngoại tiếp R. Giả sử phân giác trong các góc của tam giác
cắt đường tròn ngoại tiếp ở A1, B1,C1, tương ứng. Chứng minh rằng
(1) AA1 + BB1 +CC1 > a + b + c.
(2)
S
A1B1C1
S
ABC.
(3) Gọi độ dài ba phân giác trong là ℓa, ℓb, ℓc. Khi đó
ℓa
+
ℓb
+
ℓc
3 [APMO1997].
AA1 SIN
2
A BB1 SIN
2
B
2
CC1 SIN C
Bài giải. (1) Theo Định lý cosin ta có 2c
.AA
1
COS
2
A1B2 = c
2
+ AA1
2
− A
2 2 2 A
A1C = b + AA1 − 2b.AA1 COS .
2
Tải tài liệu tại sividoc.com
Viết đề tài giá sinh viên – ZALO:0973.287.149-TEAMLUANVAN.COM
18
Vì A1B = A1C nên c
2
− b
2
= 2(c − b).AA1 COS
A
. Nếu b= c thì
2
AA1 = 2Rvà b + c < 4R.
Nếu b = c thì b + c = 2AA1 COS
A
. Tương tự, ta cũng có
2
c + a = 2BB1 COS B và a + b = 2CC1 COS C .
2
2
Do đó a + b + c < AA1 + BB1 +CC1.
(2) Ta có
S
A1B1C1 A + B B +C C + A
S
ABC
= SIN SIN SIN SIN A SIN B SINC =
2R
2
2 2 2 2R
2
và suy ra bất đẳng thức SA1 B1C1
S
ABC.
(3) Theo Định lý sin ta có
ℓa = c SIN B và AA1 =c SIN B+A/2 .
SIN C + A/2 SIN C
Vậy
ℓa = SIN B SIN C SIN B SIN C.
AA1 SIN
2
A SIN
2
A SIN B+A/2 SIN C + A/2 SIN
2
A
Đánh giá còn lại tương tự. Từ đó có
ℓa +
ℓ
b + ℓc SIN B SIN C +SIN C SIN A +SIN A SIN B
AA1 SIN
2
A BB1 SIN
2
BCC1 SIN
2
C SIN
2
A SIN
2
B SIN
2
C
3
theo Bất đẳng thức AM-GM.
Ví dụ 1.3.3. Ký hiệu n(r) là số điểm nguyên thuộc đường tròn bán kính r > 1.
√
Chứng minh n(r) < 2π 3
r
2
.
Bài giải. Giả sử đường tròn (ℓ) bán kính r > 1 chứa n điểm với tọa độ nguyên. Ta
phải chỉ ra n(r) < 2π √ r2 . Vì r > 1 và 2π > 6 nên 2π √ r2 > 6. Vậy, ta có thể giả
3 3
thiết n 7 bởi vì n < 7 bất đẳng thức luôn đúng.
Ký hiệu điểm nguyên thuộc đường tròn (ℓ) theo thứ tự P1, P2, . . . , Pn, (theo chiều
⌢ ⌢ ⌢
quay của kim đồng hồ). Khi đó các cung P1P3, P2P4, Pn−1P1 và PnP2 phủ đường
tròn hai lần. Do vậy, tổng các số đo cung của các cung này bằng 4π . Từ đó suy
Tải tài liệu tại sividoc.com
Viết đề tài giá sinh viên – ZALO:0973.287.149-TEAMLUANVAN.COM
19
ra tồn tại ít nhất một cung không vượt quá4π
⌢
, chẳn hạn cung P1P3. Xét tam giác
4π
n
P1P2P3 nội tiếp trong cung với số đo . Vì n 7 nên cung này có thể coi như
nhỏ hơn một phần tư đường tròn. Diện
n
tích tam giác P1P2P3 lớn nhất khi P1, P3 là
hai điểm mút của cung và P2 là điểm chính giữa của cung. Khi đó
2r SIN π.2r SIN π.2r SIN 2π 2r π.2r π.2r 2π
abc
S
P1P2 P3 = = n n n n n n
.
4r 4r
4r
Như vậy S 4r
2
.π
3
. Giả sử tọa độ nguyên các đỉnh P (x , y ), i = 1, 2, 3. Khi
P1 P2P3
n
3 ii i
đó
4r
2
.π
3 1 1 1
=
1 1
.
S
P1 P2P3 x1 x2 x3
n3 2 2
Ta nhận được bất đẳng thức n(r)
√
r2 y1 y2 y3
2π . Vì π là số siêu việt và n là số nguyên
dương nên n(r) < 2π
√
r2.
3
3
√
π .r2 [Iran MO 1999].
Nhận xét 1.3.2. Kết quả yếu hơn n(r) < 6
3
Ví dụ 1.3.4. Cho tam giác ABC với trọng tâm G. Với điểm S bất kỳ, gọi I là trung
điểm SG và đường thẳng tùy ý d qua I cắt SA, SB, SC tại A1, B1,C1, tương
ứng, khác S. Chứng minh rằng
SA
+
SB
+
SC
= 6.
SA1 SB1 SC1
Bài giải. Gọi M là trung điểm BC và kéo dài AM về phía M lấy điểm N sao cho
M là trung điểm GN. Gọi giao điểm SM, SN với d là M1, N1. Khi đó
SB + SC = 2 SM , SN +SG = 2 SM , SN + SA = 2SG = 4.
SB1SC1SM1 SN1 SI SM1 SN1 SA1 SI
Vậy
SA + SB + SC = 6.
SA1 SB1SC1
Mệnh đề 1.3.6. Với ΔABC và M ∈ BC ta có
SIN
∠ BAM
+
SIN
∠ CAM
=
SIN
∠ BAC
.
AC AB AM
Chứng minh. Do SABM + SACM = SABC nên
SIN
∠ BAM +
SIN
∠ CAM =
SIN
∠ BAC .
AC AB AM
Tải tài liệu tại sividoc.com
Viết đề tài giá sinh viên – ZALO:0973.287.149-TEAMLUANVAN.COM
20
Ví dụ 1.3.5. Với tứ giác lồi ABCD, K = AD × BC, L = AB × CD, F = BD × K L,
G = AC × K L, ta luôn luôn có hệ thức
2
=
1
+
1
.
K L K F K G
Bài giải. Đặt α = ∠CK A và β = ∠CK L Theo Mệnh đề 1.3.6 ta luôn có hệ
thức sau:
SIN(α + β ) = SIN α +SIN β ,SIN(α + β ) =SIN α + SIN β
K B K L K A KC KLKD
SIN(α + β ) = SIN α +SIN β ,SIN(α + β ) =SIN α + SIN β
K B K F K D KC KGKA
và có được kết quả
SINα +SINβ +SINα +SINβ =SINα +SINβ +SINα +SINβ
KL KA KL KD KF KD KG KA
hay hệ thức 2 = 1 + 1 .
K L KFKG
Ví dụ 1.3.6. Với tam giác ABC, độ dài cạnh a, b, c và độ dài đường phân
giác trong ℓa, ℓb, ℓc ta luôn có hệ thức
COS
∠A
COS
∠B
COS
∠ C
1 1 1
2 + 2 + 2 = + + .
ℓa ℓb ℓc abc
Tải tài liệu tại sividoc.com
Viết đề tài giá sinh viên – ZALO:0973.287.149-TEAMLUANVAN.COM
21
Bài giải. Theo Mệnh đề 1.3.6 ta có
2 COS
∠A
2 COS
∠B
2 COS
∠C
1 1 1 1 1 1
2 = + , 2 = + , 2 = + .
ℓ
b
ℓa bc ca ℓc ab
Từ đó ta có hệ thức
COS
∠A
COS
∠
B
COS
∠ C
1 1 1
2 + 2 + 2 = + + .
ℓa ℓb ℓc abc
Ta có điều phải chứng minh.
Ví dụ 1.3.7. Cho tứ giác lồi ABCD. Giả sử M, N ∈ AB và P, Q ∈ CD sao cho
AM = N B và CP = QD. Chứng minh rằng, nếu SAMQD = SBN PC thì AB CD.
Bài giải. Giả sử AB và CD cắt nhau tại O. Dựng hệ tọa độ Oxy sao cho
A(u, 0), M(u + a, 0), N(v, 0), B(v + a, 0),
D(z, kz), Q(z + b, k(z + b)), P(t , kt ), C(t + b, k(t + b))
với 0 < u < v, 0 < z < t , k = 0, a, b > 0, vì AM = N B và DQ = PC. Giả thiết
S
AMQD
= S
BN PC
.
Tính
2S
AMQD = u + a − u 0 − 0 + z+ b − u − a k(z + b) − 0
z − u kv − 0 z − u − a kz − 0
= kaz + kbu + kab
2S
BNPC = t + a − v 0 − 0 + t+ b − v − a k(t + b) − 0
t − v kt − 0 t − v − a kt − 0
= kat + kbv + kab
và nhận được kaz + kbu + kab = kat + kbv + kab hay a(t − z) = −b(v − u),
vô lý. Do vậy, điều giả sử là sai và có AB CD.
Ví dụ 1.3.8. Cho tứ giác lồi ABCD. Giả sử M, N ∈ AB và P, Q ∈ CD sao cho AM =
MN = N B và CP = PQ = QD. Gọi O = AC ×BD. Chứng minh rằng SOMP = SON Q.
Bài giải. Dựng hệ tọa độ Oxy sao cho A(a, 0), C(c, 0) và phương
trình AB : y = k(x − a), CD : y = h(x − c).
Tải tài liệu tại sividoc.com
Viết đề tài giá sinh viên – ZALO:0973.287.149-TEAMLUANVAN.COM
22
Giả sử B(a + 3u, k3u), D(c + 3v, h3v). Vì B, O, D thẳng hàng nên
ku(c + 3v) = hv(a + 3u)
hay
kcu − hav = 3(h − k)uv.
Tính tọa độ M(a + u, ku), P(c + v, hv), N(a + 2u, k2u), Q(c + 2v, h2v) và
tính diện tích tam giác định hướng
2S
OMP = c + v − 0
hv − 0
=kcu − hav + (k − h)uv
a + u − 0 ku − 0
2SON Q = c + 2v − 0 h2v − 0 2kcu − 2hav +4(k − h uv
= ) .
a + 2u − 0 k2u − 0
Vì kcu − hav = 3(h − k)uv nên SOMP = huv − kuv = SON Q.
1.3.3 Đường thẳng Simson, đường thẳng Steiner
Mệnh đề 1.3.7 (Simson). Giả sử điểm M thuộc đường tròn ngoại tiếp tam
giác ABC, khác phía với A đối với BC. Hạ MA1, MB1, MC1 vuông góc với
BC,CA, AB, tương ứng. Khi đó, ba điểm B1, A1,C1 thẳng hàng.
Chứng minh. Từ B1C1 = MA SIN A, A1B1 = MC SIN C, A1C1 = MB SIN B và
aMA = bMB + cMC theo Đồng nhất thức Ptolemy suy ra
A1B1 + A1C1 = MCSINC +MBSINB = cMC + bMB =aMA .
2R 2R
Từ đây suy ra A1B1 + A1C1 = B1C1 hay ba điểm B1, A1,C1 thẳng hàng.
Tải tài liệu tại sividoc.com
Viết đề tài giá sinh viên – ZALO:0973.287.149-TEAMLUANVAN.COM
23
Mệnh đề 1.3.8 (Feuerbach). Với tứ giác lồi ABCD nội tiếp trong một
đường tròn luôn có DA
2
SDBC + DC
2
SDAB = DB
2
SDCA.
Chứng minh. Vì tứ giác ABCD nội tiếp trong đường tròn nên
DA+DC =DB
bc ab ca
theo Bổ đề 1.3.1. Vậy
aDA2
+ cDC2
= bDB2
.
DA DC DB
Từ đây suy ra
DA2
.DB.DC.a + DC2
.DA.DB.c = DB2
.DC.DA.b
và dễ dàng có đồng nhất thức DA
2
SDBC + DC
2
SDAB = DB
2
SDCA.
Mệnh đề 1.3.9. Giả sử điểm M thuộc đường tròn ngoại tiếp tam giác ABC, khác
phía với A đối với BC. Gọi A
′
, B
′
,C
′
là điểm đối xứng với điểm M qua BC,CA, AB,
tương ứng, và A0 = MA
′
× BC, B0 = MB
′
×CA và C0 = MC
′
× AB. Khi đó ta có
(1) (Steiner)Ba điểm B
′
, A
′
,C
′
thẳng hàng.
(2) Đường thẳng (B′
, A′
,C′
) đi qua trực tâm H của ΔABC, còn đường
thẳng B0C0 đi qua trung điểm đoạn MH .
Chứng minh. (1) Ta có MA0 ⊥ BC, MB0 ⊥ CA và MC0 ⊥ AB. Theo Mệnh
đề 1.3.7, A0, B0, C0 thẳng hàng. Vì A
′
C
′
A0C0, A
′
B
′
A0B0 và C0, A0, B0
thẳng hàng nên ba điểm C
′
, A
′
, B
′
cũng thẳng hàng.
Tải tài liệu tại sividoc.com
Viết đề tài giá sinh viên – ZALO:0973.287.149-TEAMLUANVAN.COM
24
(2) Kẻ các đường cao AA1 ⊥ BC, BB1 ⊥ CA và CC1 ⊥ AB. Gọi H là trực tâm tam
giác ABC và đường thẳng AH , BH ,CH cắt đường tròn (O, R) tại A2, B2,C2. Khi
đó, theo các điểm A2, B2,C2 đối xứng với H qua BC,CA, AB, tương ứng. Vì các
hình thang C
′
C2H M, B
′
B2H M cân nên ∠C
′
HC2 = ∠MC2C = ∠MAC và ∠B
′
H B2 =
∠MB2B = ∠MAB.
Vậy
∠B
′
HC
′
= ∠B”HB2 + ∠B2HC2 + ∠C2HC
′
= ∠C1AB1 + ∠B1HC1 = 180
0
.
Do đó, ba điểm B
′
, H ,C
′
thẳng hàng. Vì B0C0 là đường trung bình của
tam giác MB
′
C
′
nên B0C0 đi qua trung điểm đoạn MH .
1.4 Định lý Ceva và Định lý Menelaus
Mệnh đề 1.4.1 (Ceva 1678). Cho tam giác ABC và ba điểm M ∈ (BC), N ∈
(CA) và P ∈ (AB) sao cho AM, BN,CP không từng cặp song song. Khi đó AM,
BN,CP
đồng quy tại một điểm khi và chỉ khi
MB NC PA
= −1.
MC NA PB
Chứng minh.
Giả thiết AM, BN,CP đồng quy tại I. Khi đó ta có biến đổi diện tích
BM CN AP =
S
IBM
S
ICN
S
IAP
=
S
ABM
S
BCN
S
CAP
=
S
BAI
.
S
CBI
.
S
ACI
= 1.
MC NA PBS
IMC
S
INA
S
IPB
S
ACM
S
BAN
S
CBP
S
CAI
S
ABI
S
CBI
Vậy
MB NC PA = −1.
MC NA PB
Ngược lại, giả thiết
MB NC PA = −1.
MC NA PB
Tải tài liệu tại sividoc.com
Viết đề tài giá sinh viên – ZALO:0973.287.149-TEAMLUANVAN.COM
25
Ta chỉ ra AM, BN, CP đồng quy tại một điểm. Thật vậy, gọi I = AM × BN
và CI cắt (AB) tại Q. Khi đó ta có
BM CN AP
=−1=
BM CN AQ
.
MC NA PB MC NA QB
Vậy
AP=AQ
PB QB
và dễ dàng suy ra được Q ≡ P.
Nhận xét 1.4.1. Trong một số tài liệu người ta thường kẻ qua A một
đường song song với BC, cắt BN và CP kéo dài để chứng minh Định lý
Ceva, và phép chứng minh là rất dài.
Ví dụ 1.4.1. Cho tam giác ABC. Giả sử đường tròn nội tiếp tiếp xúc với cạnh
BC,CA, AB tại A1, B1,C1; Giả sử đường tròn bàng tiếp nằm trong ∠A tiếp xúc
với cạnh BC tại A2, tiếp xúc với cạnh AB, AC kéo dài tại Ca, Ba. Khi đó AA1,
BB1,CC1 đồng quy tại một điểm J và điểm này được gọi là điểm Gergonne và
AA2, BBa,CCa đồng quy tại một điểm Na và điểm này được gọi là điểm Nagel.
Bài giải. Vì
BA1 CB1 AC1
= −1 và
BA2 CBa ACa
= −1
A1C B1A C1B A2C BaA CaB
nên AA1, BB1, CC1 đồng quy tại một điểm J và AA2, BBa,CCa đồng quy
tại một điểm Na theo Mệnh đề 1.4.1.
Ví dụ 1.4.2. Cho tam giác ABC. Giả sử ba đường tròn bàng tiếp tiếp xúc với cạnh
BC,CA, AB tại A2, B2,C2; tương ứng. Khi đó AA2, BB2,CC2 đồng quy tại một điểm
N.
Bài giải. Ký hiệu a = BC, b = CA, c = AB. Vì AB + BA2 = AC +CA2 nên
A
2
B
=
a
+
b
−
c
.
A2C a + c − b
Tương tự, tính
B
2
C
=
b
+
c
−
a
B2A b + a − c
Tải tài liệu tại sividoc.com
Viết đề tài giá sinh viên – ZALO:0973.287.149-TEAMLUANVAN.COM
26
và
C
2
A
=
c + a − b
.
C2B c + b − a
Vì
BA
2
CB
2
AC
2 = −1
A2C B2A C2B
nên AA2, BB2, CC2 đồng quy tại một điểm N.
Tương tự, khi xét đường tròn bàng tiếp ΔABC, nhưng nằm trong góc ∠B, ∠C,
tương ứng ta cũng nhận được hai điểm Nagel Nb , Nc nữa. Như vậy, với mỗi
ΔABC ta nhận được bốn điểm Nagel N, Na , Nb, Nc và một điểm Gergonne.
Ví dụ 1.4.3. Cho tam giác ABC. Giả sử đường tròn cắt ba cạnh BC,CA,
AB của tam giác tại A1, A2; B1, B2; C1, C2, tương ứng và theo thứ tự
A,C1,C2, B, A1, A2, C, B1, B2, A. Giả sử X = C1B1 × C2B2, Y = A1C1 ×
A2C2, Z = B1A1 × B2A2. Khi đó AX , BY,CZ đồng quy tại một điểm.
Bài giải. Gọi D là giao điểm giữa AX và C2B1. Vì AD,C2B2, B1C1 đồng
quy tại X nên
SIN ∠X AB
2 .
SIN ∠XC
2
C
1 .
SIN ∠X B
1
C
2 = 1.
SIN ∠X AC1 SIN ∠XC2B1 SIN ∠X B1B2
Do ∠XC2C1 = ∠X B1B2 nên ta nhận được
SIN ∠ X
AB2 =
SIN
∠ B1C2B2 ,(1).
SIN ∠ X
AC1 SIN ∠B2B1C1
Tương tự,
SIN ∠ Y
BC2 =
SIN
∠ C1A2C2 ,(2);
SIN
∠ XCA2 =SIN ∠ A1B2A2 ,(3).
SIN ∠ Y
BA1
SIN
∠ A2C1A1 SIN ∠ ZCB1SIN ∠ B2A1B1
Từ (1), (2), (3) ta suy ra
SIN ∠X AB
2 .SIN ∠Y BC
2 .SIN ∠XCA
2 = 1.
SIN ∠X AC1 SIN ∠Y BA1 SIN ∠ZCB1
Hệ thức này chứng tỏ AX , BY,CZ đồng quy tại một điểm.
Tải tài liệu tại sividoc.com
Viết đề tài giá sinh viên – ZALO:0973.287.149-TEAMLUANVAN.COM
Ví dụ 1.4.4. Cho tam giác ABC. Chia cạnh BC thành ba đoạn bằng nhau
BA1 = A1A2 = A2C;
Tải tài liệu tại sividoc.com
Viết đề tài giá sinh viên – ZALO:0973.287.149-TEAMLUANVAN.COM
27
Chia cạnh CA thành ba đoạn bằng nhau CB1 = B1B2 = B2A; Chia cạnh AB
thành ba đoạn bằng nhau AC1 = C1C2 = C2B. Gọi D1 = BB1 × CC2, D2 = BB1
× AA2, D3 = CC1 × AA2, D4 = CC1 × BB2, D5 = AA1 × BB2, D6 = AA1 × CC2.
Chứng minh ba đường thẳng D1D4, D2D5, D3D6 đồng quy tại một điểm.
Bài giải. Theo Định lý Ceva, đường trung tuyến AA
′
của tam giác ABC
chứa điểm D1, D4; đường trung tuyến BB
′
của tam giác ABC chứa điểm
D3, D6; đường trung tuyến CC
′
của tam giác ABC chứa điểm D2, D5. Từ
đó suy ra ba đường thẳng D1D4, D2D5, D3D6 đồng quy tại một điểm.
Ví dụ 1.4.5. Cho tam giác ABC. Lấy điểm M, N, P thuộc cạnh BC, CA và AB,
1
tương ứng. Nếu AM, BN,CP đồng quy thì ta luôn có SMN P 4 SABC.
Bài giải. Đặt
u =MB , v = NC , t = PA .
MC NA PB
Giả thiết AM, BN,CP đồng qui tại một điểm. Khi đó u, v,t > 0 và uvt = 1 và
S
MNP
= 1 + uvt .
(1 + u)(1 + v)(1 + t )
S
ABC
Vì (1 + u)(1 + v)(1 + t ) 8 nên
S
MNP
= 1 + 1 2 =1 .
S
ABC (1+ u)(1 + v)(1 + t ) 8 4
Ta có điều phải chứng minh.
Mệnh đề 1.4.2 (Menelaus). Cho tam giác ABC và điểm N thuộc cạnh CA, điểm
P thuộc cạnh AB, còn điểm M thuộc cạnh BC kéo dài. Khi đó, ba điểm M, N, P
thẳng hàng khi và chỉ khi
MB
.
NC
.
PA
= 1.
MC NA PB
Tải tài liệu tại sividoc.com
Viết đề tài giá sinh viên – ZALO:0973.287.149-TEAMLUANVAN.COM
28
Chứng minh. Theo khoảng cách: Giả thiết ba điểm M, N, P thẳng hàng. Ký hiệu
khoảng cách từ A, B,C đến đường thẳng (MN P) là u, v,t . Khi đó ta có hệ thức
MB .CN .AP =v .t .u = 1 hay là MB .NC .PA = 1.
MC NA PBt u v MC NA PB
Theo diện tích: Ta có
MB NC PA
=
S
MBN
S
MCN
S
MAN
= 1.
MC NA PB
S
MCN
S
MAN
S
MBN
Giả thiết
MB.NC.PA = 1.
MC NA PB
Ta chỉ ra ba điểm M, N, P thẳng hàng. Gọi M′
là điểm giao N P × BC. Ta có
M′
B.NC.PA = 1 = MB.NC.PA.
M
′
C NA PB MC NA PB
Vậy
M
′
B = MB
M′
C MC
và cũng dễ dàng suy ra được M ≡ M′
.
Mệnh đề 1.4.3. Cho tứ giác ABCD. Giả sử đường thẳng d cắt các đường
thẳng AB, BC, CD, DA tại các điểm M, N, P, Q, tương ứng. Khi đó, ta có
T =
MA
.
NB
.
PC
.
QD
= 1.
MB NC PD QA
Chứng minh. Ký hiệu khoảng cách từ các đỉnh A, B,C, D đến d qua x, y, z,t . Dễ
x y z t
dàng kiểm tra T = . . . = 1.
Tải tài liệu tại sividoc.com
Viết đề tài giá sinh viên – ZALO:0973.287.149-TEAMLUANVAN.COM
29
Định lí 1.4.1 (Desargues). Cho hai tam giác ABC và A′
B′
C′
sao cho ba
đường thẳng AA
′
, BB
′
,CC
′
đồng quy tại điểm E . Ký hiệu X ,Y, Z lần lượt
là giao điểm giữa AB và A′
B′
; giữa BC và B′
C′
; giữa CA và C′
A′
. Chứng
minh rằng, ba điểm X ,Y, Z thẳng hàng. Ngược lại, nếu BB
′
,CC
′
cắt nhau
tại O và X ,Y, Z thẳng hàng thì AA′
, BB′
,CC′
đồng quy tại O.
Chứng minh. Áp dụng Định lý Menelaus cho ΔE AB với cát tuyến (A′
B′
X );
cho ΔE BC với cát tuyến (B′
C′
Y ) và ΔECA với cát tuyến (C′
A′
Z), ta có
A
′
E .XA. B
′
B = 1, B
′
E .YB.C
′
C = 1, C
′
E .ZC. A
′
A = 1.
A
′
A X B B
′
E B
′
B YC C
′
E C
′
C ZA A
′
E
Nhân ba hệ thức này, vế với vế, ta nhận đượcX A .Y B .ZC = 1 và theo Mệnh đề
X B Y C Z
1.4.2 ba điểm X ,Y, Z thẳng hàng. Phần còn lại được coi như một bài tập nhỏ.
1.5 Bất đẳng thức Erdos¨-Mordell cho đa giác
Trước khi phát biểu và chứng minh Bất đẳng thức Erdos¨-Mordell cho
đa giác lồi ta chứng minh Bất đẳng thức Erdos¨-Mordell cho tam giác.
Ví dụ 1.5.1. Cho tam giác ABC. Giả sử M là một điểm trong tam giác và ký hiệu
khoảng cách từ M đến cạnh BC,CA, AB là x, y, z. Đặt d1 = MA, d2 = MB, d3 = MC.
Khi đó ta có Bất đẳng thức Erdos¨-Mordell COS
π(d1 + d2 + d3) x + y + z.
3
Bài giải. Cách 1. Ta có
d1d2 COS α1 +d2d3 COS α2 + d3d1 COS α3x + y + z
√
với α1 + α2 + α3 = π . Đặt ak = dk , k = 1, 2, 3. Ta chỉ ra
a
2
1 + a
2
2 + a
2
3 2a1a2 COS α1 + 2a2a3 COS α2 + 2a3a1 COS α3
hay
(a1 − a2 COS α1 − a3 COS α3)
2
+ (a2 SIN α1 − a3 SIN α3)
2
0
đúng. Do vậy, ta nhận được bất đẳng thức COS
π(d1 + d2 + d3) x + y + z.
3
Cách 2. Ta có
d1
2
SIN
2
A = y
2
+ z
2
− 2yz COS(B +C)
Tải tài liệu tại sividoc.com
Viết đề tài giá sinh viên – ZALO:0973.287.149-TEAMLUANVAN.COM
30
= (y SIN C + z SIN B)
2
+ (y COS C − z COS B)
2
(y SIN C + z SIN B)
2
Vậy
d1 ySIN C + zSIN B .
SIN A SIN A
Tương tự, ta có
d2 zSIN A + x SIN C và d3x SIN B + y SIN A .
SIN BSIN B SIN CSIN C
Cộng ba bất đẳng thức trên có COS
π (d1 + d2 + d3) x + y + z.
3
Mệnh đề 1.5.1. Cho đa giác lồi A1A2 . . . An với n 3. Giả sử M là một điểm trong
đa giác và ký hiệu khoảng cách từ M đến cạnh AiAi+1 là ri với i = 1, 2, . .
. , n và quy ước n + 1 := 1. Đặt 2αi = ∠AiMAi+1 và dk = MAk với k = 1, . .
. , n. Ta có đồng nhất thức cho Bất đẳng thức Erdos¨-Mordell:
n n
∑ dk dk+1 SIN 2αk = ∑ rk (dk − dk+1)2
+ 4dk dk+1 SIN
2
αk .
k=1 k=1
Từ đó suy ra
n
n
∑ dk dk+1 COS αk∑ rk .
k=1 k=1
Chứng minh. Xét ΔA1MA2. Từ r1.A1A2 = d1d2 SIN 2α1 ta suy ra được:
d1d2 SIN 2α1 = r1 (d1 − d2)
2
+ 4d1d2 SIN
2
α1.
Tương tự, ta có:
dk dk+1 SIN 2αk = rk (dk − dk+1)
2
+ 4dk dk+1 SIN
2
αk
với k = 1, 2, . . . , n. Lấy tổng tất cả các hệ thức được:
n n
∑ dk dk+1 SIN 2αk = ∑ rk (dk − dk+1)2
+ 4dk dk+1 SIN
2
αk .
k=1 k=1
Vì
(dk − dk+1)
2
+ 4dk dk+1 SIN
2
αk 2
d
k
d
k+1 SIN αk
và viết
SIN 2αk = 2 SIN αk COS αk
n n
nên ta có bất đẳng thức ∑ dk dk+1 COS αk∑ rk .
k=1 k=1
Tải tài liệu tại sividoc.com
Viết đề tài giá sinh viên – ZALO:0973.287.149-TEAMLUANVAN.COM
31
Hai kết quả dưới đây là của Wolstenholme và Lenhard:
Mệnh đề 1.5.2. Cho đa giác lồi A1A2 . . . An với n 3. Giả sử M là một điểm
trong đa giác. Ký hiệu khoảng cách từ M đến cạnh AiAi+1 là ri, di = MAi, với
i = 1, 2, . . . , n và quy ước n + 1 := 1. Khi đó ta có các bất đẳng thức
(1)
π n n
dk dk+1 COS αk .
COS ∑ dk ∑
n
k=1 k=1
(2)
n 1 n
∑ dk ∑ rk .
π
k=1 COS k=1
n
π n n
Chứng minh. (1) Bất đẳng thức dk dk+1 COS αk . Theo mệnh đề
COS ∑ dk ∑
n n
n
k=1 k=1
trên ta đã có COS αk được suy ra từ một kết quả của Wolsten-
∑ rk ∑ d
k
d
k+1
k=1 k=1
holme và Lenhard.
(2) Vì
n n n 1 n
∑ dk dk+1 COS αk ∑ rk nên ∑ dk ∑ rk .
π
k=1 k=1 k=1 COS k=1
n
Mệnh đề 1.5.3 (Hayashi). Giả sử tam giác ABC có BC = a, CA = b, AB = c.
Khi đó, với bất kỳ điểm M ở trong mặt phẳng ABC, ta có bất đẳng thức:
aMB.MC + bMC.MA + cMA.MB abc.
Đặc biệt, khi M ≡ G là trọng tâm của tam giác ABC ta có bất đẳng thức:
9
am
b
m
c
+ bm
c
m
a
+ cm
a
m
b 4
abc
và khi M ≡ I là tâm đường tròn nội tiếp của tam giác ABC ta còn có bất đẳng thức:
a b c abc
+ + .
IA IB IC 4Rr
2
Chứng minh. Tương ứng đỉnh A, B,C với số phức z1, z2, z3 và điểm M
ứng với số phức z. Từ bất đẳng thức
|z − z2||z − z3| + |z − z3||z − z1| + |z − z1||z − z2| 1,
|z1 − z2||z1 − z3| |z2 − z3||z2 − z1| |z3 − z1||z3 − z2|
ta suy ra
MC.MA MA.MB
MB.MC + + 1.
bc ca ab
Tải tài liệu tại sividoc.com
Viết đề tài giá sinh viên – ZALO:0973.287.149-TEAMLUANVAN.COM
32
Vậy
a.MB.MC + b.MC.MA + c.MA.MB abc.
Khi M ≡ I là tâm đường tròn nội tiếp của tam giác ABC ta còn có bất đẳng thức.
a
+
b
+
c abc
,vì IA.IB.IB = 4Rr
2
.
IA IB IC 4Rr
2
Mệnh đề 1.5.4. Giả sử tam giác ABC có BC = a,CA = b, AB = c. Gọi A1, B1,C1 là
trung điểm của các cạnh BC,CA, AB, tương ứng. Ký hiệu khoảng cách từ điểm
M trong mặt phẳng ABC đến A1, B1,C1 là x, y, z. Khi đó có bất đẳng thức:
axMA + byMB + czMCabc
2 .
Chứng minh. Tương ứng đỉnh A, B,C với số phức z1, z2, z3 và điểm M
ứng với số phức 0. Từ bất đẳng thức
|z1||z
2
2 − z
2
3| + |z2||z
2
3 − z
2
1| + |z3||z
2
1 − z
2
2| |z1 − z2||z2 − z3||z3 − z1|
abc
2 .
Mệnh đề 1.5.5. Giả sử tam giác ABC có BC = a,CA = b, AB = c. Khi đó,
với bất kỳ điểm M ở trong mặt phẳng ABC, ta có các bất đẳng thức:
aMA
2
+ bMB
2
+ cMC
2
abc.
Hơn nữa ta còn có aH A
2
+ bH B
2
+ cHC
2
abc, trong đó H là trực tâm
của ΔABC.
Chứng minh. Tương ứng đỉnh A, B,C với số phức z1, z2, z3 và điểm M
ứng với số phức 0. Từ bất đẳng thức
|z1|
2
|z2 − z3| + |z2|
2
|z3 − z1| + |z3|
2
|z1 − z2| |z1 − z2||z2 − z3||z3 − z1|
ta có aMA
2
+ bMB
2
+ cMC
2
abc.
Mệnh đề 1.5.6. Giả sử tam giác ABC có BC = a,CA = b, AB = c. Gọi G là
trọng tâm của tam giác. Với bất kỳ điểm M ở trong mặt phẳng ABC, ta
luôn có bất đẳng thức:
(1) a
3
MA + b
3
MB + c
3
MC 3abcMG.
theo ta có axMA + byMB + czMC
Tải tài liệu tại sividoc.com
Viết đề tài giá sinh viên – ZALO:0973.287.149-TEAMLUANVAN.COM
33
(2) aMA
3
+ bMB
3
+ cMC
3
3abcMG.
Chứng minh. Tương ứng đỉnh A, B,C với số phức z1, z2, z3 và điểm M
ứng với số phức 0. Từ bất đẳng thức
|z1||z2 − z3 |
3
+ |z2||z3 − z1|
3
+ |z3||z1 − z2|
3
|z1 + z2 + z3||z1 − z2 ||z2 −
z3||z3 − z1|,
ta có a
3
MA + b
3
MB + c
3
MC3abcMG. Bất đẳng thức
aMA
3
+ bMB
3
+ cMC
3
3abcMG
được chứng minh tương tự.
Ví dụ 1.5.2. Giả sử tam giác ABC có BC = a,CA = b, AB = c. Khi đó, với
bất kỳ điểm M trong mặt phẳng ABC, ta luôn có bất đẳng thức:
abc 2
aMB.MC + bMC.MA + cMA.MB R2 MO .
Đặc biệt, khi M ≡ K là tâm đường tròn bàng tiếp của tam giác ABC và ở
trong ∠A ta còn có bất đẳng thức:
a + b + c abc(R + 2ra) .
KAKBKC 4R2r2
a
Bài giải. Tương ứng các điểm A, B,C, E và tâm đường tròn ngoại tiếp O
của ΔABC với số phức z1, z2, z3, z, 0. Từ bất đẳng thức:
|z1|
2
|z − z2||z − z3| +|z2|
2
|z − z3||z − z1| +|z3|
2
|z − z1||z − z2| z
2
|z1 − z2 ||z1− z3| |z2 − z3 ||z2 − z1 | |z3 − z1 ||z3− z2 | | |
với n = 2, ta suy ra
MB.MC MC.MAMA.MB MO
2
+ + .
bc ca ab R
2
Như vậy chúng ta có
aMB.MC + bMC.MA + cMA.MB abc MO
2
.
R2
Khi M ≡ K là tâm đường tròn bàng tiếp của tam giác ABC và ở trong ∠A ta có
KO
2
= R
2
+ 2Rra, K A.K B.KC = 4Rra
2
và như vậy có bất đẳng thức
a + b + c abc(R2
+ 2Rra) =abc(R + 2ra) .
R2 .4Rra
2
4R2
ra
2
KAKBKC
Tải tài liệu tại sividoc.com
Viết đề tài giá sinh viên – ZALO:0973.287.149-TEAMLUANVAN.COM
34
Mệnh đề 1.5.7. Giả sử tam giác ABC có BC = a,CA = b, AB = c nội tiếp
trong đường tròn tâm O bán kính R. Khi đó, với bất kỳ hai điểm E và F ở
trong mặt phẳng ABC ta luôn có bất đẳng thức:
OE.OF AE.AF +BE.BF + CE.CF .
R2 bc ca ab
Đặc biệt, khi E ≡ F ta có
AE 2
+
BE 2
+
CE 2
OE 2
.
bc ca ab R2
Chứng minh. Tương ứng đỉnh A, B,C với số phức z1, z2, z3, tâm O với số
phức z và hai điểm E , F ứng với hai số phức u, v. Từ bất đẳng thức dưới đây:
|z − u||z − v| |z1 − u||z1 − v|
|z − z1||z − z2||z − z3| |z1 − z2||z1 − z3||z − z1|
+ |z2 − u||z2 − v| + |z3 − u||z3 − v|
|z2 − z1||z3 − z2||z − z3|
− z1||z2 − z3||z − z2| |z3
ta suy ra OE.OF AE.AF +BE.BF + CE.CF .
R
2
ca
bc ab
Mệnh đề 1.5.8. Tam giác ABC với BC = a,CA = b, AB = c có đường tròn
ngoại tiếp tâm O bán kính R và đường tròn nội tiếp tâm I bán kính r. Gọi
H là trực tâm của tam giác. Ký hiệu khoảng cách từ O đến các cạnh
BC,CA, AB là x, y, z. Ta có bất đẳng thức:
(1) ax + by + cz abcMH với bất kỳ điểm M.
MA MB MC2MA.MB.MC
(2)ax +by +cz abc IH .
IC 8Rr
2
IA IB
Chứng minh. (1) Tương ứng đỉnh A, B,C, O với số phức z1, z2, z3, 0 và
M với số phức z. Từ bất đẳng thức
|z2 + z3| + |z3 + z1| + |z1 + z2|
|z1 − z2||z1 − z3||z1 − t | |z2 − z3 ||z2 − z1||z2 − t | |z3 − z1||z3 − z2||z3 − t |
|t − z1 − z2 − z3|
|t − z1||t − z2||t − z3|
ta có ngay bất đẳng nhất thức
ax + by + cz abcMH .
MC2MA.MB.MC
(2) Khi M ≡ I ta có
MA MB
ax
+
by
+
cz abc
IH =
abc
IH .
IA IB IC 2IA.IB.IC 8Rr2
Ta có điều phải chứng minh.
Tải tài liệu tại sividoc.com
Viết đề tài giá sinh viên – ZALO:0973.287.149-TEAMLUANVAN.COM
35
Chương 2
Phương pháp thể tích
2.1 Phương pháp thể tích
2.1.1 Phương pháp thể tích
Để nghiên cứu có hiệu quả, chúng ta cần phương pháp đơn giản sau
đây: Phương pháp thể tích là phương pháp giải bài toán hình trong không
gian qua thể tích của một khối nào đó. Để giải bài toán đã cho ta chọn một
khối đa diện (K), chẳng hạn, với thể tích V. Sau đó đem chia khối (K) ra
thành nhiều khối nhỏ một cách thích hợp và tính thể tích V1, . . . ,Vr của các
khối nhỏ ấy. Từ V = V1 + · · · +Vr suy ra lời giải bài toán.
Mệnh đề 2.1.1. Cho tứ diện ABCD với diện tích 4 mặt sa, sb, sc, sd ; độ
dài các đường cao ha, hb, hc, hd ; độ dài các bán kính mặt cầu ngoại,
nội, bàng tiếp R, r, Ra, Rb, Rc, Rd . Khi đó, thể tích V của tứ diện ABCD
được xác định qua các công thức
(1) V =
sa
3
ha
=
sb
3
hb
=
sc
3
hc
=
sd
3
hd
.
(2) V =
(sa + sb + sc + sd )r
=
sr
, ở đó s = sa + sb + sc + sd .
33
(3) V =
(s − s
a
)R
a =
(s − 2s
b
)R
b =
(s − s
c
)R
c =
(s − s
d
)R
d .
3333
x
1
y
1
z
1 1
(4)V= 1
x
2
y
2
z
2 1
, trong đó A(x1, y1, z1), B(x2, y2, z2), C(x3, y3, z3),
6 x3 y3 z3 1
x4 y4 z4 1
D(x4, y4, z4) trong hệ tọa độ Oxyz.
Tải tài liệu tại sividoc.com
Viết đề tài giá sinh viên – ZALO:0973.287.149-TEAMLUANVAN.COM
36
2.1.2 Thể tích qua định thức
Mệnh đề 2.1.2. Cho các vector
−→
,
−→
,
−→
trong không gian với hệ tọa độ trực
x y z
chuẩn. Ta có đồng nhất thức
−→ −→ −→ −→ −→ −→
−→−→ −→
2 x . xx . y x . z
=
−→ −→ −→ −→ −→ −→
[ x , y ]. z y . x y . y y . z.
−→ −→ −→ −→ −→ −→
z . x z . y z . z
Chứng minh. Đặt
−→
= (a, b, c),
−→
= (a
′
, b
′
, c
′
),
−→
= (a”, b”, c”). Biến đổi đơn
x y z
giản
2
−→ −→ −→
a b c a b c
2
b
′
c
′
= a
′
b
′
c
′
[ x , y ]. z = a
′
a” b” c” a” b” c”
ta nhận được hệ thức liên hệ
a a
′
a”
b b
′
b”
c c′
c”
−→ −→ −→ −→ −→ −→
−→ −→ −→ 2
x . x x . y x . z
−→ −→ −→ −→ −→ −→
.
[ x , y ]. z = y . x y . y y . z
−→ −→ −→ −→ −→ −→
z . x z . y z . z
Ta có điều phải chứng minh.
Mệnh đề 2.1.3. Giả sử a1, a2, a3 và b1, b2, b3 là cosin của các góc do
hai đường thẳng (ℓ1) và (ℓ2) tạo với các trục tọa độ Ox, Oy, Oz, tương
ứng của hệ tọa độ trực chuẩn (Oxyz). Ký hiệu α là góc giữa tia (ℓ1) và
(ℓ2). Khi đó ta có công thức biểu diễn
SIN
2
α = (a1b2 − a2b1)
2
+ (a2b3 − a3b2)
2
+ (a3b1 − a1b3)
2
.
Từ đó suy ra rằng, hai đường thẳng (ℓ1) và (ℓ2) vuông góc với nhau khi và chỉ khi
(a1b2 − a2b1)
2
+ (a2b3 − a3b2)
2
+ (a3b1 − a1b3)
2
= 1.
Bài giải. Gọi
−→ −→ −→
Khi đó
i , j , k là vector đơn vị trên các trục Ox, Oy, Oz.
−→ −→ −→ −→ −→ −→ −→ −→
u = a1 i + a2 j + a3 k , v = b1 i + b2 j + b3 k
là những vector đơn vị chỉ phương của (ℓ1) và (ℓ2), và ta có
2 −→ −→ 2
= (a1b2 − a2b1)
2
+ (a2b3 − a3b2)
2
+ (a3b1
2
.
SIN α = [ u , v ] − a1b3)
Tải tài liệu tại sividoc.com
Viết đề tài giá sinh viên – ZALO:0973.287.149-TEAMLUANVAN.COM
37
Ví dụ 2.1.1. Tính thể tích hình chóp SABC với SA = a, SB = b, SC = c và
góc giữa SB, SC; giữa SC, SA; giữa SA, SB bằng ϕ1, ϕ2, ϕ3, tương ứng.
Bài giải. Đặt
−→ −→ −→ −→ −→ −→
Từ
x = SA, y = SB, z = SC và V = VSABC.
−→ −→ −→ −→ −→ −→
−→ −→ −→
x . x x . y x . z
36 2
2
=
−→ −→ −→ −→ −→ −→
V = |[SA, SB], SC| y . xy . yy . z
−→ −→ −→ −→ −→ −→
z . x z . y z . z
a2 ab COS ϕ3 ac COS ϕ2
= ab COS ϕ3 b2 bc COS ϕ1
ac COS ϕ2 bc COS ϕ1 c
2
theo Mệnh đề 2.1.2 và ta suy ra ngay công thức tính thể tích V của tứ
diện theo cạnh và góc
V =
abc
1 − COS
2
ϕ1 − COS
2
ϕ2 − COS
2
ϕ3 + 2 COS ϕ1 COS ϕ2 COS ϕ3.
6
Mệnh đề 2.1.4. Giả sử hình chóp SABC có độ dài cạnh SA = a, SB = b, SC = c,
BC = x, CA = y, AB = z. Ta có công thức tính thể tích tứ diện:
1 2a
2
a
2
+ b
2
− z
2
a
2
+ c
2
− y
2
V = 12
√
a
2
+ b
2
− z
2
2b
2 b
2
+ c
2
− x
2
.
2
a
2
+ c
2
− y
2
b
2
+ c
2
− x
2
2c
2
Chứng minh. Như trên, có 36V
2 −→ −→ −→ 2
hay biểu diễn V qua định thức
= |[SA, SB], SC|
−→ −→ −→ −→ −→ −→ 2
ab COS ϕ3 ac COS ϕ2
x . x x . y x . z a
36 2
=
−→ −→ −→ −→ −→ −→
= ab COS ϕ3
2
bc COS ϕ1
V y . x y . y y . z b
−→ −→ −→ −→ −→ −→
ac COS ϕ2 bc COS ϕ1
2
z . x z . y z . z c
theo Mệnh đề 2.1.2. Từ đây suy ra ngay hệ thức liên hệ qua định thức sau đây:
2a
2
2ab COS ϕ3 2ac COS ϕ2
288V
2
= 2ab COS ϕ3 2b
2
2bc COS ϕ1
2ac COS ϕ2 2bc COS ϕ1 2c
2
Tải tài liệu tại sividoc.com
Viết đề tài giá sinh viên – ZALO:0973.287.149-TEAMLUANVAN.COM
38
và có
1 2a2
a2
+ b2
− z2
a2
+ c2
− y2
V = 12
√
a2
+ b2
− z2
2b
2 b2
+ c2
− x2
.
2
a
2
+ c
2
− y
2
b
2
+ c
2
− x
2
2c
2
Điều phải chứng minh.
Mệnh đề 2.1.5. Giả sử tứ diện ABCD có tọa độ 4 đỉnh là A(x1, y1, z1),
B(x2, y2, z2), C(x3, y3, z3) và D(x4, y4, z4). Khi đó ta có công thức tính
thể tích tứ diện qua tọa độ bốn đỉnh:
x1 y1 z1 1
V = 1 x2 y2 z2 1 .
6 x3 y3 z3 1
x4 y4 z4 1
Chứng minh. Vì
x1 y1 z1 1
x2 − x1 y2 − y1 z2 − z1
x2 y2 z2 1
x
3
y
3
z
3 1 = x3 − x1 y3 − y1 z3 − z1
x4 y4 z4 1 x4 − x1 y4 − y1 z4 − z1
nên ta có hệ thức
x1 y1 z1 1
x2 y2 z2 1 −→ −→ −→
x
3
y
3
z
3 1
= [AB,AC].AD = 6V.
x4 y4 z4 1
Ví dụ 2.1.2. Cho tứ diện A1A2A3A4. Chứng minh rằng, với điểm O ta có
bất đẳng thức
1
4
V
A1A2A3A4 ∏ OAi
2
+ 1.
6 i=1
Tải tài liệu tại sividoc.com
Viết đề tài giá sinh viên – ZALO:0973.287.149-TEAMLUANVAN.COM
39
Bài giải. Dựng hệ tọa độ Oxyz. Với Ai(xi, yi, zi), i = 1, 2, 3, 4 có
2
x1 y1 z1 1
36V 2
=
x
2 y2 z2 1
A1A2A3A4 x3 y3 z3 1
x4 y4 z4 1
theo Bất đẳng thức Hadamard. Vậy VA1 A2 A3 A4
4
∏(x
2
i + y
2
i + z
2
i + 1)
i=1
1
4
∏ OA
2
i + 1.
6
i=1
Ví dụ 2.1.3. Cho điểm O ở trong tứ diện ABCD. Mặt phẳng (P) qua O cắt AB, AC, AD. Ký
hiệu khoảng cách từ A, B,C và D đến (P) là ta,tb ,tc,td , tương ứng. Chứng minh
t
a
V
OBCD =
t
b
V
OCDA +
t
c
V
ODAB +
t
d
V
OABC.
Bài giải. Dựng hệ tọa độ Oxyz để O(0, 0, 0), (P) : z = 0 và A(x1, y1, z1), B(x2, y2, z2),
C(x3, y3, z3), D(x4, y4, z4) với ta = z1 > 0, tb = −z2 > 0, tc = −z3 > 0, td = −z4 > 0.
Quy ước thể tích tứ diện được tính qua định thức
0 0 0 1
x2 y2 z2 1
x2 y2 z2
t
a
V
OBCD = z1 = −z1 x3 y3 z3
x3 y3 z3 1
x
4
y
4
z
4 1
x4 y4 z4
0 0 0 1
x4 y4 z4 1
x1 y1 z1
t
b
V
OCDA = −z2 = −z2 x3 y3 z3
x3 y3 z3 1
x1 y1 z1 1 x4 y4 z4
0 0 0 1
x4 y4 z4 1
x1 y1 z1
t
c
V
ODAB = −z3 = z3 x2 y2 z2
x1 y1 z1 1
x2 y2 z2 1 x4 y4 z4
0 0 0 1
x
1
y
1
z
1
1 x1 y1 z1
Tải tài liệu tại sividoc.com
Viết đề tài giá sinh viên – ZALO:0973.287.149-TEAMLUANVAN.COM
t
d
V
OABC = −z4 = −z4 x2 y2 z2 .
x3 y3 z3 1
x2 y2 z2 1 x3 y3 z3
Tải tài liệu tại sividoc.com
Viết đề tài giá sinh viên – ZALO:0973.287.149-TEAMLUANVAN.COM
40
Như vậy, T = taVOBCD − tbVOCDA − tcVODAB − tdVOABC đúng bằng
x2 y2 z2 x1 y1 z1 x1 y1 z1
T = −z1 x3 y3 z3 + z2 x3 y3 z3 − z3 x2 y2 z2
x4 y4 z4
x
4
y
4
z
4 x4 y4 z4
x1 y1 z1 x1y1 z1 z1
=
x
2y2 z2 z2
+z4
x
2
y
2
z
2 = 0.
x3 y3 z3 x3y3 z3 z3
x
4
y
4
z
4
z
4
Tóm lại
t
a
V
OBCD =
t
b
V
OCDA +
t
c
V
ODAB +
t
d
V
OABC.
Ví dụ 2.1.4. Cho tứ diện ABCD. Gọi A1, B1,C1, D1 là trọng tâm các mặt
BCD, CDA, DAB, ABC, tương ứng. Chứng minh rằng, với bất kỳ điểm O
ở bên ngoài tứ diện ABCD có một cách chọn thích hợp u, v,t ∈ {1, −1} để
1
|V
OB1C1 D1
+ uV
OC1 D1 A1
+ vV
OD1A1 B1
+ tV
OA1B1C1
| =
27
V
ABCD
.
Bài giải. Dựng hệ Oxyz để O(0, 0, 0), A(x1, y1, z1), B(x2, y2, z2), C(x3,
y3, z3) và D(x4, y4, z4). Tọa độ trọng tâm mặt tứ diện ABCD
A1 (
x2 + x3 + x4 ,
y2 + y3 + y4
,
z2 + z3 + z4 )
3 3 3
B
1 ( x1 + x3 + x4 ,y1 + y3 + y4
,
z1 + z3 + z4 )
3 3 3
C1 (x2 + x1 + x4 ,y2 + y1 + y4
,
z2 + z1 + z4 )
3 3 3
D1 (x2 + x3 + x1 ,y2 + y3 + y1 ,z2 + z3 + z1 ).
3 3 3
Đặt
4 4 4
X = ∑ xi, Y = ∑ yi, Z = ∑ zi,
V
a
= 6V
OB1C1 D1
.
i=1 i=1 i=1
Ta có
0 0 0 1
x1 + x3 + x4
y
1+ y3 + y4 z1 + z3 + z4 1
Va = ±
3 3 3
x2 + x1 + x4 y2+ y1 + y4 z2 + z1 + z4 1
+ x1
3 3
3
+ y1 + z1
x2 + x3 y2+ y3 z2 + z3 1
3 3 3
Tải tài liệu tại sividoc.com
Viết đề tài giá sinh viên – ZALO:0973.287.149-TEAMLUANVAN.COM
41
= ±1
27
Đặt
X − x2
X − x3
X − x4
Y − y2
Y − y3
Y − y4
Z − z2
X Y Z 1
±
1 x 2 y 2 z 2
1
Z z = .
− 3 27 x3 y3 z3 1
Z − z4 x4 y4 z4 1
X Y
T =
x2 y2
x3 y3
x4 y4
X Y
±
x
1
y
1
x2 y2
x4 y4
Z 1 X Y
z2 1
±
x1 y1
z3 1
x
3
y
3
z4 1 x4 y4
Z 1 X Y
z
1 1
±
x1 y1
z2 1 x2 y2
z4 1 x3 y3
Z 1
z1 1
z3 1
z4 1
Z 1
z1 1
z2 1
z3 1
với việc chọn dấu +, − một cách thích hợp. Từ
X Y Z 1 1
x1 y1 z1 1 1
x2 y2 z2 1 1 = 0
x3 y3 z3 1 1
x4 y4 z4 1 1
suy ra
x1 y1 z1 1 X YZ 1
x2 y2 z2 1
−
x2 y2 z2 1
x3 y3 z3 1 x3 y3 z3 1
x4 y4 z4 1 x4 y4 z4 1
X YZ 1 X YZ 1 X YZ 1
+
x
1
y
1
z
1 1
−
x1 y1 z1 1
+
x
1
y
1
z
1 1
= 0.
x3 y3 z3 1 x2 y2 z2 1 x2 y2 z2 1
x4 y4 z4 1
x
4
y
4
z
4 1 x3 y3 z3 1
Như vậy
±
V
ABCD ±
27V
OB1C1 D1 ±
27V
OC1D1 A1 ±
27V
OD1A1B1 ±
27V
OA1B1C1 =
0
.
Tải tài liệu tại sividoc.com
Viết đề tài giá sinh viên – ZALO:0973.287.149-TEAMLUANVAN.COM
42
Từ đó suy ra kết quả: Tồn tại một cách chọn thích hợp u, v,t ∈ {1, −1} để
1
|V
OB1C1 D1
+ uV
OC1 D1 A1
+ vV
OD1A1 B1
+ tV
OA1B1C1
| =
27
V
ABCD
.
Ta có điều phải chứng minh.
2.2 Quan hệ bán kính mặt cầu ngoại-nội tiếp
Mệnh đề 2.2.1. Tứ diện A1A2 A3A4 có tọa độ bốn đỉnh A1(x1, y1, z1),
A2(x2, y2, z2), A3(x3, y3, z3) và A4(x4, y4, z4). Ký hiệu R là bán kính mặt
cầu ngoại tiếp tứ diện. Khi đó ta có công thức tính bán kính mặt cầu
ngoại tiếp R qua độ dài 6 cạnh tứ diện:
2l13
2
l14
2
l23
2
l24
2
+ 2l12
2
l14
2
l32
2
l34
2
+ 2l12
2
l13
2
l42
2
l43
2
− l12
4
l34
4
− l13
4
l24
4
− l14
4
l23
4
R =
24V
với V là thể tích tứ diện và li j = l ji là độ dài cạnh AiA j = A j Ai, i = j.
Chứng minh. Ta biết thể tích tứ diện không thay đổi qua một phép tịnh tiến. Do vậy,
không hạn chế có thể giả thiết tâm mặt cầu ngoại tiếp tứ diện A1A2A3A4 chính là gốc
tọa độ. Ta có hệ phương trình tọa độ của mặt cầu qua 4 đỉnh tứ diện sau đây:
x1
2
+ y1
2
+ z1
2
− R2
= 0
2 y 2 z 2 R 2 0
x 2 + 2 + 2 =
−
2 2 2 2
x3 + y3 + z3
−
R = 0
2 y 2 z 2 R 2 0
x 4 + 4 + 4 = .
−
Từ đây suy ra
R
2
= (xi − x j )
2
+ (yi − y j )
2
+ (zi − z j )
2
+ xix j + yiy j + ziz j
2
với mọi i, j = 1, 2, 3, 4, i = j. Đặt
ti j = R
2
− xix j − yiy j − ziz j .
Khi đó
ti j = t ji = (x
i −
x
j )
2
+ (
y
i −
y
j )
2
+ (
z
i −
z
j )
2
=
l
i
2
j =
l2
ji .
Tải tài liệu tại sividoc.com
Viết đề tài giá sinh viên – ZALO:0973.287.149-TEAMLUANVAN.COM
2 2 2
Tải tài liệu tại sividoc.com
Viết đề tài giá sinh viên – ZALO:0973.287.149-TEAMLUANVAN.COM
43
Xét tích sau
x1 y1 z1 R −x1
P = x2 y2 z2
R
. −
x
2
x3 y3 z3 R −x3
x4 y4 z4 R −x4
−y1 −z1 R
−y2 −z2 R
−y3 −z3 R
−y4 −z4 R
x1 y1 z1 R −x1 −x2 −x3 −x4
= x2 y2 z2
R
. −
y
1 −y2 −y3 −y4
x3 y3 z3 R −z1 −z2 −z3 −z4
x4 y4 z4 R R R R R
0 t
12
t
13
t
14
t
21 0 t
23
t
24
với ti j = t ji.
=
0
t
31
t
32
t
34
t
41
t
42
t
43 0
Vì
x1 y1 z1 R −x1
x2 y2 z2 R và −x2
x
3
y
3
z
3 R −x3
x
4
y
4
z
4 R −x4
đều bằng 6RV nên
−y1 −z1 R
−y2 −z2 R
−y3 −z3 R
−y4 −z4 R
0 l12
2
l13
2
2 2
l21
2
0 l23
2
36R
2
V
2
= 2 2
l32
2
l31
2
0
2 2
l43
2
l41
2
l42
2
2 2 2
l14
2
2 0l12
2
l13
2
l14
2
l24
2
1 l21
2
0l23
2 l24
2
2
l34
2
= .
16 l2
l2
0l2
31 32 34
2
l41
2
l42
2
l43
2
0
0
Tính định thức cuối cùng và lấy căn ta nhận được công thức xác định
bán kính mặt cầu ngoại
R =
2l13
2
l14
2
l23
2
l24
2
+ 2l12
2
l14
2
l23
2
l34
2
+ 2l12
2
l13
2
l24
2
l34
2
− l12
4
l34
4
− l13
4
l24
4
− l14
4
l23
4
.
24V
Ta có điều cần chứng minh.
Tải tài liệu tại sividoc.com
Viết đề tài giá sinh viên – ZALO:0973.287.149-TEAMLUANVAN.COM
44
Từ Mệnh đề 2.1.4 và Mệnh dề 2.2.1 ta suy ra công thức tính bán kính
mặt cầu ngoại tiếp qua độ dài 6 cạnh tứ diện.
Hệ quả 2.2.1. Tứ diện A1A2A3A4 có độ dài 6 cạnh là a = l12, b = l13, c =
l14, x = l34, y = l24, z = l23. Đặt 2S = ax + by + cz. Khi đó bán kính R của
mặt cầu ngoại tiếp tứ diện được xác định bởi công thức dưới đây:
(1) R = S(S − ax)(S − by)(S − cz) .
6V
√
(2) R = 2 2S(S − ax)(S − by)(S − cz) .
2a
2
a
2
+ b
2
− z
2
a
2
+ c
2
− y
2
a2
+ b2
− z2
2b2 b2
+ c2
− x2
a2
+ c2
− y2
b2
+ c2
− x2
2c
2
Chứng minh. (1) Theo Bất đẳng thức Ptolemy, có ax + by cz,
by + cz ax và
cz + ax by, Dễ dàng kiểm tra hệ thức
16S(S − ax)(S − by)(S − cz)
= 2l13
2
l14
2
l23
2
l24
2
+ 2l12
2
l14
2
l32
2
l34
2
+ 2l12
2
l13
2
l42
2
l43
2
− l12
4
l34
4
− l13
4
l24
4
− l14
4
l23
4
.
Do vậy ta có
R = S(S − ax)(S − by)(S − cz) .
6V
Dễ dàng có (2) để xác định R qua 6 cạnh tứ diện
2√ 2
S(S − ax)(S − by)(S − cz)
R = .
2a
2
a
2
+ b
2
− z
2
a
2
+ c
2
− y
2
a2
+ b2
− z2
2b
2 b2
+ c2
− x2
a
2
+ c
2
− y
2
b
2
+ c
2
− x
2
2c2
Ta có điều phải chứng minh.
Hệ quả 2.2.2. Tứ diện A1A2A3A4 có độ dài 6 cạnh là a = l12, b = l13 và
diện tích toàn phần là s. Gọi R, r là bán kính mặt cầu ngoại, nội tiếp của
tứ diện. Ta có hệ thức
Rr =
2l13
2
l14
2
l23
2
l24
2
+ 2l12
2
l14
2
l32
2
l34
2
+ 2l12
2
l13
2
l42
2
l43
2
− l12
4
l34
4
− l13
4
l24
4
− l14
4
l23
4
.
8s
Tải tài liệu tại sividoc.com
Viết đề tài giá sinh viên – ZALO:0973.287.149-TEAMLUANVAN.COM
45
Chứng minh. Kết quả được suy ra từ công thức thể tích V = 3sr và Mệnh
đề 2.2.1.
Tải tài liệu tại sividoc.com
Viết đề tài giá sinh viên – ZALO:0973.287.149-TEAMLUANVAN.COM
46
Chương 3
Vận dụng giải bài thi học sinh giỏi
Ví dụ 3.0.1. Qua một điểm O trong không gian ta dựng n đường thẳng Odi. Giả sử
góc giữa hai đường thẳng bất kỳ thuộc n đường thẳng Odi không nhỏ hơn α với
0<α<
π
. Chứng minh rằng n
1
.
2
1−COS
α
2
Bài giải. Ta dựng hình cầu (C) tâm O với bán kính 1 và các hình nón tròn
xoay đỉnh O, trục Odi, với đường sinh Oli sao cho ∠liOdi =
α
2 , i = 1, 2, .
. . , n. Ký hiệu Ki là phần thể tích hình nón dựng như trên nằm trong hình
cầu (C). Theo công thức tính thể tích khối tròn xoay ta có
1
4 α
V = Ki = 2 π f (x)
2
d x = π 1− COS ,
3 2
0
trong đó
α α
f (x) = TAN x với 0 x COS
2 2
và
α
với
f (x) =1 − x
2
COS x 1.
2
Vì nV VC =
4
π .1
3
nên n
1
.
3
1−COS
α
2
Ví dụ 3.0.2. Có 2017 điểm nằm bên trong một hình lập phương với độ
dài cạnh bằng 9. Chứng minh rằng, có hai điểm trong số 2017 điểm trên
với khoảng cách nhỏ hơn 1.
Bài giải. Giả sử khoảng cách giữa hai điểm bất khỳ thuộc 2017 điểm đã cho cách
nhau một khoảng cách lớn hơn hoặc bằng 1. Khi đó, các hình cầu với tâm là một
Tải tài liệu tại sividoc.com
Viết đề tài giá sinh viên – ZALO:0973.287.149-TEAMLUANVAN.COM
47
trong 2017 điểm đã cho và bán kính
1
2 có phần trong không giao nhau. Giả sử hình
lập phương ABCD.A′
B′
C′
D′
độ dài cạnh 9 chứa 2017 điểm đã cho. Dựng hình lập
phương A1B1C1D1.A′
1B′
1C1
′
D′
1 chứa hình lập phương ABCD.A′
B′
C′
D′
ở trong sao
cho mặt tương ứng song song và cách nhau một khoảng cách
1
2 , chẳng hạn
(ABCD) và hai mặt phẳng này cách nhau một khoảng cách
1
. Vì 2
hình lập phương A1B1C1D1.A
′
1B
′
1C1
′
D
′
1 chứa tất cả các phần trong
của 2017 hình cầu nên
1000 = 10
3
> 2017.
4
3 .π .
1
2
3
>
1000, vô lý. Vậy, điều giả sử là sai.
Ví dụ 3.0.3. Qua điểm M thuộc mặt Ellipsoid (E ) :
x2
+
y2
+
z2
= 1 ta dựng các
a2 b2 c2
mặt phẳng vuông góc với các trục tọa độ Ox, Oy, Oz, và có các diện tích
thiết diện Sx, Sy, Sz, tương ứng. Xác định giả trị lớn nhất của P = Sx.Sy.Sz.
Bài giải. Giả sử M(x0, y0, z0) ∈ (E ). Khi
đó
x0
2
+
y0
2
+
z0
2
= 1.
a2
b2
c2
phẳng qua M vuông góc với Ox cắt (E ) có phương trình (Ex) :
Diện tích Sx của (Ex) bằng
Thiết diện do mặt
y2 z2
x0
2
+ = 1 − .
b
2
c
2
a
2
x2
S = π bc 1 − 0 .
x a
2
Hoàn toàn tương tự, ta còn có hai kết quả
y0
2
z0
2
Sy = π ca 1 − , Sz = π ab 1 − .
b
2
c
2
Vậy, ta dễ dàng nhận được kết quả
aSx + bSy + cSz = π abc 3 −
x0
2
y0
2
z0
2
= 2π abc.
− −
a
2
b
2
c
2
Vì
2π abc = aSx + bSy + cSz 3 3
abcSx.Sy.Sz
nên
8π
3
(abc)
2
Sx.Sy.Sz .
27
Tải tài liệu tại sividoc.com
Viết đề tài giá sinh viên – ZALO:0973.287.149-TEAMLUANVAN.COM
48
Từ đây suy ra rằng, P đạt giá trị lớn nhất bằng
8π
3
(abc)2
27
khi
2π abc
aSx = bSy = cSz = .
3
Giải
2π abc x0
2
= aSx = π abc 1 −
3 a2
được
a
x0 = ± √ ;
3
Tương tự
b c
y0 = ±√ ;z0 = ± √ .
3 3
Ta có thể chọn M
a b c
, chẳng hạn.
√ ,√ , √
3 3 3
Ví dụ 3.0.4. Bên trong một hình cầu bán kính R trong không gian Euclid 3 chiều có
một khối đa diện lồi. Đánh số mỗi cạnh khối đa diện lồi qua các số 1, 2, . . . , n. Giả
sử cạnh thứ i có độ dài li và góc nhị diện của khối đa diện thuộc cạnh này là
α được đo bằng radian. Chứng minh rằng
n
(π
−
α )8π.R.
∑ l
i i=1 i i
Bài giải. Gọi khối đa diện lồi là L. Ký hiệu Kr là tập tất cả các điểm bên ngoài khối
đa diện và cách khối đa diện với khoảng cách không quá r. Ký hiệu H1 là một phần
của Kr bao gồm tất cả các điểm bên ngoài khối đa diện và nó gần đỉnh khối đa
diện nhất; Ký hiệu H2 là một phần của Kr bao gồm tất cả các điểm mà điểm gần nó
nhất là một điểm trong thuộc cạnh khối đa diện. Như vậy H1 ∩ H2 = 0/. Tất cả các
điểm thuộc H1 ∪ H2 là những điểm trong thuộc mặt cầu bán kính R + r. Do
vậy, tổng thể tích của H1 và H2 không vượt quá
4
π (R + r)
3
.
3
Tính thể tích tập H2 : Dựng các hình trụ cạnh thứ i với độ dài cạnh li và góc phẳng
nhị diện là 2π − αi và thể tích phần giao bằng
1
(π − αi)r
2
li.
2
Vậy thể tích H2 đúng bằng 1r2 ∑ (π α )l . Thể tích H1 bằng 4π r
3
. Khi đó ta có
bất đẳng thức 2 i=1 − i i 3
4 1 4
π r
3
+ r
2
∑(π − αi)li π (R + r)
3
.
3 2 3
i=1
Tải tài liệu tại sividoc.com
Viết đề tài giá sinh viên – ZALO:0973.287.149-TEAMLUANVAN.COM
49
Vậy
∑(π − αi)li
8π 3R
2
R3
3R + + .
3 r r2
i=1
Cho r → ∞ ta có
∑(π − αi)li 8π R.
i=1
Ta có điều phải chứng minh.
Ví dụ 3.0.5. Cho hình chóp SABC. Một mặt phẳng cắt ba cạnh bên SA, SB, SC tại
A1, B1,C1. Chứng minh rằng, với bất kỳ điểm trong O của tam giác A1B1C1 ta có
7 4
4 √ 2
V
SABC 4
VAOB C +4
VBOCA + 4
VCOA B .
4 1
1 1 1 1 1
Bài giải. Đã biết
V
AOB1C1 =
V
AOB1C1 .
V
AA1B1C1 .
V
ASB1C1 = s(OB1C1) .AA1 .SB1 .SC1 .
V V V s(A1B1C1)
V AS SB SC
SABC AA1B1C1 ASB1C1 SABC
Từ đây suy ra
s(OB1C1)
4 4
2
V
AOB1C1 + 2AA1 +SB1 +SC1 .
s(A1B1C1)
V
SABC ASSBSC
Tương tự, ta còn có
4 4
2
V
BOC1A1
V
SABC
4 4
2
V
COA1B1
V
SABC
Cộng bất đẳng thức được
7
4 V
4 √ 2
SABC
4
Ta có điều phải chứng minh.
s(OC1A1)
+ 2
BB1
+
SC1
+
SA1
s(A1B1C1) BS SC SA
s(OA1B1) + 2CC1 +SA1 +SB1 .
s(A1B1C1) CS SA SB
4 V
AOB1C1 + 4 V
BOC1A1 + 4 V
COA1B1 .
Ví dụ 3.0.6. Cho tam giác ABC với độ dài cạnh BC = a,CA = b, AB = c. Dựng
ra phía ngoài ba tam giác đều BCA1,CAB1, ABC1. Dựng tiếp ra phía ngoài tam
giác A1B1C1 ba tam giác đều B1C1A2, C1A1B2, A1B1C2. Chứng minh rằng:
(1) AA1 = BB1 = CC1 và AA1, BB1,CC1 đồng quy tại điểm V.
Tải tài liệu tại sividoc.com
Viết đề tài giá sinh viên – ZALO:0973.287.149-TEAMLUANVAN.COM
50
(2) Với O1, O2, O3 là tâm các tam giác đều BCA1, CAB1, ABC1, ba đường
thẳng AO1, BO2,CO3 đồng quy tại W. Hơn nữa, ta còn có đồng nhất thức
AO
2
1 +
2
3 a
2
= BO
2
2 +
2
3 b
2
= CO
2
3 +
2
3 c
2
.
(3) Tiếp tục quá trình như trên để có ΔAnBnCn. Khi đó A, A1, . . . , An thẳng
hàng và tìm giá trị lớn nhất của AnAn+1 khi b, c cố định, còn a thay đổi.
(4) Các bộ (BC, O2O3, B1C1), (CA, O3O1,C1A1), (AB, O1O2, A1B1)
gồm ba đường thẳng hoặc đồng quy hoặc song song.
(5) Với tâm đường tròn ngoại tiếp O của ΔABC, ba điểm O,V,W thẳng hàng.
Bài giải. (1) Tương ứng A, B,C ba số phức zA, zB, zC . Ta dễ dàng xác định được
zA1 = zC + e
iu
(zB − zC )
= z iu
(z z )
zB
A
+ e
1 C A
−
iu
z = z + e (z
−
z )
C1 B iu A B π
z
A2
= z + e (z
B1
z ) với u = .
C1 C1 3
−
Từ
Tải tài liệu tại sividoc.com
Viết đề tài giá sinh viên – ZALO:0973.287.149-TEAMLUANVAN.COM
zC + e
iu
(zB1 − zC ) = zC + e
iu
(zA + e
iu
(zC − zA) − zC )
Tải tài liệu tại sividoc.com
Viết đề tài giá sinh viên – ZALO:0973.287.149-TEAMLUANVAN.COM
51
ta suy ra hệ thức
zC + e
iu
(zB1 − zC ) = zC (1 − e
iu
+ e
2iu
) + zA(e
iu
− e
2iu
) = zA
vì
e
iu
= −1, e
3iu
+ 1 = 0
nên
1 − eiu
+ e2iu
= 0, eiu
− e2iu
= 1.
Như vậy ta có
zA1 − zA = e
iu
(zB − zB1 )
và suy ra AA1 = BB1. Tương tự BB1 = CC1. Do góc giữa AA1, BB1,CC1 đều bằng
π /3 nên AA1, BB1,CC1 đồng quy tại một điểm (Ta đặt giao điểm M =
AA1 × BC, N = BB1 ×CA và P = CC1 × AB. Biến đổi tỷ số sau:
T = MB .NC .PA =
S
ABA1 .
S
BCB1 .
S
CAC1
MC NA PBSACA1
S
BAB1
S
CBC1
c SIN(B + 60
0
) a SIN(C + 60
0
) b SIN(A + 60
0
)
=
b SIN(C + 600
)
.
c SIN(A + 600
)
.
a SIN(B + 600
)
= 1.
Như vậy, ba đường thẳng AA1, BB1,CC1 đồng quy theo Định lý Ceva).
(2) Dễ dàng kiểm tra
3 zO2 + e
iu
(zO3 − zO2 ) = 3zO1 .
Vậy ΔO1O2O3 đều. Ta đặt giao điểm M = AO1 × BC, N = BO2 × CA và P =
CO3 × AB. Biến đổi tỷ số qua diện tích
T = MB .NC .PA =
S
ABO1 .
S
BCO2 .
S
CAO3
MC NA PBSACO1
S
BAO2
S
CBO3
c SIN(B + 30
0
) a SIN(C + 30
0
) b SIN(A + 30
0
)
=
b SIN(C + 300
)
.
c SIN(A + 300
)
.
a SIN(B + 300
)
= 1.
Như vậy, ba đường thẳng AO1, BO2,CO3 đồng quy theo Định lý Ceva.
(3) Dễ dàng chỉ ra
z
A1
+ z
A2
= 2z
A
.
Vậy A là trung điểm của A1A2. Từ A1A2 = 2AA1 = 2BB1 (b + c). Vậy A1A2
lớn nhất bằng 2(b + c) khi A =
2
3
π
. Từ đó có a để A1A2 lớn nhất.
Tải tài liệu tại sividoc.com
Viết đề tài giá sinh viên – ZALO:0973.287.149-TEAMLUANVAN.COM
52
(4) Giả sử đường thẳng AB cắt đường thẳng A1B1 tại S. Áp dụng Định lý
hàm sin cho ΔSBA1 và ΔSAB1 có
SA1
=
a
và
SB1
=
b
.
SIN(B + 60
0
)
SIN
∠ BSA1 SIN(A + 60
0
)
SIN
∠ ASB1
Vậy
SA1 a SIN(B + 600
)
= .
SB1 b SIN(A + 60
0
)
Giả sử đường thẳng O1O2 cắt đường thẳng A1B1 tại S
′
. Áp dụng Định lý
Menelaus có
S′A
1 .
O
2
B
1 .
O
1
O
= 1
S
′
B1 O2O O1A1
và
S′A
1 =
O
2
O
.
O
1
A
1 =
aO
2
O
.
S′
B1 O1O O2B1 bO1O
Do
SIN(B + 60
0
)
O2O
=
OC SIN 60
0
nên
S′
A1 =aO2O =a SIN(B + 600
) =SA1 .
S
′
B1 b SIN(A + 60
0
)
bO1O SB1
Từ đây suy ra S ≡ S′
≡ Sc hay ba đường thẳng AB, O1O2, A1B1 đồng quy.
Tải tài liệu tại sividoc.com
Viết đề tài giá sinh viên – ZALO:0973.287.149-TEAMLUANVAN.COM
53
(5) Xét hai tam giác A1O1A và O2B1B Ba giao điểm A1O1 × BO2 = O, AA1 ×
BB1 = V và AO1 × BO2 = W thẳng hàng theo Định lý 1.4.1, Định lý Desargues.
Nhận xét 3.0.1. Nếu gọi các điểm A
′
, B
′
,C
′
là trung điểm cạnh BC, CA, AB thì
A′
O1 =
A
′
A1
, B′
O2 =
B
′
B1
, C′
O3 =
C
′
C1
ΔA
′
BO1 ∼ ΔA
′
A1B.
3 3 3
Ví dụ 3.0.7. Cho tam giác ABC. Dựng ra phía ngoài hai tam giác đều
BCA1, ABC1. Gọi M, N, P là trung điểm CA, BC1, BA1 , tương ứng.
Chứng minh tam giác MN P đều.
Bài giải. Đặt tọa vị A, B,C, M, N, P là sáu số phức a, b, c, m, n, p, tương
ứng. Khi đó zA1 = c + ei u( b − c ), zC 1 = b + ei u(a − b ), u = 3
π
a + c
b + z
C1
b + z
A1 iu
, α = e
m = , n = , p = .
2 2 2
Vì α
2
− α + 1 = 0 nên
(m
−
n)α = (a + c − 2b)α − (a − b)α
2
=(a − b) − (b − c)α .
2 2
Như vậy (m − n)α = m − p và suy ra tam giác MN P đều.
Tải tài liệu tại sividoc.com
Viết đề tài giá sinh viên – ZALO:0973.287.149-TEAMLUANVAN.COM
54
Ví dụ 3.0.8. Cho tam giác ABC với độ dài cạnh BC = a,CA = b, AB = c. Dựng
ra phía ngoài ba tam giác đều BCA1,CAB1, ABC1. Dựng tiếp ra phía ngoài tam
giác A1B1C1 ba tam giác đều sau đây: B1C1A2,C1A1B2, A1B1C2. Gọi M, N, P
là trung điểm BC,CA, AB; M1, N1, P1 là trung điểm B1C1, C1A1, A1B1, và M2,
N2 , P2 là trung điểm B2C2,C2A2, A2B2, tương ứng. Chứng minh rằng
(1) AA1 2MM1, BB1 2N N1 ,CC1 2PP1; MM1 , N N1, PP1 đồng quy.
= = =
(2) A2M2 AM1 MM0, trong đó M0 là trung điểm của AA1.
= =
(4) Hạ MM2 ⊥ B1C1, N N2 ⊥ C1A1, PP2 ⊥ A1B1. Khi đó MM2, N N2
và PP2 đồng quy.
Bài giải. (1) Tương ứng A, B,C ba số phức zA, zB,
zC . Ta có
2z
M =
z
B +
z
C ,
2z
M1 =
z
B1 +
z
C1 .
−→
Viết IJ = zJ − zI . Khi đó
−→
−→ iu
2MM1 = 2zM1 − 2zM = zB1 + zC1 − zB − zC = zA − zC + e (zC − zB) = A1A.
Tải tài liệu tại sividoc.com
Viết đề tài giá sinh viên – ZALO:0973.287.149-TEAMLUANVAN.COM
55
Vậy AA1 MM1, AA1 = 2MM1. Ba đoạn MM1, N N1 , PP1 đồng quy được suy ra từ
việc xét ΔMN P và phép quay eiu .
(2) Từ AA1 2MM1 suy ra AM1
= =
suy ra A2M2 AM1 MM0.
MM0. Ta lại có AM1
=
A2M2 và cũng dễ dàng
= =
(3) Sử dụng kết quả AA1 = BB1 = CC1 = d và công thức đường trung tuyến ta có
2MB1
2
= d
2
+ b
2
−
a
2
a
2
, 2MC1
2
= d
2
+ c
2
− .
2 2
Vậy
MB
2
−
MC
2
=b
2
− c
2
1 1 2
hay
M2B
2
−
M2C
2
=b
2
− c
2
.
1 1 2
Tương tự, có
N2C
2
−
N2A
2
=c
2
− a
2
và P2A
2
P2B
2
= a
2
− b
2
.
1 1 2 1
−
1 2
Khi đó
M2B2
1 − M2C1
2
+ N2C1
2
− N2A2
1 + P2A2
1 − P2B2
1 = 0.
Do đó MM2 , N N2, PP2 đồng quy tại một điểm.
Ví dụ 3.0.9. Cho tam giác ABC với độ dài cạnh BC = a,CA = b, AB = c. Xác định
điểm M để MA + MB + MC là nhỏ nhất khi
∠A
2π
.
3
Bài giải. Dựng tam giác đều BCA1 ra phía ngoài tam giác ABC. Khi M là điểm
Torricelli của tam giác ABC ta có MA + MB + MC = AA1. Khi N = M thì theo
Bất đẳng thức Ptolemy ta có N B.CA1 + NC.BA1
NA1 .BC hay N B + NC
NA1
và suy ra NA + N B + NC NA + NA1 AA1. Do vậy MA + MB + MC nhỏ
nhất bằng AA1 khi M là điểm Torricelli của ΔABC.
Tải tài liệu tại sividoc.com
Viết đề tài giá sinh viên – ZALO:0973.287.149-TEAMLUANVAN.COM
56
Kết luận
1 Những kết quả đã đạt được
Luận văn “Phương pháp diện tích và thể tích trong hình học sơ cấp”
đã đạt được các kết quả sau:
1. Trình bày về phương pháp diện tích trong hình học. Những vấn đề
đã trình bày là liên quan đến Định lý Pythagore, Định lý Stewart,
Ceva, Menelaus và Bất đẳng thức Erdos¨-Mordell cho đa giác.
2. Trình bày về phương pháp thể tích, các khía cạnh của thể tích thông qua định
thức và mối quan hệ liên quan đến bán kính của mặt cầu nội và ngoại tiếp.
3. Trình bày chi tiết những vận dụng các kết quả đạt được để giải các
bài thi học sinh giỏi.
2 Đề xuất một số hướng nghiên cứu tiếp theo
Sau những kết quả đã đạt được trong luận văn, tác giả đặt phương
hướng nghiên cứu tiếp theo là làm rõ và khai thác sâu sắc hơn những
phương pháp đã trình bày trong luận văn thông qua các bài thi học sinh
giỏi, các bài toán nâng cao. Tác giả sẽ chọn ra một số bài tập điển hình sau
đó sẽ khai thác và phát triển. Đồng thời, tác giả cũng cố gắng nghiên cứu
thêm các phương pháp giải toán điển hình và truyền thống khác trong hình
học vào giải toán, chẳng hạn như phương pháp tọa độ và ứng dụng.
Tải tài liệu tại sividoc.com
Viết đề tài giá sinh viên – ZALO:0973.287.149-TEAMLUANVAN.COM
57
Tài liệu tham khảo
Tiếng Việt
[1] Nguyễn Văn Mậu, Đàm Văn Nhỉ (2015), Đồng nhất thức và phương
pháp tọa độ trong hình học, Nhà xuất bản ĐHQG Hà Nội.
[2] Trần Tuấn Nam, Đàm Văn Nhỉ, Trần Trung Tình, Nguyễn Anh Tuấn (2016),
Giáo trình Hình học sơ cấp, Nhà xuất bản ĐHSP Thành phố Hồ Chí Minh.
Tiếng Anh
[3] H.S.M. Coxeter, S. L. Greitzer (1967), Geometry Revisited, The
Mathemati-cal Association America.
[4] A. Pogorelov (1987), Geometry, Mir Moscow.

More Related Content

Similar to Phương Pháp Diện Tích Và Thể Tích Trong Hình Học Sơ Cấp.doc

Chuyên đề Hệ thức lượng trong tam giác vuông
Chuyên đề Hệ thức lượng trong tam giác vuôngChuyên đề Hệ thức lượng trong tam giác vuông
Chuyên đề Hệ thức lượng trong tam giác vuông
Toán THCS
 
Xuctu.com de thi_tuyen_sinh_10_lhp_tdn_chuyen_dhsp(hcm)
Xuctu.com de thi_tuyen_sinh_10_lhp_tdn_chuyen_dhsp(hcm)Xuctu.com de thi_tuyen_sinh_10_lhp_tdn_chuyen_dhsp(hcm)
Xuctu.com de thi_tuyen_sinh_10_lhp_tdn_chuyen_dhsp(hcm)
Minh Đức
 
Cđ dãy số viết theo quy luật
Cđ dãy số viết theo quy luậtCđ dãy số viết theo quy luật
Cđ dãy số viết theo quy luật
Cảnh
 

Similar to Phương Pháp Diện Tích Và Thể Tích Trong Hình Học Sơ Cấp.doc (20)

Bất đẳng thức Trong lớp các hàm lượng giác và lượng giác ngược.docx
Bất đẳng thức Trong lớp các hàm lượng giác và lượng giác ngược.docxBất đẳng thức Trong lớp các hàm lượng giác và lượng giác ngược.docx
Bất đẳng thức Trong lớp các hàm lượng giác và lượng giác ngược.docx
 
Phương Trình Bậc Bốn Và Các Hệ Thức Hình Học Trong Tứ Giác Hai Tâm.doc
Phương Trình Bậc Bốn Và Các Hệ Thức Hình Học Trong Tứ Giác Hai Tâm.docPhương Trình Bậc Bốn Và Các Hệ Thức Hình Học Trong Tứ Giác Hai Tâm.doc
Phương Trình Bậc Bốn Và Các Hệ Thức Hình Học Trong Tứ Giác Hai Tâm.doc
 
Đề tài: Phương pháp giải hệ phương trình và hệ bất phương trình
Đề tài: Phương pháp giải hệ phương trình và hệ bất phương trìnhĐề tài: Phương pháp giải hệ phương trình và hệ bất phương trình
Đề tài: Phương pháp giải hệ phương trình và hệ bất phương trình
 
Phương Trình Bậc Bốn Và Các Hệ Thức Hình Học Trong Tứ Giác Hai Tâm.docx
Phương Trình Bậc Bốn Và Các Hệ Thức Hình Học Trong Tứ Giác Hai Tâm.docxPhương Trình Bậc Bốn Và Các Hệ Thức Hình Học Trong Tứ Giác Hai Tâm.docx
Phương Trình Bậc Bốn Và Các Hệ Thức Hình Học Trong Tứ Giác Hai Tâm.docx
 
M t so dạng toán Liên quan đen xác suat r i rạc và ứng dụng.docx
M t so dạng toán Liên quan đen xác suat r i rạc  và ứng dụng.docxM t so dạng toán Liên quan đen xác suat r i rạc  và ứng dụng.docx
M t so dạng toán Liên quan đen xác suat r i rạc và ứng dụng.docx
 
M T So Dạng Toán Liên Quan Đen Xác Suat R I Rạc Và Ứng Dụng.docx
M T So Dạng Toán Liên Quan Đen Xác Suat R I Rạc Và Ứng Dụng.docxM T So Dạng Toán Liên Quan Đen Xác Suat R I Rạc Và Ứng Dụng.docx
M T So Dạng Toán Liên Quan Đen Xác Suat R I Rạc Và Ứng Dụng.docx
 
Chuyên đề Hệ thức lượng trong tam giác vuông
Chuyên đề Hệ thức lượng trong tam giác vuôngChuyên đề Hệ thức lượng trong tam giác vuông
Chuyên đề Hệ thức lượng trong tam giác vuông
 
Về Phương Trình Diophantine Dạng Ax2 - By2 = C.docx
Về Phương Trình Diophantine Dạng Ax2 - By2 = C.docxVề Phương Trình Diophantine Dạng Ax2 - By2 = C.docx
Về Phương Trình Diophantine Dạng Ax2 - By2 = C.docx
 
Sáng kiến kinh nghiệm giải Hình học không gian bằng phương pháp tọa độ
Sáng kiến kinh nghiệm giải Hình học không gian bằng phương pháp tọa độSáng kiến kinh nghiệm giải Hình học không gian bằng phương pháp tọa độ
Sáng kiến kinh nghiệm giải Hình học không gian bằng phương pháp tọa độ
 
Kế hoạch dạy học theo chủ đề Hình học 9 học kì 1
Kế hoạch dạy học theo chủ đề Hình học 9 học kì 1Kế hoạch dạy học theo chủ đề Hình học 9 học kì 1
Kế hoạch dạy học theo chủ đề Hình học 9 học kì 1
 
Luận văn: Các bài toán về hệ thức lượng trong tam giác, HOT, 9đ
Luận văn: Các bài toán về hệ thức lượng trong tam giác, HOT, 9đLuận văn: Các bài toán về hệ thức lượng trong tam giác, HOT, 9đ
Luận văn: Các bài toán về hệ thức lượng trong tam giác, HOT, 9đ
 
Xuctu.com de thi_tuyen_sinh_10_lhp_tdn_chuyen_dhsp(hcm)
Xuctu.com de thi_tuyen_sinh_10_lhp_tdn_chuyen_dhsp(hcm)Xuctu.com de thi_tuyen_sinh_10_lhp_tdn_chuyen_dhsp(hcm)
Xuctu.com de thi_tuyen_sinh_10_lhp_tdn_chuyen_dhsp(hcm)
 
Bài Toán Cực Trị Với Điều Kiện Ràng Buộc Bất Đẳng Thức, Hệ Bất Đẳng Thức.docx
Bài Toán Cực Trị Với Điều Kiện Ràng Buộc Bất Đẳng Thức, Hệ Bất Đẳng Thức.docxBài Toán Cực Trị Với Điều Kiện Ràng Buộc Bất Đẳng Thức, Hệ Bất Đẳng Thức.docx
Bài Toán Cực Trị Với Điều Kiện Ràng Buộc Bất Đẳng Thức, Hệ Bất Đẳng Thức.docx
 
Phương trình diophantine dạng X2 − dy2 = ±4.docx
Phương trình diophantine dạng X2 − dy2 = ±4.docxPhương trình diophantine dạng X2 − dy2 = ±4.docx
Phương trình diophantine dạng X2 − dy2 = ±4.docx
 
Bat Phương Trình Hàm Sinh B I Các Đại Lư Ng Trung Bình B C Tùy Ý Và Các Dạng ...
Bat Phương Trình Hàm Sinh B I Các Đại Lư Ng Trung Bình B C Tùy Ý Và Các Dạng ...Bat Phương Trình Hàm Sinh B I Các Đại Lư Ng Trung Bình B C Tùy Ý Và Các Dạng ...
Bat Phương Trình Hàm Sinh B I Các Đại Lư Ng Trung Bình B C Tùy Ý Và Các Dạng ...
 
(School.antoree.com) Đề thi thử Đại học môn Toán chuyên Vĩnh Phúc lần...
(School.antoree.com) Đề thi thử Đại học môn Toán chuyên Vĩnh Phúc lần...(School.antoree.com) Đề thi thử Đại học môn Toán chuyên Vĩnh Phúc lần...
(School.antoree.com) Đề thi thử Đại học môn Toán chuyên Vĩnh Phúc lần...
 
The tich khoi da dien VDVDC
The tich khoi da dien   VDVDCThe tich khoi da dien   VDVDC
The tich khoi da dien VDVDC
 
Cđ dãy số viết theo quy luật
Cđ dãy số viết theo quy luậtCđ dãy số viết theo quy luật
Cđ dãy số viết theo quy luật
 
Hệ Thức Lượng Giác Và Ứng Dụng.doc
Hệ Thức Lượng Giác Và Ứng Dụng.docHệ Thức Lượng Giác Và Ứng Dụng.doc
Hệ Thức Lượng Giác Và Ứng Dụng.doc
 
Tuyển tập đề thi vào lớp 10 môn Toán THPT - Thầy Thích
Tuyển tập đề thi vào lớp 10 môn Toán THPT - Thầy ThíchTuyển tập đề thi vào lớp 10 môn Toán THPT - Thầy Thích
Tuyển tập đề thi vào lớp 10 môn Toán THPT - Thầy Thích
 

More from DV Viết Luận văn luanvanmaster.com ZALO 0973287149

More from DV Viết Luận văn luanvanmaster.com ZALO 0973287149 (20)

Ảnh Hưởng Của Marketing Quan Hệ Đến Lòng Trung Thành Của Khách Hàng.Tình Huốn...
Ảnh Hưởng Của Marketing Quan Hệ Đến Lòng Trung Thành Của Khách Hàng.Tình Huốn...Ảnh Hưởng Của Marketing Quan Hệ Đến Lòng Trung Thành Của Khách Hàng.Tình Huốn...
Ảnh Hưởng Của Marketing Quan Hệ Đến Lòng Trung Thành Của Khách Hàng.Tình Huốn...
 
Phát triển nguồn nhân lực tại Uỷ ban nhân dân huyện Trà Bồng, tỉnh Quảng Ngãi...
Phát triển nguồn nhân lực tại Uỷ ban nhân dân huyện Trà Bồng, tỉnh Quảng Ngãi...Phát triển nguồn nhân lực tại Uỷ ban nhân dân huyện Trà Bồng, tỉnh Quảng Ngãi...
Phát triển nguồn nhân lực tại Uỷ ban nhân dân huyện Trà Bồng, tỉnh Quảng Ngãi...
 
Báo cáo tốt Nghiệp tài chính hợp nhất tại tổng công ty Indochina gol...
Báo cáo tốt Nghiệp  tài chính hợp nhất tại tổng công ty Indochina gol...Báo cáo tốt Nghiệp  tài chính hợp nhất tại tổng công ty Indochina gol...
Báo cáo tốt Nghiệp tài chính hợp nhất tại tổng công ty Indochina gol...
 
Tạo động lực thúc đẩy nhân viên làm việc tại ngân hàng TMCP Ngoại Thương Việt...
Tạo động lực thúc đẩy nhân viên làm việc tại ngân hàng TMCP Ngoại Thương Việt...Tạo động lực thúc đẩy nhân viên làm việc tại ngân hàng TMCP Ngoại Thương Việt...
Tạo động lực thúc đẩy nhân viên làm việc tại ngân hàng TMCP Ngoại Thương Việt...
 
Phát triển công nghiệp trên địa bàn Thành phố Tam Kỳ, Tỉnh Quảng Na...
Phát triển công nghiệp trên địa bàn Thành phố Tam Kỳ, Tỉnh Quảng Na...Phát triển công nghiệp trên địa bàn Thành phố Tam Kỳ, Tỉnh Quảng Na...
Phát triển công nghiệp trên địa bàn Thành phố Tam Kỳ, Tỉnh Quảng Na...
 
Giải pháp phát triển cho vay xuất nhập khẩu tại ngân hàng NN&PTNN ch...
Giải pháp phát triển cho vay xuất nhập khẩu tại ngân hàng NN&PTNN ch...Giải pháp phát triển cho vay xuất nhập khẩu tại ngân hàng NN&PTNN ch...
Giải pháp phát triển cho vay xuất nhập khẩu tại ngân hàng NN&PTNN ch...
 
Hoàn thiện công tác lập báo cáo tài chính hợp nhất tại tổng công ...
Hoàn thiện công tác lập báo cáo tài chính hợp nhất tại tổng công ...Hoàn thiện công tác lập báo cáo tài chính hợp nhất tại tổng công ...
Hoàn thiện công tác lập báo cáo tài chính hợp nhất tại tổng công ...
 
Luận Văn Thạc Sĩ Quản trị thành tích nhân viên tại Cục Hải quan TP Đà Nẵng.doc
Luận Văn Thạc Sĩ  Quản trị thành tích nhân viên tại Cục Hải quan TP Đà Nẵng.docLuận Văn Thạc Sĩ  Quản trị thành tích nhân viên tại Cục Hải quan TP Đà Nẵng.doc
Luận Văn Thạc Sĩ Quản trị thành tích nhân viên tại Cục Hải quan TP Đà Nẵng.doc
 
Hoàn thiện công tác quản lý thuế thu nhập cá nhân tại cục thuế Tỉ...
Hoàn thiện công tác quản lý thuế thu nhập cá nhân tại cục thuế Tỉ...Hoàn thiện công tác quản lý thuế thu nhập cá nhân tại cục thuế Tỉ...
Hoàn thiện công tác quản lý thuế thu nhập cá nhân tại cục thuế Tỉ...
 
Đề Tài Phát triển bền vững nông nghiệp Huyện Ba Tơ, Tỉnh Quảng Ngãi....
Đề Tài Phát triển bền vững nông nghiệp Huyện Ba Tơ, Tỉnh Quảng Ngãi....Đề Tài Phát triển bền vững nông nghiệp Huyện Ba Tơ, Tỉnh Quảng Ngãi....
Đề Tài Phát triển bền vững nông nghiệp Huyện Ba Tơ, Tỉnh Quảng Ngãi....
 
Hoàn thiện công tác bảo trợ xã hội trên địa bàn huyện Phong Điền, tỉnh Thừa T...
Hoàn thiện công tác bảo trợ xã hội trên địa bàn huyện Phong Điền, tỉnh Thừa T...Hoàn thiện công tác bảo trợ xã hội trên địa bàn huyện Phong Điền, tỉnh Thừa T...
Hoàn thiện công tác bảo trợ xã hội trên địa bàn huyện Phong Điền, tỉnh Thừa T...
 
Đề Tài Luận VănPhát triển sản phẩm du lịch tại thành phố Đà Nẵng.doc
Đề Tài Luận VănPhát triển sản phẩm du lịch tại thành phố Đà Nẵng.docĐề Tài Luận VănPhát triển sản phẩm du lịch tại thành phố Đà Nẵng.doc
Đề Tài Luận VănPhát triển sản phẩm du lịch tại thành phố Đà Nẵng.doc
 
Đào tạo nghề cho lao động thuộc diện thu hồi đất trên địa bàn Thàn...
Đào tạo nghề cho lao động thuộc diện thu hồi đất trên địa bàn Thàn...Đào tạo nghề cho lao động thuộc diện thu hồi đất trên địa bàn Thàn...
Đào tạo nghề cho lao động thuộc diện thu hồi đất trên địa bàn Thàn...
 
Tóm Tắt Luận Văn Thạc Sĩ Quản Trị Kinh Doanh Xây dựng chính sách Marketing tạ...
Tóm Tắt Luận Văn Thạc Sĩ Quản Trị Kinh Doanh Xây dựng chính sách Marketing tạ...Tóm Tắt Luận Văn Thạc Sĩ Quản Trị Kinh Doanh Xây dựng chính sách Marketing tạ...
Tóm Tắt Luận Văn Thạc Sĩ Quản Trị Kinh Doanh Xây dựng chính sách Marketing tạ...
 
Đề Tài Nghiên cứu rủi ro cảm nhận đối với mua hàng thời trang trực tuyến.docx
Đề Tài Nghiên cứu rủi ro cảm nhận đối với mua hàng thời trang trực tuyến.docxĐề Tài Nghiên cứu rủi ro cảm nhận đối với mua hàng thời trang trực tuyến.docx
Đề Tài Nghiên cứu rủi ro cảm nhận đối với mua hàng thời trang trực tuyến.docx
 
Giải pháp nâng cao động lực thúc đẩy người lao động tại công ty khai...
Giải pháp nâng cao động lực thúc đẩy người lao động tại công ty khai...Giải pháp nâng cao động lực thúc đẩy người lao động tại công ty khai...
Giải pháp nâng cao động lực thúc đẩy người lao động tại công ty khai...
 
Giải pháp phát triển dịch vụ ngân hàng điện tử tại ngân hàng đầu ...
Giải pháp phát triển dịch vụ ngân hàng điện tử tại ngân hàng đầu ...Giải pháp phát triển dịch vụ ngân hàng điện tử tại ngân hàng đầu ...
Giải pháp phát triển dịch vụ ngân hàng điện tử tại ngân hàng đầu ...
 
Giải pháp phát triển dịch vụ ngân hàng điện tử tại ngân hàng đầu ...
Giải pháp phát triển dịch vụ ngân hàng điện tử tại ngân hàng đầu ...Giải pháp phát triển dịch vụ ngân hàng điện tử tại ngân hàng đầu ...
Giải pháp phát triển dịch vụ ngân hàng điện tử tại ngân hàng đầu ...
 
Quản trị quan hệ khách hàng tại Chi nhánh Viettel Đà Nẵng – Tập đoàn Viễn thô...
Quản trị quan hệ khách hàng tại Chi nhánh Viettel Đà Nẵng – Tập đoàn Viễn thô...Quản trị quan hệ khách hàng tại Chi nhánh Viettel Đà Nẵng – Tập đoàn Viễn thô...
Quản trị quan hệ khách hàng tại Chi nhánh Viettel Đà Nẵng – Tập đoàn Viễn thô...
 
Đề Tài Đánh giá thành tích đội ngũ giảng viên trường Đại Học Phạm ...
Đề Tài Đánh giá thành tích đội ngũ giảng viên trường Đại Học Phạm ...Đề Tài Đánh giá thành tích đội ngũ giảng viên trường Đại Học Phạm ...
Đề Tài Đánh giá thành tích đội ngũ giảng viên trường Đại Học Phạm ...
 

Recently uploaded

VẤN ĐỀ 12 VI PHẠM HÀNH CHÍNH VÀ.pptx
VẤN ĐỀ 12 VI PHẠM HÀNH CHÍNH VÀ.pptxVẤN ĐỀ 12 VI PHẠM HÀNH CHÍNH VÀ.pptx
VẤN ĐỀ 12 VI PHẠM HÀNH CHÍNH VÀ.pptx
Gingvin36HC
 

Recently uploaded (20)

4.NGÂN HÀNG KĨ THUẬT SỐ-slide CHƯƠNG 3.pptx
4.NGÂN HÀNG KĨ THUẬT SỐ-slide CHƯƠNG 3.pptx4.NGÂN HÀNG KĨ THUẬT SỐ-slide CHƯƠNG 3.pptx
4.NGÂN HÀNG KĨ THUẬT SỐ-slide CHƯƠNG 3.pptx
 
22 ĐỀ THI THỬ TUYỂN SINH TIẾNG ANH VÀO 10 SỞ GD – ĐT THÁI BÌNH NĂM HỌC 2023-2...
22 ĐỀ THI THỬ TUYỂN SINH TIẾNG ANH VÀO 10 SỞ GD – ĐT THÁI BÌNH NĂM HỌC 2023-2...22 ĐỀ THI THỬ TUYỂN SINH TIẾNG ANH VÀO 10 SỞ GD – ĐT THÁI BÌNH NĂM HỌC 2023-2...
22 ĐỀ THI THỬ TUYỂN SINH TIẾNG ANH VÀO 10 SỞ GD – ĐT THÁI BÌNH NĂM HỌC 2023-2...
 
CÁC NHÂN TỐ ẢNH HƯỞNG ĐẾN HIỆU QUẢ HOẠT ĐỘNG CỦA NGÂN HÀNG THƯƠNG MẠI CỔ PHẦN...
CÁC NHÂN TỐ ẢNH HƯỞNG ĐẾN HIỆU QUẢ HOẠT ĐỘNG CỦA NGÂN HÀNG THƯƠNG MẠI CỔ PHẦN...CÁC NHÂN TỐ ẢNH HƯỞNG ĐẾN HIỆU QUẢ HOẠT ĐỘNG CỦA NGÂN HÀNG THƯƠNG MẠI CỔ PHẦN...
CÁC NHÂN TỐ ẢNH HƯỞNG ĐẾN HIỆU QUẢ HOẠT ĐỘNG CỦA NGÂN HÀNG THƯƠNG MẠI CỔ PHẦN...
 
40 ĐỀ LUYỆN THI ĐÁNH GIÁ NĂNG LỰC ĐẠI HỌC QUỐC GIA HÀ NỘI NĂM 2024 (ĐỀ 31-39)...
40 ĐỀ LUYỆN THI ĐÁNH GIÁ NĂNG LỰC ĐẠI HỌC QUỐC GIA HÀ NỘI NĂM 2024 (ĐỀ 31-39)...40 ĐỀ LUYỆN THI ĐÁNH GIÁ NĂNG LỰC ĐẠI HỌC QUỐC GIA HÀ NỘI NĂM 2024 (ĐỀ 31-39)...
40 ĐỀ LUYỆN THI ĐÁNH GIÁ NĂNG LỰC ĐẠI HỌC QUỐC GIA HÀ NỘI NĂM 2024 (ĐỀ 31-39)...
 
XÂY DỰNG KẾ HOẠCH KINH DOANH CHO CÔNG TY KHÁCH SẠN SÀI GÒN CENTER ĐẾN NĂM 2025
XÂY DỰNG KẾ HOẠCH KINH DOANH CHO CÔNG TY KHÁCH SẠN SÀI GÒN CENTER ĐẾN NĂM 2025XÂY DỰNG KẾ HOẠCH KINH DOANH CHO CÔNG TY KHÁCH SẠN SÀI GÒN CENTER ĐẾN NĂM 2025
XÂY DỰNG KẾ HOẠCH KINH DOANH CHO CÔNG TY KHÁCH SẠN SÀI GÒN CENTER ĐẾN NĂM 2025
 
NGÂN HÀNG KĨ THUẬT SỐ-slide CHƯƠNG 1 B 1 2024.pptx
NGÂN HÀNG KĨ THUẬT SỐ-slide CHƯƠNG 1 B 1 2024.pptxNGÂN HÀNG KĨ THUẬT SỐ-slide CHƯƠNG 1 B 1 2024.pptx
NGÂN HÀNG KĨ THUẬT SỐ-slide CHƯƠNG 1 B 1 2024.pptx
 
Báo cáo tốt nghiệp Đánh giá thực trạng an toàn vệ sinh lao động và rủi ro lao...
Báo cáo tốt nghiệp Đánh giá thực trạng an toàn vệ sinh lao động và rủi ro lao...Báo cáo tốt nghiệp Đánh giá thực trạng an toàn vệ sinh lao động và rủi ro lao...
Báo cáo tốt nghiệp Đánh giá thực trạng an toàn vệ sinh lao động và rủi ro lao...
 
Hoàn thiện hoạt động kiểm soát rủi ro tín dụng trong cho vay doanh nghiệp tại...
Hoàn thiện hoạt động kiểm soát rủi ro tín dụng trong cho vay doanh nghiệp tại...Hoàn thiện hoạt động kiểm soát rủi ro tín dụng trong cho vay doanh nghiệp tại...
Hoàn thiện hoạt động kiểm soát rủi ro tín dụng trong cho vay doanh nghiệp tại...
 
CÁC NHÂN TỐ ẢNH HƯỞNG ĐẾN HIỆU QUẢ HOẠT ĐỘNG CỦA NGÂN HÀNG THƯƠNG MẠI CỔ PHẦN...
CÁC NHÂN TỐ ẢNH HƯỞNG ĐẾN HIỆU QUẢ HOẠT ĐỘNG CỦA NGÂN HÀNG THƯƠNG MẠI CỔ PHẦN...CÁC NHÂN TỐ ẢNH HƯỞNG ĐẾN HIỆU QUẢ HOẠT ĐỘNG CỦA NGÂN HÀNG THƯƠNG MẠI CỔ PHẦN...
CÁC NHÂN TỐ ẢNH HƯỞNG ĐẾN HIỆU QUẢ HOẠT ĐỘNG CỦA NGÂN HÀNG THƯƠNG MẠI CỔ PHẦN...
 
VẤN ĐỀ 12 VI PHẠM HÀNH CHÍNH VÀ.pptx
VẤN ĐỀ 12 VI PHẠM HÀNH CHÍNH VÀ.pptxVẤN ĐỀ 12 VI PHẠM HÀNH CHÍNH VÀ.pptx
VẤN ĐỀ 12 VI PHẠM HÀNH CHÍNH VÀ.pptx
 
Hướng dẫn viết tiểu luận cuối khóa lớp bồi dưỡng chức danh biên tập viên hạng 3
Hướng dẫn viết tiểu luận cuối khóa lớp bồi dưỡng chức danh biên tập viên hạng 3Hướng dẫn viết tiểu luận cuối khóa lớp bồi dưỡng chức danh biên tập viên hạng 3
Hướng dẫn viết tiểu luận cuối khóa lớp bồi dưỡng chức danh biên tập viên hạng 3
 
Trích dẫn theo Harvard với Microsoft Word
Trích dẫn theo Harvard với Microsoft WordTrích dẫn theo Harvard với Microsoft Word
Trích dẫn theo Harvard với Microsoft Word
 
Hoàn thiện công tác kiểm soát chi NSNN qua Kho bạc Nhà nước huyện Tri Tôn – t...
Hoàn thiện công tác kiểm soát chi NSNN qua Kho bạc Nhà nước huyện Tri Tôn – t...Hoàn thiện công tác kiểm soát chi NSNN qua Kho bạc Nhà nước huyện Tri Tôn – t...
Hoàn thiện công tác kiểm soát chi NSNN qua Kho bạc Nhà nước huyện Tri Tôn – t...
 
Đào tạo, bồi dưỡng phát triển viên chức Đài Truyền hình Việt Nam
Đào tạo, bồi dưỡng phát triển viên chức Đài Truyền hình Việt NamĐào tạo, bồi dưỡng phát triển viên chức Đài Truyền hình Việt Nam
Đào tạo, bồi dưỡng phát triển viên chức Đài Truyền hình Việt Nam
 
Bài giảng chương 8: Phương trình vi phân cấp một và cấp hai
Bài giảng chương 8: Phương trình vi phân cấp một và cấp haiBài giảng chương 8: Phương trình vi phân cấp một và cấp hai
Bài giảng chương 8: Phương trình vi phân cấp một và cấp hai
 
35 ĐỀ LUYỆN THI ĐÁNH GIÁ NĂNG LỰC ĐẠI HỌC QUỐC GIA THÀNH PHỐ HỒ CHÍ MINH NĂM ...
35 ĐỀ LUYỆN THI ĐÁNH GIÁ NĂNG LỰC ĐẠI HỌC QUỐC GIA THÀNH PHỐ HỒ CHÍ MINH NĂM ...35 ĐỀ LUYỆN THI ĐÁNH GIÁ NĂNG LỰC ĐẠI HỌC QUỐC GIA THÀNH PHỐ HỒ CHÍ MINH NĂM ...
35 ĐỀ LUYỆN THI ĐÁNH GIÁ NĂNG LỰC ĐẠI HỌC QUỐC GIA THÀNH PHỐ HỒ CHÍ MINH NĂM ...
 
40 ĐỀ LUYỆN THI ĐÁNH GIÁ NĂNG LỰC ĐẠI HỌC QUỐC GIA HÀ NỘI NĂM 2024 (ĐỀ 1-20) ...
40 ĐỀ LUYỆN THI ĐÁNH GIÁ NĂNG LỰC ĐẠI HỌC QUỐC GIA HÀ NỘI NĂM 2024 (ĐỀ 1-20) ...40 ĐỀ LUYỆN THI ĐÁNH GIÁ NĂNG LỰC ĐẠI HỌC QUỐC GIA HÀ NỘI NĂM 2024 (ĐỀ 1-20) ...
40 ĐỀ LUYỆN THI ĐÁNH GIÁ NĂNG LỰC ĐẠI HỌC QUỐC GIA HÀ NỘI NĂM 2024 (ĐỀ 1-20) ...
 
PHIẾU KHẢO SÁT MỨC ĐỘ HÀI LÒNG VỀ CHẤT LƯỢNG DỊCH VỤ VẬN CHUYỂN HÀNG KHÁCH BẰ...
PHIẾU KHẢO SÁT MỨC ĐỘ HÀI LÒNG VỀ CHẤT LƯỢNG DỊCH VỤ VẬN CHUYỂN HÀNG KHÁCH BẰ...PHIẾU KHẢO SÁT MỨC ĐỘ HÀI LÒNG VỀ CHẤT LƯỢNG DỊCH VỤ VẬN CHUYỂN HÀNG KHÁCH BẰ...
PHIẾU KHẢO SÁT MỨC ĐỘ HÀI LÒNG VỀ CHẤT LƯỢNG DỊCH VỤ VẬN CHUYỂN HÀNG KHÁCH BẰ...
 
Nhân vật người mang lốt cóc trong truyện cổ tích thần kỳ Việt Nam
Nhân vật người mang lốt cóc trong truyện cổ tích thần kỳ Việt NamNhân vật người mang lốt cóc trong truyện cổ tích thần kỳ Việt Nam
Nhân vật người mang lốt cóc trong truyện cổ tích thần kỳ Việt Nam
 
Tăng trưởng tín dụng khách hàng cá nhân tại Ngân hàng Thương Mại Cổ Phần Công...
Tăng trưởng tín dụng khách hàng cá nhân tại Ngân hàng Thương Mại Cổ Phần Công...Tăng trưởng tín dụng khách hàng cá nhân tại Ngân hàng Thương Mại Cổ Phần Công...
Tăng trưởng tín dụng khách hàng cá nhân tại Ngân hàng Thương Mại Cổ Phần Công...
 

Phương Pháp Diện Tích Và Thể Tích Trong Hình Học Sơ Cấp.doc

  • 1. Tải tài liệu tại sividoc.com Viết đề tài giá sinh viên – ZALO:0973.287.149-TEAMLUANVAN.COM ĐẠI HỌC THÁI NGUYÊN TRƯỜNG ĐẠI HỌC KHOA HỌC PHẠM THỊ THANH THỦY PHƯƠNG PHÁP DIỆN TÍCH VÀ THỂ TÍCH TRONG HÌNH HỌC SƠ CẤP LUẬN VĂN THẠC SĨ TOÁN HỌC THÁI NGUYÊN - 2017
  • 2. Tải tài liệu tại sividoc.com Viết đề tài giá sinh viên – ZALO:0973.287.149-TEAMLUANVAN.COM ĐẠI HỌC THÁI NGUYÊN TRƯỜNG ĐẠI HỌC KHOA HỌC PHẠM THỊ THANH THỦY PHƯƠNG PHÁP DIỆN TÍCH VÀ THỂ TÍCH TRONG HÌNH HỌC SƠ CẤP LUẬN VĂN THẠC SĨ TOÁN HỌC Chuyên ngành:Phương pháp Toán sơ cấp Mã số: 60 46 01 13 NGƯỜI HƯỚNG DẪN KHOA HỌC PGS.TS. ĐÀM VĂN NHỈ THÁI NGUYÊN - 2017
  • 3. Tải tài liệu tại sividoc.com Viết đề tài giá sinh viên – ZALO:0973.287.149-TEAMLUANVAN.COM iii Mục lục Mở đầu iv Chương 1. Phương pháp diện tích 2 1.1 Định lý Pythagore . . . . . . . . . . . . . . . . . . . . . . . . . . 2 1.1.1 Tam giác vuông . . . . . . . . . . . . . . . . . . . . . . . 2 1.1.2 Hệ tọa độ Descarte vuông góc . . . . . . . . . . . . . . . 6 1.2 Định lý Stewart . . . . . . . . . . . . . . . . . . . . . . . . . . . 8 1.3 Phương pháp diện tích . . . . . . . . . . . . . . . . . . . . . . . 13 1.3.1 Phương pháp diện tích . . . . . . . . . . . . . . . . . . . 13 1.3.2 Định lý Ptolemy và mở rộng . . . . . . . . . . . . . . . . 15 1.3.3 Đường thẳng Simson, đường thẳng Steiner . . . . . . . . 22 1.4 Định lý Ceva và Định lý Menelaus . . . . . . . . . . . . . . . . . 24 1.5 Bất đẳng thức Erdos¨-Mordell cho đa giác . . . . . . . . . . . . . 29 Chương 2. Phương pháp thể tích 35 2.1 Phương pháp thể tích . . . . . . . . . . . . . . . . . . . . . . . . 35 2.1.1 Phương pháp thể tích . . . . . . . . . . . . . . . . . . . . 35 2.1.2 Thể tích qua định thức . . . . . . . . . . . . . . . . . . . 36 2.2 Quan hệ bán kính mặt cầu ngoại-nội tiếp . . . . . . . . . . . . . . 42 Chương 3. Vận dụng giải bài thi học sinh giỏi 46 Kết luận 56 Tài liệu tham khảo 57
  • 4. Tải tài liệu tại sividoc.com Viết đề tài giá sinh viên – ZALO:0973.287.149-TEAMLUANVAN.COM iv Mở đầu Hình học là một trong những phân nhánh Toán học xuất hiện sớm nhất của nhân loại. Nhiệm vụ của hình học có thể được mô tả ngắn gọn là trả lời cho các câu hỏi về hình dạng, kích thước, vị trí tương đối của các hình khối, và các tính chất của không gian. Các phương pháp giải toán trong hình học sơ cấp vốn vô cùng phong phú và đa dạng. Điều đó hoàn toàn dễ hiểu vì hình học là một môn học truyền thống trong nhà trường phổ thông và các trường đại học sư phạm. Dưới sự hướng dẫn của PGS.TS. Đàm Văn Nhỉ, tác giả luận văn này có mục đích trình bày về các phương pháp diện tích và thể tích trong hình học và những thảo luận về các bài thi học sinh giỏi, nhằm làm phong phú lý thuyết vừa trình bày và tạo cái nhìn đa chiều nhiều khía cạnh hơn cho giải toán hình học. Ngoài các phần Mở đầu, Kết luận, Tài liệu tham khảo, Chỉ mục, nội dung của luận văn được trình bày trong ba chương: • Chương 1. Phương pháp diện tích. Chương này sẽ trình bày các kết quả về phương pháp diện tích và ứng dụng vào giải toán hình học sơ cấp. Các chủ đề sẽ được thảo luận là các Định lý Pythagore, Định lý Stewart, Ceva, Menelaus và Bất đẳng thức Erdos¨-Mordell cho đa giác. • Chương 2. Phương pháp thể tích. Chương này dành để trình bày về phương pháp thể tích trong hình học, đặc biệt lưu ý đến thể tích qua định thức và một quan hệ liên quan đến bán kính của mặt cầu nội và ngoại tiếp. • Chương 3. Vận dụng giải bài thi học sinh giỏi. Chương này sẽ trình bày lời giải của một số bài thi học sinh giỏi điển hình liên quan đến các phương pháp diện tích và thể tích của Chương 1 và Chương 2.
  • 5. Tải tài liệu tại sividoc.com Viết đề tài giá sinh viên – ZALO:0973.287.149-TEAMLUANVAN.COM v Tác giả hi vọng rằng, bản luận văn này có thể làm tài liệu tham khảo hữu ích cho những ai quan tâm đến Hình học sơ cấp và ứng dụng. Nó sẽ có ích trong việc bồi dưỡng giáo viên, các học sinh khá giỏi, và những ai quan tâm đến toán sơ cấp và muốn mở rộng nhãn quan nói chung. Luận văn này đã được tác giả đầu tư nghiên cứu dưới sự hướng dẫn của PGS.TS. Đàm Văn Nhỉ nhưng do nhiều lí do, luận văn chắc chắn sẽ không tránh khỏi những thiếu sót. Tác giả mong muốn sẽ nhận được nhiều đóng góp của các quý Thầy Cô, các anh chị em đồng nghiệp để luận văn này hoàn chỉnh hơn. Thái Nguyên, ngày 20 tháng 5 năm 2017 Tác giả Phạm Thị Thanh Thủy
  • 6. Tải tài liệu tại sividoc.com Viết đề tài giá sinh viên – ZALO:0973.287.149-TEAMLUANVAN.COM 2 Chương 1 Phương pháp diện tích Hình học sơ cấp phát triển được là dựa trên nhiều kết quả của toán học cao cấp. Ví dụ đơn giản là để có thể đo độ dài một đoạn thẳng hay diện tích một hình vuông theo một đoạn thẳng được chọn làm đơn vị đo ta đã phải sử dụng kết quả về giới hạn, liên tục và tích phân xác định. Vấn đề lý giải quá trình hình thành kết quả nào đấy qua toán cao cấp là cần thiết và sẽ thường sử dụng tỷ số các đoạn thẳng hoặc diện tích trong chứng minh. Từ đó ta có thể phát hiện ra nhiều kết quả mới nữa. 1.1 Định lý Pythagore 1.1.1 Tam giác vuông Dựa vào tiên đề số đo độ dài một đoạn thẳng và nhiều kết quả trong lý thuyết về giới hạn ta sẽ sử dụng mệnh đề dưới đây để tính diện tích một hình vuông cạnh a. Mệnh đề 1.1.1. Diện tích hình vuông ABCD với độ dài cạnh AB = a (đơn vị dài) đúng bằng a 2 đơn vị diện tích. Chứng minh. Dựng hệ tọa độ Axy : A(0, 0), B(a, 0), C(a, a), D(0, a). Khi đó a a SABCD = a DX = ax = a 2 . 0 0 Như vậy, diện tích hình vuông ABCD cạnh a đúng bằng a2 đơn vị diện tích. Mệnh đề 1.1.2. Tam giác vuông ABC có độ dài cạnh a = BC, b = CA, c = AB và ∠BAC = 900 . Hạ đường cao AH ⊥ BC. Đặt h = AH và diện tích tam giác qua S. Khi đó ta có các đồng nhất thức
  • 7. Tải tài liệu tại sividoc.com Viết đề tài giá sinh viên – ZALO:0973.287.149-TEAMLUANVAN.COM 3 (1) a 2 = b 2 + c 2 [Pythagore]. (2) b2 = a.BH và c2 = a.CH . (3) a.h = b.c, h2 = BH .CH và 1 = 1 + 1 . h2 b2 c2 (4) 2S = a.h = b.c. Chứng minh. Dựng hình vuông ABCD với cạnh AB = a. Dựng vào bên trong hình vuông ABCD bốn tam giác vuông bằng nhau ABA1, BCB1, CDC1 và DAD1 bằng tam giác vuông ABA1. Khi đó ta có hình vuông A1B1C1D1 với A1B1 = |b − c|. Ta có SABCD bằng tổng diện tích bốn tam giác vuông ABA1, BCB1, CDC1, DAD1 và hình vuông A1B1C1D1. Vậy, ta có hệ thức a2 = 4. b 2 .c + (b − c)2 = b2 + c2 . Các kết quả còn lại là hiển nhiên. Hệ quả 1.1.1. Với biểu diễn b = a SIN B, c = a COS B trong tam giác vuông ABC ta có SIN 2 B + COS 2 B = 1. Chứng minh. Từ a 2 = b 2 + c 2 = a 2 (SIN 2 B + COS 2 B) theo Định lý 1.1.2 ta nhận được hệ thức SIN 2 B + COS 2 B = 1. Hệ quả 1.1.2. Trong mặt phẳng cho hai đường thẳng d và d′ . Lấy A, B thuộc d và C, D thuôc d′. Khi đó d ⊥ d′ nếu và chỉ nếu AC2 + BD2 = AD2 + BC2. Chứng minh. Kết quả được suy ra từ Định lý Pythagore.
  • 8. Tải tài liệu tại sividoc.com Viết đề tài giá sinh viên – ZALO:0973.287.149-TEAMLUANVAN.COM 4 Hệ quả 1.1.3 (Steiner). Cho tam giác ABC. Lấy M, N, P thuộc đường thẳng BC, CA, AB, tương ứng. Dựng đường thẳng mM ⊥ BC, nN ⊥ CA và pP ⊥ AB. Ba đường thẳng mM, nN và pP đồng quy tại một điểm khi và chỉ khi MC 2 + NA 2 + PB 2 = MB 2 + PA 2 + NC 2 . Chứng minh. Kết quả được suy ra trực tiếp từ Định lý Pythagore. Ví dụ 1.1.1. Cho tam giác ABC. Gọi M, N, P là trung điểm các cạnh BC,CA, AB, tương ứng. Dựng ra phía ngoài tam giác ABC các hình vuông BCA1A2 , CAB1B2, ABC1C2. Gọi A0 , B0,C0 là trung điểm A1A2, B1B2,C1C2, tương ứng và dựng các đường thẳng mM ⊥ B0C0, nN ⊥ C0A0 và pP ⊥ A0B0. Chứng minh rằng, ba đường thẳng mM, nN, pP đồng quy tại một điểm. Bài giải. Đặt α = ∠ABC0 và a = BC, b = CA, c = AB, S = SABC . Theo Định lý Côsin ta có MC 2 =4c 2 + b 2 + 2S và MB 2 =4b 2 + c 2 + 2S. Ta nhận được 0 4 0 4 MB0 2 − MC0 2 = 3 (b 2 − c 2 ). 4 Hoàn toàn tương tự, ta cũng có hai kết quả NC0 2 − NA0 2 = 3 (c 2 − a 2 ) và PA0 2 − PB0 2 = 3 (a 2 − b 2 ). 4 4 Ta có MB 2 0 − MC0 2 + NC0 2 − NA 2 0 + PA 2 0 − PB 2 0 = 0. Vậy, ba đường thẳng mM, nN, pP đồng quy tại một điểm theo hệ quả trên. Ví dụ 1.1.2. Cho tam giác ABC và các đường cao AA1, BB1,CC1. Dựng các đường thẳng AM ⊥ B1C1, BN ⊥ C1A1 và CP ⊥ A1B1. Chứng minh AM, BN,CP đồng quy.
  • 9. Tải tài liệu tại sividoc.com Viết đề tài giá sinh viên – ZALO:0973.287.149-TEAMLUANVAN.COM 5 Bài giải. Ta có MB 2 1 − MC1 2 = AB 2 1 − AC1 2 = (AB 2 − BB 2 1) − (AC 2 − CC1 2 ). Hoàn toàn tương tự ta cũng có NC1 2 − NA 2 1 và PA 2 1 − PB 2 1. Từ đó suy ra AM, BN,CP đồng quy theo ví dụ trên. Ví dụ 1.1.3. Tam giác ABC có ∠A = 900 và đường cao AH = h. Giả sử M là một điểm tùy ý trong tam giác và khoảng cách từ M đến BC,CA, AB là x, y, z. Xác định giá trị nhỏ nhất của tổng T = x 2 + y 2 + z 2 . Hạ MI ⊥ AH , I ∈ AH . Vì y 2 + z 2 = MA 2 nên ta có thể đánh giá như sau: T = MA 2 + x 2 AI 2 + x 2 = (h − x) 2 + x 2 = 2x 2 − 2hx + h 2 . Vậy T = 2 x − h 2 + h2 h2 2 2 2 và Tnn =h 2 khi x = h hay M là trung điểm đường cao AH . 2 2 Ví dụ 1.1.4 (Bulgarian MO and TST 2013). Cho tam giác nhọn ABC. Lấy các điểm M, N, P thuộc các cạnh BC,CA, AB, tương ứng, sao cho tam giác APN, BMP, CN M nhọn và ký hiệu Ha, Hb, Hc là trực tâm của chúng. Chứng minh rằng, nếu ba đường thẳng AHa, BHb,CHc đồng quy thì ba đường thẳng MHa, N Hb , PHc cũng đồng quy. Bài giải. Ký hiệu A1, B1,C1 là hình chiếu vuông góc của A, B,C lên N P, PM, MN, tương ứng. Vì đường thẳng AHa, BHb,CHc đồng quy tại một điểm nên ta có hệ thức (NC1 2 − MC1 2 ) + (MB 2 1 − PB 2 1) + (PA 2 1 − NA 2 1) = 0. Vậy 0 = CN 2 −CM 2 + BM 2 − BP 2 + AP 2 − AN 2 Bài giải.
  • 10. Tải tài liệu tại sividoc.com Viết đề tài giá sinh viên – ZALO:0973.287.149-TEAMLUANVAN.COM 6 = CN 2 − AN 2 + AP 2 − BP 2 + BM 2 −CM 2 Ta suy ra ba đường thẳng vuông góc hạ từ M, N, P xuống cạnh BC,CA, AB, tương ứng, đồng quy tại điểm K. Vì các tứ giác K MHcN và K MHbP là những hình bình hành nên tứ giác PHb HcN cũng là một hình bình hành. Khi đó hai đường chéo PHc, N Hb cắt nhau tại điểm T, trung điểm mỗi đường. Tương tự, MHa cũng nhận T làm trung điểm hay ba đường thẳng MHa, N Hb , PHc đồng quy tại điểm T. Ví dụ 1.1.5. Với hai số dương a, b xét ABCD độ dài cạnh AB = a + b. Lấy M thuộc cạnh AB với AM = a, BM = b. Giả sử N chạy trên cạnh BC và P thuộc cạnh AD sao cho N M ⊥ MP. Xác định giá trị nhỏ nhất của tam giác MN P. Bài giải. và suy ra của SMN P Đặt x = BN, y = AP. Từ điều kiện MN ⊥ MP ta suy ra MN 2 +MP 2 = N P 2 xy = ab. Khi đó SMN P = ax + by √ = ab. Vậy, giá trị nhỏ nhất abxy 2 bằng ab khi x = b, y = a. Ví dụ 1.1.6. Coi bốn xã như bốn đỉnh hình vuông ABCD với độ dài cạnh AB = 10 km. Chứng minh rằng, có thể xây mạng đường nối bốn xã với tổng độ dài nhỏ hơn 28 km. Bài giải. Gọi O là tâm hình vuông. Lấy I, J là trung điểm đoạn AO, BO, tương ứng. Mạng đường nối bốn xã là AI, IJ, JB, ID, JC. Tổng độ dài mạng đường này √ √ bằng d = 5 + 5 2 + 5 10 < 28, (đúng). 1.1.2 Hệ tọa độ Descarte vuông góc Trước khi trình bày hệ tọa độ Carte vuông góc chúng ta sẽ chứng minh lại Định lý Thales và nhắc lại khái niệm số đo đoạn thẳng. Giả sử ta chọn đoạn AB làm đơn vị đo. Theo tiên đề về số đo, ứng với mỗi đoạn thẳng MN luôn có một số thực x để MN = |x|.AB và MN = x.AB. Từ đó ta định nghĩa tỷ số giữa hai đoạn thẳng: Giả sử MN = |x|.AB, PQ = |y|.AB với y = 0. Tỷ số giữa hai đoạn được định nghĩa như sau: MN := |x| và MN := x . PQ |y| PQ y Định lí 1.1.1 (Thales’s Theorem). Với hai đường thẳng d, d′ và ba đường thẳng song song a, b, c cắt d, d ′ tại A, B,C và A ′ , B ′ ,C ′ , tương ứng, ta luôn có AB = A′B′ . BC B′ C′
  • 11. Tải tài liệu tại sividoc.com Viết đề tài giá sinh viên – ZALO:0973.287.149-TEAMLUANVAN.COM 7 Chứng minh. Dựng Aa1 d ′ và B1 AB1 = A ′ B ′ , B1C1 = B ′ C ′ . Như vậy = Aa1 × b,C1 = Aa1 × c. Khi đó ta nhận được AB = AB 1 = A′B′ . BCB1C1B ′ C ′ Ví dụ 1.1.7. Cho hình vuông ABCD. Lấy điểm K thuộc cạnh AB và N thuộc cạnh AD thỏa mãn AK.AN = 2BK.DN. Giả sử các đoạn thẳng CK,CN cắt BD tại điểm L, M. Chứng minh rằng, năm điểm A, K, L, M, N cùng nằm trên một đường tròn. Bài giải. Ta chỉ cần xét hình vuông với độ dài cạnh 1. Đặt a = AK, b = AN. Từ AK.AN = 2BK.DN ta suy ra ab = 2(1 − a)(1 − b) và ta nhận được 2 − 2a − 2b + ab = 0. Dựa vào Định lý Thales ta dễ dàng tính được BL = √ (1 − a) , √ . 2 BM = 2 2 − a 2 − b Vậy BL.BM = (2 2(1 − a) b) = 1 − a = BA.BK − a)(2 − và suy ra tứ giác AK LM nội tiếp trong một đường tròn. Tương tự, tứ giác ALMN cũng nội tiếp trong một đường tròn. Từ đó suy ra năm điểm A, K, L, M, N cùng nằm trên một đường tròn. Trong mặt phẳng (P), ta xét hai đưòng thẳng x ′ x, y ′ y với x ′ x ⊥ y ′ y và O = x ′ x × y ′ y. Với điểm gốc O và định hướng đường thẳng x ′ x và y ′ y để biến mỗi đường thành một trục. Khi đó ta đã có một Hệ tọa độ Carte vuông góc Oxy với gốc tọa độ O và trục hoành x ′ x, trục tung y ′ y. Mặt phẳng (P) với Hệ tọa độ Carte vuông góc Oxy được gọi đơn giản là hệ tọa độ Oxy. Mỗi điểm M thuộc trục x ′ x được gắn với đúng một số thực a ∈ R và viết OM = a. Mỗi điểm N thuộc trục y ′ y được gắn với đúng một số thực b ∈ R và viết ON = b. Đây là những số đo đại số. Giả sử điểm A tùy ý thuộc mặt phẳng tọa độ Oxy. Hạ AM ⊥ x′ x, M ∈ x′ x và hạ AN ⊥ y ′ y, N ∈ y ′ y. Dễ dàng thấy rằng, mỗi điểm A thuộc mặt phẳng (P) tương ứng một-một với một cặp số thực (a, b), trong đó OM = a, ON = b. Cặp số thực (a, b) được gọi là tọa độ điểm A và viết A(a, b).
  • 12. Tải tài liệu tại sividoc.com Viết đề tài giá sinh viên – ZALO:0973.287.149-TEAMLUANVAN.COM 8 Khoảng cách: Giả sử điểm A(x1, y1) và điểm B(x2, y2). Theo Định lý Pythagore, khoảng cách giữa hai điểm A và B hay độ dài đoạn AB bằng AB = (x2 − x1) 2 + (y2 − y1) 2 . Chia đoạn theo tỷ số: Giả sử điểm A(x1, y1) và điểm B(x2, y2). Ta xác định điểm I thuộc đoạn AB sao cho IA = γ2 . Ta chỉ cần xét trường hợp AB không song song BI γ1 với trục tọa độ nào. Hạ AA1, IM, BB1 ⊥ x ′ x. Gọi I(x, y). Khi đó γ2 γ1 Giải ra được Tương tự, ta cũng có = IA = MA 1 = x 1 − x . − x2BIB1Mx x =γ1 x 1 + γ2 x 2 . γ1+γ2 y =γ1 y 1 + γ2 y 2 . γ1+γ2 Như vậy, tọa độ điểm chia I γ1 x1 + γ2x2 ,γ1y1 + γ2 y2 . γ1+γ2 γ1+γ2 1.2 Định lý Stewart Tiếp theo, ta chứng minh Định lý Stewart sau đây. Định lí 1.2.1 (Stewart). Với ba điểm M, N, P thẳng hàng và mọi điểm I có hệ thức T = IM2 .NP + IN2 .PM + IP2 .MN = −MN.NP.PM. Chứng minh. Dựng hệ tọa độ Oxy sao cho M(0, 0), N(b, 0), P(c, 0). Giả sử I(x, y). Khi đó T = (x 2 + y 2 )(c − b) + [(x − b) 2 + y 2 ](−c) + [(x − c) 2 + y 2 ]b. Dễ dàng suy ra hệ thức T = bc(c − b) = −MN.N P.PM. Hệ quả 1.2.1. Cho tam giác ABC với BC = a,CA = b, AB = c. Gọi M là trung điểm cạnh BC và đặt ma = AM. Gọi N là chân đường phân giác trong AN của góc ∠A và đặt ℓ = AN; Gọi L là chân đường phân giác ngoài AL của góc ∠A khi b = c và đặt la = AL. Khi đó ta có các kết quả dưới đây:
  • 13. Tải tài liệu tại sividoc.com Viết đề tài giá sinh viên – ZALO:0973.287.149-TEAMLUANVAN.COM 9 (1) m2 = 2b2 + 2c2 − a2 . a 4 (2)ℓ2 = bc[(b + c)2 − a2] . a (b + c) 2 (3)l2 = bc[a2 − (c − b)2] . a (c − b) 2 Chứng minh. (1) Với trung điểm M của cạnh BC ta luôn có m2 = 2b 2 + 2c 2 − a 2 a 4 theo Định lý 1.2.1, ở đó AM = ma. (2) Khi AN = ℓa là phân giác trong của góc ∠A thì BN = ac và CN = ab . b + c b + c Theo Định lý 1.2.1 có ℓ2a = bc[(b + c)2 − a2] . (3) Khi b = c và AL = la là phân giác ngoài của góc ∠A thì BL = ac và CL = ab . c − b c − b Theo Định lý 1.2.1 có l2 = bc[a2 − (c − b)2 ] . a (c − b) 2 Hệ quả 1.2.2 (Steiner-Lehmus). Cho tam giác ABC với BC = a, CA = b, AB = c. Giả sử ℓa, ℓb, ℓc là độ dài các đường phân giác trong của ΔABC. Nếu ℓa = ℓb thì tam giác ABC cân. Chứng minh. Ta biết ℓ 2 = bc[(b + c) 2 − a 2 ] và ℓ 2 =ca[(a + c) 2 − b 2 ] . a (b + c)2 b (a + c)2 Nếu ℓa = ℓb thì bc[(b + c) 2 − a 2 ] ca[(a + c) 2 − b 2 ] = . (b + c)2 (a + c)2 Ta có a = b hay tam giác ABC cân với AB = AC. (b + c)2
  • 14. Tải tài liệu tại sividoc.com Viết đề tài giá sinh viên – ZALO:0973.287.149-TEAMLUANVAN.COM 10 Ví dụ 1.2.1. Cho tam giác ABC với BC = a,CA = b, AB = c. Giả sử ha, hb, hc, ℓa, ℓb, ℓc và ma, mb, mc là độ dài các đường cao, các đường phân giác trong và các đường trung tuyến của ΔABC. Ta có các bất đẳng thức (1) ha ℓa ma. Dấu = xảy ra khi b = c. ℓa ℓb ℓc √ + √ + (2) + + 2 bc ca ma mb mc b + cc + a a = b = c. √ ab .Dấu = xảy ra khi và chỉ khi a + b Bài giải. (1) Hiển nhiên ha ℓa. Vì ℓa 2 = 4bc (b + c) 2 − a 2 4bc 1 ma 2 (b + c) 2 2(b 2 + c 2 ) − a 2 (b + c) 2 theo Chú ý 1.2.1 nên ℓa ma. Hiển nhiên, dấu bằng xảy ra khi và chỉ khi b = c. (2) Bởi có √ √ √ ℓa ℓ b 2 , ℓc 2 ab 2 bc , ca ma b + cmb c + amc a + b nên ℓb ℓc √ √ √ ℓa + + ab . + + 2 bc ca ma mb mc b + cc + aa + b Dấu = xảy ra khi và chỉ khi a = b = c. Ví dụ 1.2.2. Cho ΔABC với BC = a,CA = b, AB = c, 2 p = a + b + c, bán kính các đường tròn bàng tiếp là ra, rb, rc. Giả sử ℓa, ℓb, ℓc và ma, mb, mc là độ dài các đường phân giác trong và các đường trung tuyến của ΔABC. Khi đó ta có các kết quả sau: (1) m 2 + m 2 + m 2 = 3 a2 + b2 + c2 , và a b c 4 b + c 2 ℓa 2 c + a 2 ℓb 2 a + b 2 ℓc 2 2 + + = 4 p. bc ca ab (2) ℓ 2 + ℓ 2 + ℓ 2 p 2 . a b c (3) r2 + r2 + r2 m2 + m2 + m2 +(a − b)2 + (b − c)2 + (c − a)2 . a b c a b c 4 (4) r2 + r 2 + r 2 + ℓ 2 +ℓ 2 +ℓ 2 m 2 + m 2 + m 2 + p 2 . a b c a b c a b c Chú ý rằng các bất đẳng thức này sẽ trở thành đẳng thức khi và chỉ khi a = b = c.
  • 15. Tải tài liệu tại sividoc.com Viết đề tài giá sinh viên – ZALO:0973.287.149-TEAMLUANVAN.COM 11 Bài giải. (1) suy ra từ Hệ quả 1.2.1 và (2) được suy ra từ (1). (3) Đặt x = p − a, y = p − b, z = p − c với p = a + b + c . Biến đổi được 2 ra 2 + rb 2 + rc 2 = x + y + z yz + zx + xy y x z = xy + yz + zx + x 2 y +z + y 2 z + x + z 2 x +y z y x z y x xy + yz + zx + 2x 2 + 2y 2 + 2z 2 = 3 a 2 + b 2 + c 2 + a − b 2 + b − c 2 + c − a 2 . 4 Do vậy có được r 2 + r 2 + r 2 m 2 + m 2 + m 2 +(a − b) 2 + (b − c) 2 + (c − a) 2 . a b c a b c 4 (4) Ta có ℓ 2 = bc[(b + c)2 − a2 ] = bc a 2 bc bc a2 . − b + c 2 − 4 a (b + c) 2 Như vậy ℓa 2 + ℓb 2 + ℓc 2 bc + ca + ab − a2 + b2 + c2 . 4 Kết hợp với bất đẳng thức (2) được ra 2 + rb 2 + rc 2 + ℓa 2 + ℓb 2 + ℓc 2 ma 2 + mb 2 + mc 2 + p2 . Ví dụ 1.2.3. Cho ΔABC với diện tích S và BC = a,CA = b, AB = c. Giả sử ha, hb, hc và ma, mb, mc là độ dài các đường cao và các đường trung tuyến của ΔABC. Khi đó ta có bất đẳng thức (m2 + m 2 + m 2 )(h 2 + h 2 + h 2 ) 27S 2 . a b ca b c Bài giải. Vì ma 2 + mb 2 + mc 2 = 3 (a 2 + b 2 + c 2 ) 4 nên (ma 2 + mb 2 + mc 2 )(ha 2 + hb 2 + hc 2 ) 3 3 3 (abc) 2 3 3 (hahbhc) 2 = 27S 2 . 4
  • 16. Tải tài liệu tại sividoc.com Viết đề tài giá sinh viên – ZALO:0973.287.149-TEAMLUANVAN.COM 12 Ví dụ 1.2.4 (Garfunkel). Tam giác ABC có độ dài ba đường trung tuyến ma, mb, mc. Gọi G là trọng tâm tam giác ABC và các đường thẳng AG, BG,CG cắt đường tròn ngoại tiếp tam giác tại A′, B′,C′, tương ứng. Khi đó ta có bất đẳng thức A′ A +B′ B +C′ C 4. m a m b m c Bài giải. Ký hiệu a, b, c cho độ dài ba cạnh ΔABC. Khi đó ta dễ dàng có các hệ thức GA = 2b 2 + 2c 2 − a 2 GA′ a2 + b2 + c2 GB = 2c2 + 2a2 − b2 GB′ a2 + b2 + c2 GC = 2a2 + 2b2 − c2 . GC ′ a2 + b2 + c2 Cộng ba hệ thức, ta sẽ nhận được hệ thức GA + GB + GC = 3 GA ′ GB ′ GC ′ và suy ra bất đẳng thức GB′ GC′ GA′ + + 3. GA GB GC Vì AA′ = 1 + GA′ GA GA nên 3.AA′ = 1+ GA ′ . 2 maGA Tương tự, ta cũng có 3 .BB ′ = 1 +GB ′ và 3. CC ′ = 1 +GC ′ . 2 mb GB 2 m c GC Cộng ba hệ thức này ta nhận được bất đẳng thức A ′ A + B ′ B +C ′ C 4. m a mb m c Ví dụ 1.2.5. Tam giác ABC có độ dài ba đường trung tuyến ma, mb, mc và bán kính đường tròn ngoại tiếp R. Khi đó ta có bất đẳng thức 1 + 1 + 1 2 . m a m b m c R
  • 17. Tải tài liệu tại sividoc.com Viết đề tài giá sinh viên – ZALO:0973.287.149-TEAMLUANVAN.COM 13 Bài giải. Vì AA ′ , BB ′ ,CC ′ 2R nên 2R +2R +2R A′ A +B′ B + C′ C 4. ma m b m c ma m b m c Từ đó suy ra bất đẳng thức 1 + 1 + 1 2 . m a m b m c R Ta có điều phải chứng minh. 1.3 Phương pháp diện tích 1.3.1 Phương pháp diện tích Phương pháp diện tích là phương pháp giải bài toán hình trong mặt phẳng qua diện tích của một miền phẳng nào đó. Để giải bài toán đã cho ta chọn một miền phẳng (D) với diện tích S. Sau đó đem chia miền (D) ra thành nhiều miền nhỏ một cách thích hợp và tính diện tích S1, . . . , Sr của các miền nhỏ ấy. Từ S = S1 + · · · + Sr suy ra lời giải bài toán. Mệnh đề 1.3.1. Cho tam giác ABC với độ dài ba cạnh a, b, c với độ dài các đường cao ha, hb, hc; độ dài các bán kính đường tròn ngoại, nội và bàng tiếp là R, r, r1, r2, r3. Khi đó diện tích S của tam giác ABC được xác định qua (1) S = ah 2 a = bh 2 b = ch 2 c . (2) S = bcSIN A = ca SIN B = ab SIN C . 222 (3) S = abc . 4R (4) S = 2R 2 SIN A SIN B SIN C. (5) S = (a + b + c)r = pr. 2 (6) S = (p − a)r1 = (p − b)r2 = (p − c)r3. (7) S =p(p − a)(p − b)(p − c). Mệnh đề 1.3.2. Hình tròn bán kính R có diện tích S = π .R 2 đơn vị diện tích.
  • 18. Tải tài liệu tại sividoc.com Viết đề tài giá sinh viên – ZALO:0973.287.149-TEAMLUANVAN.COM 14 Chứng minh. Giả sử đường tròn x2 + y2 = R2 trong mặt phẳng Oxy. Ta có diện tích R √ 2 π /2 2 2 (đơn vị diện tích). 2 2 t DT = π .R S = 4 0 R − x DX=4R 0 SIN Mệnh đề 1.3.3. Cho tứ giác lồi ABCD với độ dài bốn cạnh AB = a, BC = b, CD = c và DA = d. Khi đó ta có công thức tính diện tích S của tứ giác ABCD : 4(ab − cd)2 − (a2 + b2 − c2 − d2 )2 + 16abcd SIN 2 ∠B + ∠D 2 S = . 4 Khi tứ giác ABCD ngoại tiếp đường tròn thì S = √ abcd SIN ∠B + ∠D . 2 Chứng minh. Ta có 2S = 2SABC + 2SADC = ab SIN ∠B + cd SIN ∠D. Từ a2 + b2 − 2ab COS B = AC2 = c2 + d2 − 2cd COS D ta suy ra 2ab COS B − 2cd COS D = a2 + b2 − c2 − d2 . Biến đổi biểu thức sau: (a 2 + b 2 − c 2 − d 2 ) 2 + 16S 2 = 4(ab COS B − cd COS D) 2 +4(ab SIN ∠B + cd SIN ∠D)2 = 4(a2 b2 + c2 d2 ) − 8abcd COS(∠B + ∠D) = 4(ab − cd) 2 + 16abcd SIN ∠B+∠D 2 và 4(ab − cd) 2 − (a 2 + b 2 − c 2 − d 2 ) 2 + 16abcd SIN 2 ∠B+∠D S = 2 . 4 Khi tứ giác ABCD ngoại tiếp đường tròn thì 4(ab − cd) 2 − (a 2 + b 2 − c 2 − d 2 ) 2 = 0 và ta nhận được S = √ abcd SIN ∠B + ∠D . 2 Mệnh đề 1.3.4 (Bretschneider). Cho tứ giác lồi ABCD với độ dài bốn cạnh AB = a, BC = b, CD = c, DA = d và AC = x, BD = y. Khi đó ta có công thức tính diện tích S của tứ giác ABCD : S = 4x2 y2 − (a2 − b2 + c2 − d2 )2 . 4
  • 19. Tải tài liệu tại sividoc.com Viết đề tài giá sinh viên – ZALO:0973.287.149-TEAMLUANVAN.COM 15 Chứng minh. Ta có 2S = xy SIN α , ở đó α là góc giữa AC và BD. Dễ thấy 2xy COS α = |a2 − b2 + c2 − d2 |. Vậy S = 4x 2 y 2 − (a 2 − b 2 + c 2 − d 2 ) 2 . 4 Mệnh đề 1.3.5. Cho tứ giác lồi ABCD nội tiếp trong một đường tròn với độ dài bốn cạnh AB = a, BC = b, CD = c và DA = d. Khi đó, với p = a + b + c + d ta có 2 công thức tính diện tích tứ giác: S= (p − a)(p − b)(p − c)(p − d). Chứng minh. Vì 2S = (ab + cd) SIN B và COS B = a 2 + b 2 − c 2 − d 2 2(ab + cd) nên ta có biến đổi sau đây để tính SIN B qua độ dài các cạnh và chu vi: 2 2 2 2 2 SIN 2 B= 1 COS 2 B= 1 a + b − c − d − − 2(ab + cd) = [(a + b) 2 − (c − d) 2 ][(c + d) 2 − (a − b) 2 ] 4(ab + cd) 2 4(p − a)(p − b)(p − c)(p − d) = và nhận được (ab + cd) 2 (ab + cd) SIN B = 2 (p − a)(p − b)(p − c)(p − d). Từ đây ta có được công thức tính diện tích S = (p − a)(p − b)(p − c)(p − d). 1.3.2 Định lý Ptolemy và mở rộng Định lí 1.3.1. [Ptolemy] Cho tứ giác lồi ABCD nội tiếp trong một đường tròn. Khi đó ta luôn có AC.DB = AB.DC + BC.DA hay AB + BC = AC . DA.DC DB.DC DA.DC
  • 20. Tải tài liệu tại sividoc.com Viết đề tài giá sinh viên – ZALO:0973.287.149-TEAMLUANVAN.COM 16 Chứng minh. Đặt a = AB, b = BC, c = CD, d = DA và x = AC, y = BD. Tứ giác lồi ABCD nội tiếp trong một đường tròn (T ). Kẻ BE AC với E thuộc (T ). Khi đó 2SABCD = 2SAECD = (ac + bd) SIN ∠E AD. Vì 2SABCD = xy SIN ∠E AD, ở đó α là góc giữa AC và BD. Vậy ac + bd = xy. Chia hai vế cho DA.DB.DC ta nhận được hệ thức dạng phân thức AB + BC = AC . DA.DC DB.DC DA.DC Nhận xét 1.3.1. (i) Nếu tứ giác ABCD lõm và nội tiếp trong một đường tròn với độ dài bốn cạnh AB = a, BC = b, CD = c, DA = d và độ dài hai đường chéo AC = x, BD = y thì luôn có ac − bd = xy khi tứ giác ACBD lồi hoặc bd − ac = xy khi tứ giác ABDC lồi. Do vậy, với việc chọn dấu ± một cách thích hợp ta luôn có ac ± bd ± xy = 0. (ii) Biểu diễn Đồng nhất thức Ptolemy dạng phân thức là dựa vào phương pháp phân tích phân thức thành tổng đại số các phân thức đơn giản. Định lí 1.3.2. Với số nguyên n 3, đa giác lồi A1 . . . AnM nội tiếp trong một đường tròn có đồng nhất thức A1A2 + A 2 A 3 +···+ A n−1 A n = A n A 1 . MA1.MA2 MA2.MA3 MAn−1.MAn MAn.MA1 Chứng minh. Quy nạp theo n. Với n = 3, kết luận đúng theo Định lý 1.3.1. Giả sử kết luận đúng cho n. Khi đó có đồng nhất thức A 1 A 2 + A2A3 +···+ A n−1 A n = AnA1 . MA1.MA2 MA2.MA3 MAn−1.MAn MAn.MA1 Với đa giác lồi A1 . . . AnAn+1M nội tiếp trong một đường tròn, có A1A2 + A 2 A 3 +···+ A n−1 A n = A n A 1 MA1.MA2 MA2.MA3 MAn−1.MAn MAn.MA1
  • 21. Tải tài liệu tại sividoc.com Viết đề tài giá sinh viên – ZALO:0973.287.149-TEAMLUANVAN.COM 17 cho đa giác lồi A1 . . . AnM và cho tứ giác A1AnAn+1M ta có hệ thức A 1 A n + A n A n+1 = A 1 A n+1 . MA1.MAn MAn.MAn+1 MA1.MAn+1 Cộng hai hệ thức này lại, ta nhận được Đồng nhất thức Ptolemy tổng quát cho đa giác lồi A1A2 + A2A3 +···+ A n A n+1 = A n+1 A 1 . MA1.MA2 MA2.MA3 MAn.MAn+1 MAn+1.MA1 Ví dụ 1.3.1. Tam giác ABC có bán kính đường tròn ngoại tiếp và nội tiếp R, r. Giả sử ba phân giác trong các góc của tam giác cắt đường tròn ngoại tiếp ở A1, B1,C1. Chứng minh rằng AA1.BB1.CC1 16R2 r. Bài giải. Theo Bổ đề 1.3.1 ta có AA1 = A1B(b + c) =b + c.2R SIN A 2 a a vì A1B = A1C và bằng 2R SIN A . Ta cũng có hệ thức tương tự cho BB1,CC1. Vậy 2 AA1.BB1.CC1 = (b + c)(c + a)(a + b) 8R 3 SIN A .SIN B .SIN C 2 abc 2 2 8abc .2R 2 .4R SIN A r. SIN B .SIN C = 16R 2 r. abc 2 2 2 Tóm lại, ta nhận được AA1.BB1.CC1 16R 2 r. Ví dụ 1.3.2. Tam giác ABC có a = BC, b = CA, c = AB và bán kính đường tròn ngoại tiếp R. Giả sử phân giác trong các góc của tam giác cắt đường tròn ngoại tiếp ở A1, B1,C1, tương ứng. Chứng minh rằng (1) AA1 + BB1 +CC1 > a + b + c. (2) S A1B1C1 S ABC. (3) Gọi độ dài ba phân giác trong là ℓa, ℓb, ℓc. Khi đó ℓa + ℓb + ℓc 3 [APMO1997]. AA1 SIN 2 A BB1 SIN 2 B 2 CC1 SIN C Bài giải. (1) Theo Định lý cosin ta có 2c .AA 1 COS 2 A1B2 = c 2 + AA1 2 − A 2 2 2 A A1C = b + AA1 − 2b.AA1 COS . 2
  • 22. Tải tài liệu tại sividoc.com Viết đề tài giá sinh viên – ZALO:0973.287.149-TEAMLUANVAN.COM 18 Vì A1B = A1C nên c 2 − b 2 = 2(c − b).AA1 COS A . Nếu b= c thì 2 AA1 = 2Rvà b + c < 4R. Nếu b = c thì b + c = 2AA1 COS A . Tương tự, ta cũng có 2 c + a = 2BB1 COS B và a + b = 2CC1 COS C . 2 2 Do đó a + b + c < AA1 + BB1 +CC1. (2) Ta có S A1B1C1 A + B B +C C + A S ABC = SIN SIN SIN SIN A SIN B SINC = 2R 2 2 2 2 2R 2 và suy ra bất đẳng thức SA1 B1C1 S ABC. (3) Theo Định lý sin ta có ℓa = c SIN B và AA1 =c SIN B+A/2 . SIN C + A/2 SIN C Vậy ℓa = SIN B SIN C SIN B SIN C. AA1 SIN 2 A SIN 2 A SIN B+A/2 SIN C + A/2 SIN 2 A Đánh giá còn lại tương tự. Từ đó có ℓa + ℓ b + ℓc SIN B SIN C +SIN C SIN A +SIN A SIN B AA1 SIN 2 A BB1 SIN 2 BCC1 SIN 2 C SIN 2 A SIN 2 B SIN 2 C 3 theo Bất đẳng thức AM-GM. Ví dụ 1.3.3. Ký hiệu n(r) là số điểm nguyên thuộc đường tròn bán kính r > 1. √ Chứng minh n(r) < 2π 3 r 2 . Bài giải. Giả sử đường tròn (ℓ) bán kính r > 1 chứa n điểm với tọa độ nguyên. Ta phải chỉ ra n(r) < 2π √ r2 . Vì r > 1 và 2π > 6 nên 2π √ r2 > 6. Vậy, ta có thể giả 3 3 thiết n 7 bởi vì n < 7 bất đẳng thức luôn đúng. Ký hiệu điểm nguyên thuộc đường tròn (ℓ) theo thứ tự P1, P2, . . . , Pn, (theo chiều ⌢ ⌢ ⌢ quay của kim đồng hồ). Khi đó các cung P1P3, P2P4, Pn−1P1 và PnP2 phủ đường tròn hai lần. Do vậy, tổng các số đo cung của các cung này bằng 4π . Từ đó suy
  • 23. Tải tài liệu tại sividoc.com Viết đề tài giá sinh viên – ZALO:0973.287.149-TEAMLUANVAN.COM 19 ra tồn tại ít nhất một cung không vượt quá4π ⌢ , chẳn hạn cung P1P3. Xét tam giác 4π n P1P2P3 nội tiếp trong cung với số đo . Vì n 7 nên cung này có thể coi như nhỏ hơn một phần tư đường tròn. Diện n tích tam giác P1P2P3 lớn nhất khi P1, P3 là hai điểm mút của cung và P2 là điểm chính giữa của cung. Khi đó 2r SIN π.2r SIN π.2r SIN 2π 2r π.2r π.2r 2π abc S P1P2 P3 = = n n n n n n . 4r 4r 4r Như vậy S 4r 2 .π 3 . Giả sử tọa độ nguyên các đỉnh P (x , y ), i = 1, 2, 3. Khi P1 P2P3 n 3 ii i đó 4r 2 .π 3 1 1 1 = 1 1 . S P1 P2P3 x1 x2 x3 n3 2 2 Ta nhận được bất đẳng thức n(r) √ r2 y1 y2 y3 2π . Vì π là số siêu việt và n là số nguyên dương nên n(r) < 2π √ r2. 3 3 √ π .r2 [Iran MO 1999]. Nhận xét 1.3.2. Kết quả yếu hơn n(r) < 6 3 Ví dụ 1.3.4. Cho tam giác ABC với trọng tâm G. Với điểm S bất kỳ, gọi I là trung điểm SG và đường thẳng tùy ý d qua I cắt SA, SB, SC tại A1, B1,C1, tương ứng, khác S. Chứng minh rằng SA + SB + SC = 6. SA1 SB1 SC1 Bài giải. Gọi M là trung điểm BC và kéo dài AM về phía M lấy điểm N sao cho M là trung điểm GN. Gọi giao điểm SM, SN với d là M1, N1. Khi đó SB + SC = 2 SM , SN +SG = 2 SM , SN + SA = 2SG = 4. SB1SC1SM1 SN1 SI SM1 SN1 SA1 SI Vậy SA + SB + SC = 6. SA1 SB1SC1 Mệnh đề 1.3.6. Với ΔABC và M ∈ BC ta có SIN ∠ BAM + SIN ∠ CAM = SIN ∠ BAC . AC AB AM Chứng minh. Do SABM + SACM = SABC nên SIN ∠ BAM + SIN ∠ CAM = SIN ∠ BAC . AC AB AM
  • 24. Tải tài liệu tại sividoc.com Viết đề tài giá sinh viên – ZALO:0973.287.149-TEAMLUANVAN.COM 20 Ví dụ 1.3.5. Với tứ giác lồi ABCD, K = AD × BC, L = AB × CD, F = BD × K L, G = AC × K L, ta luôn luôn có hệ thức 2 = 1 + 1 . K L K F K G Bài giải. Đặt α = ∠CK A và β = ∠CK L Theo Mệnh đề 1.3.6 ta luôn có hệ thức sau: SIN(α + β ) = SIN α +SIN β ,SIN(α + β ) =SIN α + SIN β K B K L K A KC KLKD SIN(α + β ) = SIN α +SIN β ,SIN(α + β ) =SIN α + SIN β K B K F K D KC KGKA và có được kết quả SINα +SINβ +SINα +SINβ =SINα +SINβ +SINα +SINβ KL KA KL KD KF KD KG KA hay hệ thức 2 = 1 + 1 . K L KFKG Ví dụ 1.3.6. Với tam giác ABC, độ dài cạnh a, b, c và độ dài đường phân giác trong ℓa, ℓb, ℓc ta luôn có hệ thức COS ∠A COS ∠B COS ∠ C 1 1 1 2 + 2 + 2 = + + . ℓa ℓb ℓc abc
  • 25. Tải tài liệu tại sividoc.com Viết đề tài giá sinh viên – ZALO:0973.287.149-TEAMLUANVAN.COM 21 Bài giải. Theo Mệnh đề 1.3.6 ta có 2 COS ∠A 2 COS ∠B 2 COS ∠C 1 1 1 1 1 1 2 = + , 2 = + , 2 = + . ℓ b ℓa bc ca ℓc ab Từ đó ta có hệ thức COS ∠A COS ∠ B COS ∠ C 1 1 1 2 + 2 + 2 = + + . ℓa ℓb ℓc abc Ta có điều phải chứng minh. Ví dụ 1.3.7. Cho tứ giác lồi ABCD. Giả sử M, N ∈ AB và P, Q ∈ CD sao cho AM = N B và CP = QD. Chứng minh rằng, nếu SAMQD = SBN PC thì AB CD. Bài giải. Giả sử AB và CD cắt nhau tại O. Dựng hệ tọa độ Oxy sao cho A(u, 0), M(u + a, 0), N(v, 0), B(v + a, 0), D(z, kz), Q(z + b, k(z + b)), P(t , kt ), C(t + b, k(t + b)) với 0 < u < v, 0 < z < t , k = 0, a, b > 0, vì AM = N B và DQ = PC. Giả thiết S AMQD = S BN PC . Tính 2S AMQD = u + a − u 0 − 0 + z+ b − u − a k(z + b) − 0 z − u kv − 0 z − u − a kz − 0 = kaz + kbu + kab 2S BNPC = t + a − v 0 − 0 + t+ b − v − a k(t + b) − 0 t − v kt − 0 t − v − a kt − 0 = kat + kbv + kab và nhận được kaz + kbu + kab = kat + kbv + kab hay a(t − z) = −b(v − u), vô lý. Do vậy, điều giả sử là sai và có AB CD. Ví dụ 1.3.8. Cho tứ giác lồi ABCD. Giả sử M, N ∈ AB và P, Q ∈ CD sao cho AM = MN = N B và CP = PQ = QD. Gọi O = AC ×BD. Chứng minh rằng SOMP = SON Q. Bài giải. Dựng hệ tọa độ Oxy sao cho A(a, 0), C(c, 0) và phương trình AB : y = k(x − a), CD : y = h(x − c).
  • 26. Tải tài liệu tại sividoc.com Viết đề tài giá sinh viên – ZALO:0973.287.149-TEAMLUANVAN.COM 22 Giả sử B(a + 3u, k3u), D(c + 3v, h3v). Vì B, O, D thẳng hàng nên ku(c + 3v) = hv(a + 3u) hay kcu − hav = 3(h − k)uv. Tính tọa độ M(a + u, ku), P(c + v, hv), N(a + 2u, k2u), Q(c + 2v, h2v) và tính diện tích tam giác định hướng 2S OMP = c + v − 0 hv − 0 =kcu − hav + (k − h)uv a + u − 0 ku − 0 2SON Q = c + 2v − 0 h2v − 0 2kcu − 2hav +4(k − h uv = ) . a + 2u − 0 k2u − 0 Vì kcu − hav = 3(h − k)uv nên SOMP = huv − kuv = SON Q. 1.3.3 Đường thẳng Simson, đường thẳng Steiner Mệnh đề 1.3.7 (Simson). Giả sử điểm M thuộc đường tròn ngoại tiếp tam giác ABC, khác phía với A đối với BC. Hạ MA1, MB1, MC1 vuông góc với BC,CA, AB, tương ứng. Khi đó, ba điểm B1, A1,C1 thẳng hàng. Chứng minh. Từ B1C1 = MA SIN A, A1B1 = MC SIN C, A1C1 = MB SIN B và aMA = bMB + cMC theo Đồng nhất thức Ptolemy suy ra A1B1 + A1C1 = MCSINC +MBSINB = cMC + bMB =aMA . 2R 2R Từ đây suy ra A1B1 + A1C1 = B1C1 hay ba điểm B1, A1,C1 thẳng hàng.
  • 27. Tải tài liệu tại sividoc.com Viết đề tài giá sinh viên – ZALO:0973.287.149-TEAMLUANVAN.COM 23 Mệnh đề 1.3.8 (Feuerbach). Với tứ giác lồi ABCD nội tiếp trong một đường tròn luôn có DA 2 SDBC + DC 2 SDAB = DB 2 SDCA. Chứng minh. Vì tứ giác ABCD nội tiếp trong đường tròn nên DA+DC =DB bc ab ca theo Bổ đề 1.3.1. Vậy aDA2 + cDC2 = bDB2 . DA DC DB Từ đây suy ra DA2 .DB.DC.a + DC2 .DA.DB.c = DB2 .DC.DA.b và dễ dàng có đồng nhất thức DA 2 SDBC + DC 2 SDAB = DB 2 SDCA. Mệnh đề 1.3.9. Giả sử điểm M thuộc đường tròn ngoại tiếp tam giác ABC, khác phía với A đối với BC. Gọi A ′ , B ′ ,C ′ là điểm đối xứng với điểm M qua BC,CA, AB, tương ứng, và A0 = MA ′ × BC, B0 = MB ′ ×CA và C0 = MC ′ × AB. Khi đó ta có (1) (Steiner)Ba điểm B ′ , A ′ ,C ′ thẳng hàng. (2) Đường thẳng (B′ , A′ ,C′ ) đi qua trực tâm H của ΔABC, còn đường thẳng B0C0 đi qua trung điểm đoạn MH . Chứng minh. (1) Ta có MA0 ⊥ BC, MB0 ⊥ CA và MC0 ⊥ AB. Theo Mệnh đề 1.3.7, A0, B0, C0 thẳng hàng. Vì A ′ C ′ A0C0, A ′ B ′ A0B0 và C0, A0, B0 thẳng hàng nên ba điểm C ′ , A ′ , B ′ cũng thẳng hàng.
  • 28. Tải tài liệu tại sividoc.com Viết đề tài giá sinh viên – ZALO:0973.287.149-TEAMLUANVAN.COM 24 (2) Kẻ các đường cao AA1 ⊥ BC, BB1 ⊥ CA và CC1 ⊥ AB. Gọi H là trực tâm tam giác ABC và đường thẳng AH , BH ,CH cắt đường tròn (O, R) tại A2, B2,C2. Khi đó, theo các điểm A2, B2,C2 đối xứng với H qua BC,CA, AB, tương ứng. Vì các hình thang C ′ C2H M, B ′ B2H M cân nên ∠C ′ HC2 = ∠MC2C = ∠MAC và ∠B ′ H B2 = ∠MB2B = ∠MAB. Vậy ∠B ′ HC ′ = ∠B”HB2 + ∠B2HC2 + ∠C2HC ′ = ∠C1AB1 + ∠B1HC1 = 180 0 . Do đó, ba điểm B ′ , H ,C ′ thẳng hàng. Vì B0C0 là đường trung bình của tam giác MB ′ C ′ nên B0C0 đi qua trung điểm đoạn MH . 1.4 Định lý Ceva và Định lý Menelaus Mệnh đề 1.4.1 (Ceva 1678). Cho tam giác ABC và ba điểm M ∈ (BC), N ∈ (CA) và P ∈ (AB) sao cho AM, BN,CP không từng cặp song song. Khi đó AM, BN,CP đồng quy tại một điểm khi và chỉ khi MB NC PA = −1. MC NA PB Chứng minh. Giả thiết AM, BN,CP đồng quy tại I. Khi đó ta có biến đổi diện tích BM CN AP = S IBM S ICN S IAP = S ABM S BCN S CAP = S BAI . S CBI . S ACI = 1. MC NA PBS IMC S INA S IPB S ACM S BAN S CBP S CAI S ABI S CBI Vậy MB NC PA = −1. MC NA PB Ngược lại, giả thiết MB NC PA = −1. MC NA PB
  • 29. Tải tài liệu tại sividoc.com Viết đề tài giá sinh viên – ZALO:0973.287.149-TEAMLUANVAN.COM 25 Ta chỉ ra AM, BN, CP đồng quy tại một điểm. Thật vậy, gọi I = AM × BN và CI cắt (AB) tại Q. Khi đó ta có BM CN AP =−1= BM CN AQ . MC NA PB MC NA QB Vậy AP=AQ PB QB và dễ dàng suy ra được Q ≡ P. Nhận xét 1.4.1. Trong một số tài liệu người ta thường kẻ qua A một đường song song với BC, cắt BN và CP kéo dài để chứng minh Định lý Ceva, và phép chứng minh là rất dài. Ví dụ 1.4.1. Cho tam giác ABC. Giả sử đường tròn nội tiếp tiếp xúc với cạnh BC,CA, AB tại A1, B1,C1; Giả sử đường tròn bàng tiếp nằm trong ∠A tiếp xúc với cạnh BC tại A2, tiếp xúc với cạnh AB, AC kéo dài tại Ca, Ba. Khi đó AA1, BB1,CC1 đồng quy tại một điểm J và điểm này được gọi là điểm Gergonne và AA2, BBa,CCa đồng quy tại một điểm Na và điểm này được gọi là điểm Nagel. Bài giải. Vì BA1 CB1 AC1 = −1 và BA2 CBa ACa = −1 A1C B1A C1B A2C BaA CaB nên AA1, BB1, CC1 đồng quy tại một điểm J và AA2, BBa,CCa đồng quy tại một điểm Na theo Mệnh đề 1.4.1. Ví dụ 1.4.2. Cho tam giác ABC. Giả sử ba đường tròn bàng tiếp tiếp xúc với cạnh BC,CA, AB tại A2, B2,C2; tương ứng. Khi đó AA2, BB2,CC2 đồng quy tại một điểm N. Bài giải. Ký hiệu a = BC, b = CA, c = AB. Vì AB + BA2 = AC +CA2 nên A 2 B = a + b − c . A2C a + c − b Tương tự, tính B 2 C = b + c − a B2A b + a − c
  • 30. Tải tài liệu tại sividoc.com Viết đề tài giá sinh viên – ZALO:0973.287.149-TEAMLUANVAN.COM 26 và C 2 A = c + a − b . C2B c + b − a Vì BA 2 CB 2 AC 2 = −1 A2C B2A C2B nên AA2, BB2, CC2 đồng quy tại một điểm N. Tương tự, khi xét đường tròn bàng tiếp ΔABC, nhưng nằm trong góc ∠B, ∠C, tương ứng ta cũng nhận được hai điểm Nagel Nb , Nc nữa. Như vậy, với mỗi ΔABC ta nhận được bốn điểm Nagel N, Na , Nb, Nc và một điểm Gergonne. Ví dụ 1.4.3. Cho tam giác ABC. Giả sử đường tròn cắt ba cạnh BC,CA, AB của tam giác tại A1, A2; B1, B2; C1, C2, tương ứng và theo thứ tự A,C1,C2, B, A1, A2, C, B1, B2, A. Giả sử X = C1B1 × C2B2, Y = A1C1 × A2C2, Z = B1A1 × B2A2. Khi đó AX , BY,CZ đồng quy tại một điểm. Bài giải. Gọi D là giao điểm giữa AX và C2B1. Vì AD,C2B2, B1C1 đồng quy tại X nên SIN ∠X AB 2 . SIN ∠XC 2 C 1 . SIN ∠X B 1 C 2 = 1. SIN ∠X AC1 SIN ∠XC2B1 SIN ∠X B1B2 Do ∠XC2C1 = ∠X B1B2 nên ta nhận được SIN ∠ X AB2 = SIN ∠ B1C2B2 ,(1). SIN ∠ X AC1 SIN ∠B2B1C1 Tương tự, SIN ∠ Y BC2 = SIN ∠ C1A2C2 ,(2); SIN ∠ XCA2 =SIN ∠ A1B2A2 ,(3). SIN ∠ Y BA1 SIN ∠ A2C1A1 SIN ∠ ZCB1SIN ∠ B2A1B1 Từ (1), (2), (3) ta suy ra SIN ∠X AB 2 .SIN ∠Y BC 2 .SIN ∠XCA 2 = 1. SIN ∠X AC1 SIN ∠Y BA1 SIN ∠ZCB1 Hệ thức này chứng tỏ AX , BY,CZ đồng quy tại một điểm.
  • 31. Tải tài liệu tại sividoc.com Viết đề tài giá sinh viên – ZALO:0973.287.149-TEAMLUANVAN.COM Ví dụ 1.4.4. Cho tam giác ABC. Chia cạnh BC thành ba đoạn bằng nhau BA1 = A1A2 = A2C;
  • 32. Tải tài liệu tại sividoc.com Viết đề tài giá sinh viên – ZALO:0973.287.149-TEAMLUANVAN.COM 27 Chia cạnh CA thành ba đoạn bằng nhau CB1 = B1B2 = B2A; Chia cạnh AB thành ba đoạn bằng nhau AC1 = C1C2 = C2B. Gọi D1 = BB1 × CC2, D2 = BB1 × AA2, D3 = CC1 × AA2, D4 = CC1 × BB2, D5 = AA1 × BB2, D6 = AA1 × CC2. Chứng minh ba đường thẳng D1D4, D2D5, D3D6 đồng quy tại một điểm. Bài giải. Theo Định lý Ceva, đường trung tuyến AA ′ của tam giác ABC chứa điểm D1, D4; đường trung tuyến BB ′ của tam giác ABC chứa điểm D3, D6; đường trung tuyến CC ′ của tam giác ABC chứa điểm D2, D5. Từ đó suy ra ba đường thẳng D1D4, D2D5, D3D6 đồng quy tại một điểm. Ví dụ 1.4.5. Cho tam giác ABC. Lấy điểm M, N, P thuộc cạnh BC, CA và AB, 1 tương ứng. Nếu AM, BN,CP đồng quy thì ta luôn có SMN P 4 SABC. Bài giải. Đặt u =MB , v = NC , t = PA . MC NA PB Giả thiết AM, BN,CP đồng qui tại một điểm. Khi đó u, v,t > 0 và uvt = 1 và S MNP = 1 + uvt . (1 + u)(1 + v)(1 + t ) S ABC Vì (1 + u)(1 + v)(1 + t ) 8 nên S MNP = 1 + 1 2 =1 . S ABC (1+ u)(1 + v)(1 + t ) 8 4 Ta có điều phải chứng minh. Mệnh đề 1.4.2 (Menelaus). Cho tam giác ABC và điểm N thuộc cạnh CA, điểm P thuộc cạnh AB, còn điểm M thuộc cạnh BC kéo dài. Khi đó, ba điểm M, N, P thẳng hàng khi và chỉ khi MB . NC . PA = 1. MC NA PB
  • 33. Tải tài liệu tại sividoc.com Viết đề tài giá sinh viên – ZALO:0973.287.149-TEAMLUANVAN.COM 28 Chứng minh. Theo khoảng cách: Giả thiết ba điểm M, N, P thẳng hàng. Ký hiệu khoảng cách từ A, B,C đến đường thẳng (MN P) là u, v,t . Khi đó ta có hệ thức MB .CN .AP =v .t .u = 1 hay là MB .NC .PA = 1. MC NA PBt u v MC NA PB Theo diện tích: Ta có MB NC PA = S MBN S MCN S MAN = 1. MC NA PB S MCN S MAN S MBN Giả thiết MB.NC.PA = 1. MC NA PB Ta chỉ ra ba điểm M, N, P thẳng hàng. Gọi M′ là điểm giao N P × BC. Ta có M′ B.NC.PA = 1 = MB.NC.PA. M ′ C NA PB MC NA PB Vậy M ′ B = MB M′ C MC và cũng dễ dàng suy ra được M ≡ M′ . Mệnh đề 1.4.3. Cho tứ giác ABCD. Giả sử đường thẳng d cắt các đường thẳng AB, BC, CD, DA tại các điểm M, N, P, Q, tương ứng. Khi đó, ta có T = MA . NB . PC . QD = 1. MB NC PD QA Chứng minh. Ký hiệu khoảng cách từ các đỉnh A, B,C, D đến d qua x, y, z,t . Dễ x y z t dàng kiểm tra T = . . . = 1.
  • 34. Tải tài liệu tại sividoc.com Viết đề tài giá sinh viên – ZALO:0973.287.149-TEAMLUANVAN.COM 29 Định lí 1.4.1 (Desargues). Cho hai tam giác ABC và A′ B′ C′ sao cho ba đường thẳng AA ′ , BB ′ ,CC ′ đồng quy tại điểm E . Ký hiệu X ,Y, Z lần lượt là giao điểm giữa AB và A′ B′ ; giữa BC và B′ C′ ; giữa CA và C′ A′ . Chứng minh rằng, ba điểm X ,Y, Z thẳng hàng. Ngược lại, nếu BB ′ ,CC ′ cắt nhau tại O và X ,Y, Z thẳng hàng thì AA′ , BB′ ,CC′ đồng quy tại O. Chứng minh. Áp dụng Định lý Menelaus cho ΔE AB với cát tuyến (A′ B′ X ); cho ΔE BC với cát tuyến (B′ C′ Y ) và ΔECA với cát tuyến (C′ A′ Z), ta có A ′ E .XA. B ′ B = 1, B ′ E .YB.C ′ C = 1, C ′ E .ZC. A ′ A = 1. A ′ A X B B ′ E B ′ B YC C ′ E C ′ C ZA A ′ E Nhân ba hệ thức này, vế với vế, ta nhận đượcX A .Y B .ZC = 1 và theo Mệnh đề X B Y C Z 1.4.2 ba điểm X ,Y, Z thẳng hàng. Phần còn lại được coi như một bài tập nhỏ. 1.5 Bất đẳng thức Erdos¨-Mordell cho đa giác Trước khi phát biểu và chứng minh Bất đẳng thức Erdos¨-Mordell cho đa giác lồi ta chứng minh Bất đẳng thức Erdos¨-Mordell cho tam giác. Ví dụ 1.5.1. Cho tam giác ABC. Giả sử M là một điểm trong tam giác và ký hiệu khoảng cách từ M đến cạnh BC,CA, AB là x, y, z. Đặt d1 = MA, d2 = MB, d3 = MC. Khi đó ta có Bất đẳng thức Erdos¨-Mordell COS π(d1 + d2 + d3) x + y + z. 3 Bài giải. Cách 1. Ta có d1d2 COS α1 +d2d3 COS α2 + d3d1 COS α3x + y + z √ với α1 + α2 + α3 = π . Đặt ak = dk , k = 1, 2, 3. Ta chỉ ra a 2 1 + a 2 2 + a 2 3 2a1a2 COS α1 + 2a2a3 COS α2 + 2a3a1 COS α3 hay (a1 − a2 COS α1 − a3 COS α3) 2 + (a2 SIN α1 − a3 SIN α3) 2 0 đúng. Do vậy, ta nhận được bất đẳng thức COS π(d1 + d2 + d3) x + y + z. 3 Cách 2. Ta có d1 2 SIN 2 A = y 2 + z 2 − 2yz COS(B +C)
  • 35. Tải tài liệu tại sividoc.com Viết đề tài giá sinh viên – ZALO:0973.287.149-TEAMLUANVAN.COM 30 = (y SIN C + z SIN B) 2 + (y COS C − z COS B) 2 (y SIN C + z SIN B) 2 Vậy d1 ySIN C + zSIN B . SIN A SIN A Tương tự, ta có d2 zSIN A + x SIN C và d3x SIN B + y SIN A . SIN BSIN B SIN CSIN C Cộng ba bất đẳng thức trên có COS π (d1 + d2 + d3) x + y + z. 3 Mệnh đề 1.5.1. Cho đa giác lồi A1A2 . . . An với n 3. Giả sử M là một điểm trong đa giác và ký hiệu khoảng cách từ M đến cạnh AiAi+1 là ri với i = 1, 2, . . . , n và quy ước n + 1 := 1. Đặt 2αi = ∠AiMAi+1 và dk = MAk với k = 1, . . . , n. Ta có đồng nhất thức cho Bất đẳng thức Erdos¨-Mordell: n n ∑ dk dk+1 SIN 2αk = ∑ rk (dk − dk+1)2 + 4dk dk+1 SIN 2 αk . k=1 k=1 Từ đó suy ra n n ∑ dk dk+1 COS αk∑ rk . k=1 k=1 Chứng minh. Xét ΔA1MA2. Từ r1.A1A2 = d1d2 SIN 2α1 ta suy ra được: d1d2 SIN 2α1 = r1 (d1 − d2) 2 + 4d1d2 SIN 2 α1. Tương tự, ta có: dk dk+1 SIN 2αk = rk (dk − dk+1) 2 + 4dk dk+1 SIN 2 αk với k = 1, 2, . . . , n. Lấy tổng tất cả các hệ thức được: n n ∑ dk dk+1 SIN 2αk = ∑ rk (dk − dk+1)2 + 4dk dk+1 SIN 2 αk . k=1 k=1 Vì (dk − dk+1) 2 + 4dk dk+1 SIN 2 αk 2 d k d k+1 SIN αk và viết SIN 2αk = 2 SIN αk COS αk n n nên ta có bất đẳng thức ∑ dk dk+1 COS αk∑ rk . k=1 k=1
  • 36. Tải tài liệu tại sividoc.com Viết đề tài giá sinh viên – ZALO:0973.287.149-TEAMLUANVAN.COM 31 Hai kết quả dưới đây là của Wolstenholme và Lenhard: Mệnh đề 1.5.2. Cho đa giác lồi A1A2 . . . An với n 3. Giả sử M là một điểm trong đa giác. Ký hiệu khoảng cách từ M đến cạnh AiAi+1 là ri, di = MAi, với i = 1, 2, . . . , n và quy ước n + 1 := 1. Khi đó ta có các bất đẳng thức (1) π n n dk dk+1 COS αk . COS ∑ dk ∑ n k=1 k=1 (2) n 1 n ∑ dk ∑ rk . π k=1 COS k=1 n π n n Chứng minh. (1) Bất đẳng thức dk dk+1 COS αk . Theo mệnh đề COS ∑ dk ∑ n n n k=1 k=1 trên ta đã có COS αk được suy ra từ một kết quả của Wolsten- ∑ rk ∑ d k d k+1 k=1 k=1 holme và Lenhard. (2) Vì n n n 1 n ∑ dk dk+1 COS αk ∑ rk nên ∑ dk ∑ rk . π k=1 k=1 k=1 COS k=1 n Mệnh đề 1.5.3 (Hayashi). Giả sử tam giác ABC có BC = a, CA = b, AB = c. Khi đó, với bất kỳ điểm M ở trong mặt phẳng ABC, ta có bất đẳng thức: aMB.MC + bMC.MA + cMA.MB abc. Đặc biệt, khi M ≡ G là trọng tâm của tam giác ABC ta có bất đẳng thức: 9 am b m c + bm c m a + cm a m b 4 abc và khi M ≡ I là tâm đường tròn nội tiếp của tam giác ABC ta còn có bất đẳng thức: a b c abc + + . IA IB IC 4Rr 2 Chứng minh. Tương ứng đỉnh A, B,C với số phức z1, z2, z3 và điểm M ứng với số phức z. Từ bất đẳng thức |z − z2||z − z3| + |z − z3||z − z1| + |z − z1||z − z2| 1, |z1 − z2||z1 − z3| |z2 − z3||z2 − z1| |z3 − z1||z3 − z2| ta suy ra MC.MA MA.MB MB.MC + + 1. bc ca ab
  • 37. Tải tài liệu tại sividoc.com Viết đề tài giá sinh viên – ZALO:0973.287.149-TEAMLUANVAN.COM 32 Vậy a.MB.MC + b.MC.MA + c.MA.MB abc. Khi M ≡ I là tâm đường tròn nội tiếp của tam giác ABC ta còn có bất đẳng thức. a + b + c abc ,vì IA.IB.IB = 4Rr 2 . IA IB IC 4Rr 2 Mệnh đề 1.5.4. Giả sử tam giác ABC có BC = a,CA = b, AB = c. Gọi A1, B1,C1 là trung điểm của các cạnh BC,CA, AB, tương ứng. Ký hiệu khoảng cách từ điểm M trong mặt phẳng ABC đến A1, B1,C1 là x, y, z. Khi đó có bất đẳng thức: axMA + byMB + czMCabc 2 . Chứng minh. Tương ứng đỉnh A, B,C với số phức z1, z2, z3 và điểm M ứng với số phức 0. Từ bất đẳng thức |z1||z 2 2 − z 2 3| + |z2||z 2 3 − z 2 1| + |z3||z 2 1 − z 2 2| |z1 − z2||z2 − z3||z3 − z1| abc 2 . Mệnh đề 1.5.5. Giả sử tam giác ABC có BC = a,CA = b, AB = c. Khi đó, với bất kỳ điểm M ở trong mặt phẳng ABC, ta có các bất đẳng thức: aMA 2 + bMB 2 + cMC 2 abc. Hơn nữa ta còn có aH A 2 + bH B 2 + cHC 2 abc, trong đó H là trực tâm của ΔABC. Chứng minh. Tương ứng đỉnh A, B,C với số phức z1, z2, z3 và điểm M ứng với số phức 0. Từ bất đẳng thức |z1| 2 |z2 − z3| + |z2| 2 |z3 − z1| + |z3| 2 |z1 − z2| |z1 − z2||z2 − z3||z3 − z1| ta có aMA 2 + bMB 2 + cMC 2 abc. Mệnh đề 1.5.6. Giả sử tam giác ABC có BC = a,CA = b, AB = c. Gọi G là trọng tâm của tam giác. Với bất kỳ điểm M ở trong mặt phẳng ABC, ta luôn có bất đẳng thức: (1) a 3 MA + b 3 MB + c 3 MC 3abcMG. theo ta có axMA + byMB + czMC
  • 38. Tải tài liệu tại sividoc.com Viết đề tài giá sinh viên – ZALO:0973.287.149-TEAMLUANVAN.COM 33 (2) aMA 3 + bMB 3 + cMC 3 3abcMG. Chứng minh. Tương ứng đỉnh A, B,C với số phức z1, z2, z3 và điểm M ứng với số phức 0. Từ bất đẳng thức |z1||z2 − z3 | 3 + |z2||z3 − z1| 3 + |z3||z1 − z2| 3 |z1 + z2 + z3||z1 − z2 ||z2 − z3||z3 − z1|, ta có a 3 MA + b 3 MB + c 3 MC3abcMG. Bất đẳng thức aMA 3 + bMB 3 + cMC 3 3abcMG được chứng minh tương tự. Ví dụ 1.5.2. Giả sử tam giác ABC có BC = a,CA = b, AB = c. Khi đó, với bất kỳ điểm M trong mặt phẳng ABC, ta luôn có bất đẳng thức: abc 2 aMB.MC + bMC.MA + cMA.MB R2 MO . Đặc biệt, khi M ≡ K là tâm đường tròn bàng tiếp của tam giác ABC và ở trong ∠A ta còn có bất đẳng thức: a + b + c abc(R + 2ra) . KAKBKC 4R2r2 a Bài giải. Tương ứng các điểm A, B,C, E và tâm đường tròn ngoại tiếp O của ΔABC với số phức z1, z2, z3, z, 0. Từ bất đẳng thức: |z1| 2 |z − z2||z − z3| +|z2| 2 |z − z3||z − z1| +|z3| 2 |z − z1||z − z2| z 2 |z1 − z2 ||z1− z3| |z2 − z3 ||z2 − z1 | |z3 − z1 ||z3− z2 | | | với n = 2, ta suy ra MB.MC MC.MAMA.MB MO 2 + + . bc ca ab R 2 Như vậy chúng ta có aMB.MC + bMC.MA + cMA.MB abc MO 2 . R2 Khi M ≡ K là tâm đường tròn bàng tiếp của tam giác ABC và ở trong ∠A ta có KO 2 = R 2 + 2Rra, K A.K B.KC = 4Rra 2 và như vậy có bất đẳng thức a + b + c abc(R2 + 2Rra) =abc(R + 2ra) . R2 .4Rra 2 4R2 ra 2 KAKBKC
  • 39. Tải tài liệu tại sividoc.com Viết đề tài giá sinh viên – ZALO:0973.287.149-TEAMLUANVAN.COM 34 Mệnh đề 1.5.7. Giả sử tam giác ABC có BC = a,CA = b, AB = c nội tiếp trong đường tròn tâm O bán kính R. Khi đó, với bất kỳ hai điểm E và F ở trong mặt phẳng ABC ta luôn có bất đẳng thức: OE.OF AE.AF +BE.BF + CE.CF . R2 bc ca ab Đặc biệt, khi E ≡ F ta có AE 2 + BE 2 + CE 2 OE 2 . bc ca ab R2 Chứng minh. Tương ứng đỉnh A, B,C với số phức z1, z2, z3, tâm O với số phức z và hai điểm E , F ứng với hai số phức u, v. Từ bất đẳng thức dưới đây: |z − u||z − v| |z1 − u||z1 − v| |z − z1||z − z2||z − z3| |z1 − z2||z1 − z3||z − z1| + |z2 − u||z2 − v| + |z3 − u||z3 − v| |z2 − z1||z3 − z2||z − z3| − z1||z2 − z3||z − z2| |z3 ta suy ra OE.OF AE.AF +BE.BF + CE.CF . R 2 ca bc ab Mệnh đề 1.5.8. Tam giác ABC với BC = a,CA = b, AB = c có đường tròn ngoại tiếp tâm O bán kính R và đường tròn nội tiếp tâm I bán kính r. Gọi H là trực tâm của tam giác. Ký hiệu khoảng cách từ O đến các cạnh BC,CA, AB là x, y, z. Ta có bất đẳng thức: (1) ax + by + cz abcMH với bất kỳ điểm M. MA MB MC2MA.MB.MC (2)ax +by +cz abc IH . IC 8Rr 2 IA IB Chứng minh. (1) Tương ứng đỉnh A, B,C, O với số phức z1, z2, z3, 0 và M với số phức z. Từ bất đẳng thức |z2 + z3| + |z3 + z1| + |z1 + z2| |z1 − z2||z1 − z3||z1 − t | |z2 − z3 ||z2 − z1||z2 − t | |z3 − z1||z3 − z2||z3 − t | |t − z1 − z2 − z3| |t − z1||t − z2||t − z3| ta có ngay bất đẳng nhất thức ax + by + cz abcMH . MC2MA.MB.MC (2) Khi M ≡ I ta có MA MB ax + by + cz abc IH = abc IH . IA IB IC 2IA.IB.IC 8Rr2 Ta có điều phải chứng minh.
  • 40. Tải tài liệu tại sividoc.com Viết đề tài giá sinh viên – ZALO:0973.287.149-TEAMLUANVAN.COM 35 Chương 2 Phương pháp thể tích 2.1 Phương pháp thể tích 2.1.1 Phương pháp thể tích Để nghiên cứu có hiệu quả, chúng ta cần phương pháp đơn giản sau đây: Phương pháp thể tích là phương pháp giải bài toán hình trong không gian qua thể tích của một khối nào đó. Để giải bài toán đã cho ta chọn một khối đa diện (K), chẳng hạn, với thể tích V. Sau đó đem chia khối (K) ra thành nhiều khối nhỏ một cách thích hợp và tính thể tích V1, . . . ,Vr của các khối nhỏ ấy. Từ V = V1 + · · · +Vr suy ra lời giải bài toán. Mệnh đề 2.1.1. Cho tứ diện ABCD với diện tích 4 mặt sa, sb, sc, sd ; độ dài các đường cao ha, hb, hc, hd ; độ dài các bán kính mặt cầu ngoại, nội, bàng tiếp R, r, Ra, Rb, Rc, Rd . Khi đó, thể tích V của tứ diện ABCD được xác định qua các công thức (1) V = sa 3 ha = sb 3 hb = sc 3 hc = sd 3 hd . (2) V = (sa + sb + sc + sd )r = sr , ở đó s = sa + sb + sc + sd . 33 (3) V = (s − s a )R a = (s − 2s b )R b = (s − s c )R c = (s − s d )R d . 3333 x 1 y 1 z 1 1 (4)V= 1 x 2 y 2 z 2 1 , trong đó A(x1, y1, z1), B(x2, y2, z2), C(x3, y3, z3), 6 x3 y3 z3 1 x4 y4 z4 1 D(x4, y4, z4) trong hệ tọa độ Oxyz.
  • 41. Tải tài liệu tại sividoc.com Viết đề tài giá sinh viên – ZALO:0973.287.149-TEAMLUANVAN.COM 36 2.1.2 Thể tích qua định thức Mệnh đề 2.1.2. Cho các vector −→ , −→ , −→ trong không gian với hệ tọa độ trực x y z chuẩn. Ta có đồng nhất thức −→ −→ −→ −→ −→ −→ −→−→ −→ 2 x . xx . y x . z = −→ −→ −→ −→ −→ −→ [ x , y ]. z y . x y . y y . z. −→ −→ −→ −→ −→ −→ z . x z . y z . z Chứng minh. Đặt −→ = (a, b, c), −→ = (a ′ , b ′ , c ′ ), −→ = (a”, b”, c”). Biến đổi đơn x y z giản 2 −→ −→ −→ a b c a b c 2 b ′ c ′ = a ′ b ′ c ′ [ x , y ]. z = a ′ a” b” c” a” b” c” ta nhận được hệ thức liên hệ a a ′ a” b b ′ b” c c′ c” −→ −→ −→ −→ −→ −→ −→ −→ −→ 2 x . x x . y x . z −→ −→ −→ −→ −→ −→ . [ x , y ]. z = y . x y . y y . z −→ −→ −→ −→ −→ −→ z . x z . y z . z Ta có điều phải chứng minh. Mệnh đề 2.1.3. Giả sử a1, a2, a3 và b1, b2, b3 là cosin của các góc do hai đường thẳng (ℓ1) và (ℓ2) tạo với các trục tọa độ Ox, Oy, Oz, tương ứng của hệ tọa độ trực chuẩn (Oxyz). Ký hiệu α là góc giữa tia (ℓ1) và (ℓ2). Khi đó ta có công thức biểu diễn SIN 2 α = (a1b2 − a2b1) 2 + (a2b3 − a3b2) 2 + (a3b1 − a1b3) 2 . Từ đó suy ra rằng, hai đường thẳng (ℓ1) và (ℓ2) vuông góc với nhau khi và chỉ khi (a1b2 − a2b1) 2 + (a2b3 − a3b2) 2 + (a3b1 − a1b3) 2 = 1. Bài giải. Gọi −→ −→ −→ Khi đó i , j , k là vector đơn vị trên các trục Ox, Oy, Oz. −→ −→ −→ −→ −→ −→ −→ −→ u = a1 i + a2 j + a3 k , v = b1 i + b2 j + b3 k là những vector đơn vị chỉ phương của (ℓ1) và (ℓ2), và ta có 2 −→ −→ 2 = (a1b2 − a2b1) 2 + (a2b3 − a3b2) 2 + (a3b1 2 . SIN α = [ u , v ] − a1b3)
  • 42. Tải tài liệu tại sividoc.com Viết đề tài giá sinh viên – ZALO:0973.287.149-TEAMLUANVAN.COM 37 Ví dụ 2.1.1. Tính thể tích hình chóp SABC với SA = a, SB = b, SC = c và góc giữa SB, SC; giữa SC, SA; giữa SA, SB bằng ϕ1, ϕ2, ϕ3, tương ứng. Bài giải. Đặt −→ −→ −→ −→ −→ −→ Từ x = SA, y = SB, z = SC và V = VSABC. −→ −→ −→ −→ −→ −→ −→ −→ −→ x . x x . y x . z 36 2 2 = −→ −→ −→ −→ −→ −→ V = |[SA, SB], SC| y . xy . yy . z −→ −→ −→ −→ −→ −→ z . x z . y z . z a2 ab COS ϕ3 ac COS ϕ2 = ab COS ϕ3 b2 bc COS ϕ1 ac COS ϕ2 bc COS ϕ1 c 2 theo Mệnh đề 2.1.2 và ta suy ra ngay công thức tính thể tích V của tứ diện theo cạnh và góc V = abc 1 − COS 2 ϕ1 − COS 2 ϕ2 − COS 2 ϕ3 + 2 COS ϕ1 COS ϕ2 COS ϕ3. 6 Mệnh đề 2.1.4. Giả sử hình chóp SABC có độ dài cạnh SA = a, SB = b, SC = c, BC = x, CA = y, AB = z. Ta có công thức tính thể tích tứ diện: 1 2a 2 a 2 + b 2 − z 2 a 2 + c 2 − y 2 V = 12 √ a 2 + b 2 − z 2 2b 2 b 2 + c 2 − x 2 . 2 a 2 + c 2 − y 2 b 2 + c 2 − x 2 2c 2 Chứng minh. Như trên, có 36V 2 −→ −→ −→ 2 hay biểu diễn V qua định thức = |[SA, SB], SC| −→ −→ −→ −→ −→ −→ 2 ab COS ϕ3 ac COS ϕ2 x . x x . y x . z a 36 2 = −→ −→ −→ −→ −→ −→ = ab COS ϕ3 2 bc COS ϕ1 V y . x y . y y . z b −→ −→ −→ −→ −→ −→ ac COS ϕ2 bc COS ϕ1 2 z . x z . y z . z c theo Mệnh đề 2.1.2. Từ đây suy ra ngay hệ thức liên hệ qua định thức sau đây: 2a 2 2ab COS ϕ3 2ac COS ϕ2 288V 2 = 2ab COS ϕ3 2b 2 2bc COS ϕ1 2ac COS ϕ2 2bc COS ϕ1 2c 2
  • 43. Tải tài liệu tại sividoc.com Viết đề tài giá sinh viên – ZALO:0973.287.149-TEAMLUANVAN.COM 38 và có 1 2a2 a2 + b2 − z2 a2 + c2 − y2 V = 12 √ a2 + b2 − z2 2b 2 b2 + c2 − x2 . 2 a 2 + c 2 − y 2 b 2 + c 2 − x 2 2c 2 Điều phải chứng minh. Mệnh đề 2.1.5. Giả sử tứ diện ABCD có tọa độ 4 đỉnh là A(x1, y1, z1), B(x2, y2, z2), C(x3, y3, z3) và D(x4, y4, z4). Khi đó ta có công thức tính thể tích tứ diện qua tọa độ bốn đỉnh: x1 y1 z1 1 V = 1 x2 y2 z2 1 . 6 x3 y3 z3 1 x4 y4 z4 1 Chứng minh. Vì x1 y1 z1 1 x2 − x1 y2 − y1 z2 − z1 x2 y2 z2 1 x 3 y 3 z 3 1 = x3 − x1 y3 − y1 z3 − z1 x4 y4 z4 1 x4 − x1 y4 − y1 z4 − z1 nên ta có hệ thức x1 y1 z1 1 x2 y2 z2 1 −→ −→ −→ x 3 y 3 z 3 1 = [AB,AC].AD = 6V. x4 y4 z4 1 Ví dụ 2.1.2. Cho tứ diện A1A2A3A4. Chứng minh rằng, với điểm O ta có bất đẳng thức 1 4 V A1A2A3A4 ∏ OAi 2 + 1. 6 i=1
  • 44. Tải tài liệu tại sividoc.com Viết đề tài giá sinh viên – ZALO:0973.287.149-TEAMLUANVAN.COM 39 Bài giải. Dựng hệ tọa độ Oxyz. Với Ai(xi, yi, zi), i = 1, 2, 3, 4 có 2 x1 y1 z1 1 36V 2 = x 2 y2 z2 1 A1A2A3A4 x3 y3 z3 1 x4 y4 z4 1 theo Bất đẳng thức Hadamard. Vậy VA1 A2 A3 A4 4 ∏(x 2 i + y 2 i + z 2 i + 1) i=1 1 4 ∏ OA 2 i + 1. 6 i=1 Ví dụ 2.1.3. Cho điểm O ở trong tứ diện ABCD. Mặt phẳng (P) qua O cắt AB, AC, AD. Ký hiệu khoảng cách từ A, B,C và D đến (P) là ta,tb ,tc,td , tương ứng. Chứng minh t a V OBCD = t b V OCDA + t c V ODAB + t d V OABC. Bài giải. Dựng hệ tọa độ Oxyz để O(0, 0, 0), (P) : z = 0 và A(x1, y1, z1), B(x2, y2, z2), C(x3, y3, z3), D(x4, y4, z4) với ta = z1 > 0, tb = −z2 > 0, tc = −z3 > 0, td = −z4 > 0. Quy ước thể tích tứ diện được tính qua định thức 0 0 0 1 x2 y2 z2 1 x2 y2 z2 t a V OBCD = z1 = −z1 x3 y3 z3 x3 y3 z3 1 x 4 y 4 z 4 1 x4 y4 z4 0 0 0 1 x4 y4 z4 1 x1 y1 z1 t b V OCDA = −z2 = −z2 x3 y3 z3 x3 y3 z3 1 x1 y1 z1 1 x4 y4 z4 0 0 0 1 x4 y4 z4 1 x1 y1 z1 t c V ODAB = −z3 = z3 x2 y2 z2 x1 y1 z1 1 x2 y2 z2 1 x4 y4 z4 0 0 0 1 x 1 y 1 z 1 1 x1 y1 z1
  • 45. Tải tài liệu tại sividoc.com Viết đề tài giá sinh viên – ZALO:0973.287.149-TEAMLUANVAN.COM t d V OABC = −z4 = −z4 x2 y2 z2 . x3 y3 z3 1 x2 y2 z2 1 x3 y3 z3
  • 46. Tải tài liệu tại sividoc.com Viết đề tài giá sinh viên – ZALO:0973.287.149-TEAMLUANVAN.COM 40 Như vậy, T = taVOBCD − tbVOCDA − tcVODAB − tdVOABC đúng bằng x2 y2 z2 x1 y1 z1 x1 y1 z1 T = −z1 x3 y3 z3 + z2 x3 y3 z3 − z3 x2 y2 z2 x4 y4 z4 x 4 y 4 z 4 x4 y4 z4 x1 y1 z1 x1y1 z1 z1 = x 2y2 z2 z2 +z4 x 2 y 2 z 2 = 0. x3 y3 z3 x3y3 z3 z3 x 4 y 4 z 4 z 4 Tóm lại t a V OBCD = t b V OCDA + t c V ODAB + t d V OABC. Ví dụ 2.1.4. Cho tứ diện ABCD. Gọi A1, B1,C1, D1 là trọng tâm các mặt BCD, CDA, DAB, ABC, tương ứng. Chứng minh rằng, với bất kỳ điểm O ở bên ngoài tứ diện ABCD có một cách chọn thích hợp u, v,t ∈ {1, −1} để 1 |V OB1C1 D1 + uV OC1 D1 A1 + vV OD1A1 B1 + tV OA1B1C1 | = 27 V ABCD . Bài giải. Dựng hệ Oxyz để O(0, 0, 0), A(x1, y1, z1), B(x2, y2, z2), C(x3, y3, z3) và D(x4, y4, z4). Tọa độ trọng tâm mặt tứ diện ABCD A1 ( x2 + x3 + x4 , y2 + y3 + y4 , z2 + z3 + z4 ) 3 3 3 B 1 ( x1 + x3 + x4 ,y1 + y3 + y4 , z1 + z3 + z4 ) 3 3 3 C1 (x2 + x1 + x4 ,y2 + y1 + y4 , z2 + z1 + z4 ) 3 3 3 D1 (x2 + x3 + x1 ,y2 + y3 + y1 ,z2 + z3 + z1 ). 3 3 3 Đặt 4 4 4 X = ∑ xi, Y = ∑ yi, Z = ∑ zi, V a = 6V OB1C1 D1 . i=1 i=1 i=1 Ta có 0 0 0 1 x1 + x3 + x4 y 1+ y3 + y4 z1 + z3 + z4 1 Va = ± 3 3 3 x2 + x1 + x4 y2+ y1 + y4 z2 + z1 + z4 1 + x1 3 3 3 + y1 + z1 x2 + x3 y2+ y3 z2 + z3 1 3 3 3
  • 47. Tải tài liệu tại sividoc.com Viết đề tài giá sinh viên – ZALO:0973.287.149-TEAMLUANVAN.COM 41 = ±1 27 Đặt X − x2 X − x3 X − x4 Y − y2 Y − y3 Y − y4 Z − z2 X Y Z 1 ± 1 x 2 y 2 z 2 1 Z z = . − 3 27 x3 y3 z3 1 Z − z4 x4 y4 z4 1 X Y T = x2 y2 x3 y3 x4 y4 X Y ± x 1 y 1 x2 y2 x4 y4 Z 1 X Y z2 1 ± x1 y1 z3 1 x 3 y 3 z4 1 x4 y4 Z 1 X Y z 1 1 ± x1 y1 z2 1 x2 y2 z4 1 x3 y3 Z 1 z1 1 z3 1 z4 1 Z 1 z1 1 z2 1 z3 1 với việc chọn dấu +, − một cách thích hợp. Từ X Y Z 1 1 x1 y1 z1 1 1 x2 y2 z2 1 1 = 0 x3 y3 z3 1 1 x4 y4 z4 1 1 suy ra x1 y1 z1 1 X YZ 1 x2 y2 z2 1 − x2 y2 z2 1 x3 y3 z3 1 x3 y3 z3 1 x4 y4 z4 1 x4 y4 z4 1 X YZ 1 X YZ 1 X YZ 1 + x 1 y 1 z 1 1 − x1 y1 z1 1 + x 1 y 1 z 1 1 = 0. x3 y3 z3 1 x2 y2 z2 1 x2 y2 z2 1 x4 y4 z4 1 x 4 y 4 z 4 1 x3 y3 z3 1 Như vậy ± V ABCD ± 27V OB1C1 D1 ± 27V OC1D1 A1 ± 27V OD1A1B1 ± 27V OA1B1C1 = 0 .
  • 48. Tải tài liệu tại sividoc.com Viết đề tài giá sinh viên – ZALO:0973.287.149-TEAMLUANVAN.COM 42 Từ đó suy ra kết quả: Tồn tại một cách chọn thích hợp u, v,t ∈ {1, −1} để 1 |V OB1C1 D1 + uV OC1 D1 A1 + vV OD1A1 B1 + tV OA1B1C1 | = 27 V ABCD . Ta có điều phải chứng minh. 2.2 Quan hệ bán kính mặt cầu ngoại-nội tiếp Mệnh đề 2.2.1. Tứ diện A1A2 A3A4 có tọa độ bốn đỉnh A1(x1, y1, z1), A2(x2, y2, z2), A3(x3, y3, z3) và A4(x4, y4, z4). Ký hiệu R là bán kính mặt cầu ngoại tiếp tứ diện. Khi đó ta có công thức tính bán kính mặt cầu ngoại tiếp R qua độ dài 6 cạnh tứ diện: 2l13 2 l14 2 l23 2 l24 2 + 2l12 2 l14 2 l32 2 l34 2 + 2l12 2 l13 2 l42 2 l43 2 − l12 4 l34 4 − l13 4 l24 4 − l14 4 l23 4 R = 24V với V là thể tích tứ diện và li j = l ji là độ dài cạnh AiA j = A j Ai, i = j. Chứng minh. Ta biết thể tích tứ diện không thay đổi qua một phép tịnh tiến. Do vậy, không hạn chế có thể giả thiết tâm mặt cầu ngoại tiếp tứ diện A1A2A3A4 chính là gốc tọa độ. Ta có hệ phương trình tọa độ của mặt cầu qua 4 đỉnh tứ diện sau đây: x1 2 + y1 2 + z1 2 − R2 = 0 2 y 2 z 2 R 2 0 x 2 + 2 + 2 = − 2 2 2 2 x3 + y3 + z3 − R = 0 2 y 2 z 2 R 2 0 x 4 + 4 + 4 = . − Từ đây suy ra R 2 = (xi − x j ) 2 + (yi − y j ) 2 + (zi − z j ) 2 + xix j + yiy j + ziz j 2 với mọi i, j = 1, 2, 3, 4, i = j. Đặt ti j = R 2 − xix j − yiy j − ziz j . Khi đó ti j = t ji = (x i − x j ) 2 + ( y i − y j ) 2 + ( z i − z j ) 2 = l i 2 j = l2 ji .
  • 49. Tải tài liệu tại sividoc.com Viết đề tài giá sinh viên – ZALO:0973.287.149-TEAMLUANVAN.COM 2 2 2
  • 50. Tải tài liệu tại sividoc.com Viết đề tài giá sinh viên – ZALO:0973.287.149-TEAMLUANVAN.COM 43 Xét tích sau x1 y1 z1 R −x1 P = x2 y2 z2 R . − x 2 x3 y3 z3 R −x3 x4 y4 z4 R −x4 −y1 −z1 R −y2 −z2 R −y3 −z3 R −y4 −z4 R x1 y1 z1 R −x1 −x2 −x3 −x4 = x2 y2 z2 R . − y 1 −y2 −y3 −y4 x3 y3 z3 R −z1 −z2 −z3 −z4 x4 y4 z4 R R R R R 0 t 12 t 13 t 14 t 21 0 t 23 t 24 với ti j = t ji. = 0 t 31 t 32 t 34 t 41 t 42 t 43 0 Vì x1 y1 z1 R −x1 x2 y2 z2 R và −x2 x 3 y 3 z 3 R −x3 x 4 y 4 z 4 R −x4 đều bằng 6RV nên −y1 −z1 R −y2 −z2 R −y3 −z3 R −y4 −z4 R 0 l12 2 l13 2 2 2 l21 2 0 l23 2 36R 2 V 2 = 2 2 l32 2 l31 2 0 2 2 l43 2 l41 2 l42 2 2 2 2 l14 2 2 0l12 2 l13 2 l14 2 l24 2 1 l21 2 0l23 2 l24 2 2 l34 2 = . 16 l2 l2 0l2 31 32 34 2 l41 2 l42 2 l43 2 0 0 Tính định thức cuối cùng và lấy căn ta nhận được công thức xác định bán kính mặt cầu ngoại R = 2l13 2 l14 2 l23 2 l24 2 + 2l12 2 l14 2 l23 2 l34 2 + 2l12 2 l13 2 l24 2 l34 2 − l12 4 l34 4 − l13 4 l24 4 − l14 4 l23 4 . 24V Ta có điều cần chứng minh.
  • 51. Tải tài liệu tại sividoc.com Viết đề tài giá sinh viên – ZALO:0973.287.149-TEAMLUANVAN.COM 44 Từ Mệnh đề 2.1.4 và Mệnh dề 2.2.1 ta suy ra công thức tính bán kính mặt cầu ngoại tiếp qua độ dài 6 cạnh tứ diện. Hệ quả 2.2.1. Tứ diện A1A2A3A4 có độ dài 6 cạnh là a = l12, b = l13, c = l14, x = l34, y = l24, z = l23. Đặt 2S = ax + by + cz. Khi đó bán kính R của mặt cầu ngoại tiếp tứ diện được xác định bởi công thức dưới đây: (1) R = S(S − ax)(S − by)(S − cz) . 6V √ (2) R = 2 2S(S − ax)(S − by)(S − cz) . 2a 2 a 2 + b 2 − z 2 a 2 + c 2 − y 2 a2 + b2 − z2 2b2 b2 + c2 − x2 a2 + c2 − y2 b2 + c2 − x2 2c 2 Chứng minh. (1) Theo Bất đẳng thức Ptolemy, có ax + by cz, by + cz ax và cz + ax by, Dễ dàng kiểm tra hệ thức 16S(S − ax)(S − by)(S − cz) = 2l13 2 l14 2 l23 2 l24 2 + 2l12 2 l14 2 l32 2 l34 2 + 2l12 2 l13 2 l42 2 l43 2 − l12 4 l34 4 − l13 4 l24 4 − l14 4 l23 4 . Do vậy ta có R = S(S − ax)(S − by)(S − cz) . 6V Dễ dàng có (2) để xác định R qua 6 cạnh tứ diện 2√ 2 S(S − ax)(S − by)(S − cz) R = . 2a 2 a 2 + b 2 − z 2 a 2 + c 2 − y 2 a2 + b2 − z2 2b 2 b2 + c2 − x2 a 2 + c 2 − y 2 b 2 + c 2 − x 2 2c2 Ta có điều phải chứng minh. Hệ quả 2.2.2. Tứ diện A1A2A3A4 có độ dài 6 cạnh là a = l12, b = l13 và diện tích toàn phần là s. Gọi R, r là bán kính mặt cầu ngoại, nội tiếp của tứ diện. Ta có hệ thức Rr = 2l13 2 l14 2 l23 2 l24 2 + 2l12 2 l14 2 l32 2 l34 2 + 2l12 2 l13 2 l42 2 l43 2 − l12 4 l34 4 − l13 4 l24 4 − l14 4 l23 4 . 8s
  • 52. Tải tài liệu tại sividoc.com Viết đề tài giá sinh viên – ZALO:0973.287.149-TEAMLUANVAN.COM 45 Chứng minh. Kết quả được suy ra từ công thức thể tích V = 3sr và Mệnh đề 2.2.1.
  • 53. Tải tài liệu tại sividoc.com Viết đề tài giá sinh viên – ZALO:0973.287.149-TEAMLUANVAN.COM 46 Chương 3 Vận dụng giải bài thi học sinh giỏi Ví dụ 3.0.1. Qua một điểm O trong không gian ta dựng n đường thẳng Odi. Giả sử góc giữa hai đường thẳng bất kỳ thuộc n đường thẳng Odi không nhỏ hơn α với 0<α< π . Chứng minh rằng n 1 . 2 1−COS α 2 Bài giải. Ta dựng hình cầu (C) tâm O với bán kính 1 và các hình nón tròn xoay đỉnh O, trục Odi, với đường sinh Oli sao cho ∠liOdi = α 2 , i = 1, 2, . . . , n. Ký hiệu Ki là phần thể tích hình nón dựng như trên nằm trong hình cầu (C). Theo công thức tính thể tích khối tròn xoay ta có 1 4 α V = Ki = 2 π f (x) 2 d x = π 1− COS , 3 2 0 trong đó α α f (x) = TAN x với 0 x COS 2 2 và α với f (x) =1 − x 2 COS x 1. 2 Vì nV VC = 4 π .1 3 nên n 1 . 3 1−COS α 2 Ví dụ 3.0.2. Có 2017 điểm nằm bên trong một hình lập phương với độ dài cạnh bằng 9. Chứng minh rằng, có hai điểm trong số 2017 điểm trên với khoảng cách nhỏ hơn 1. Bài giải. Giả sử khoảng cách giữa hai điểm bất khỳ thuộc 2017 điểm đã cho cách nhau một khoảng cách lớn hơn hoặc bằng 1. Khi đó, các hình cầu với tâm là một
  • 54. Tải tài liệu tại sividoc.com Viết đề tài giá sinh viên – ZALO:0973.287.149-TEAMLUANVAN.COM 47 trong 2017 điểm đã cho và bán kính 1 2 có phần trong không giao nhau. Giả sử hình lập phương ABCD.A′ B′ C′ D′ độ dài cạnh 9 chứa 2017 điểm đã cho. Dựng hình lập phương A1B1C1D1.A′ 1B′ 1C1 ′ D′ 1 chứa hình lập phương ABCD.A′ B′ C′ D′ ở trong sao cho mặt tương ứng song song và cách nhau một khoảng cách 1 2 , chẳng hạn (ABCD) và hai mặt phẳng này cách nhau một khoảng cách 1 . Vì 2 hình lập phương A1B1C1D1.A ′ 1B ′ 1C1 ′ D ′ 1 chứa tất cả các phần trong của 2017 hình cầu nên 1000 = 10 3 > 2017. 4 3 .π . 1 2 3 > 1000, vô lý. Vậy, điều giả sử là sai. Ví dụ 3.0.3. Qua điểm M thuộc mặt Ellipsoid (E ) : x2 + y2 + z2 = 1 ta dựng các a2 b2 c2 mặt phẳng vuông góc với các trục tọa độ Ox, Oy, Oz, và có các diện tích thiết diện Sx, Sy, Sz, tương ứng. Xác định giả trị lớn nhất của P = Sx.Sy.Sz. Bài giải. Giả sử M(x0, y0, z0) ∈ (E ). Khi đó x0 2 + y0 2 + z0 2 = 1. a2 b2 c2 phẳng qua M vuông góc với Ox cắt (E ) có phương trình (Ex) : Diện tích Sx của (Ex) bằng Thiết diện do mặt y2 z2 x0 2 + = 1 − . b 2 c 2 a 2 x2 S = π bc 1 − 0 . x a 2 Hoàn toàn tương tự, ta còn có hai kết quả y0 2 z0 2 Sy = π ca 1 − , Sz = π ab 1 − . b 2 c 2 Vậy, ta dễ dàng nhận được kết quả aSx + bSy + cSz = π abc 3 − x0 2 y0 2 z0 2 = 2π abc. − − a 2 b 2 c 2 Vì 2π abc = aSx + bSy + cSz 3 3 abcSx.Sy.Sz nên 8π 3 (abc) 2 Sx.Sy.Sz . 27
  • 55. Tải tài liệu tại sividoc.com Viết đề tài giá sinh viên – ZALO:0973.287.149-TEAMLUANVAN.COM 48 Từ đây suy ra rằng, P đạt giá trị lớn nhất bằng 8π 3 (abc)2 27 khi 2π abc aSx = bSy = cSz = . 3 Giải 2π abc x0 2 = aSx = π abc 1 − 3 a2 được a x0 = ± √ ; 3 Tương tự b c y0 = ±√ ;z0 = ± √ . 3 3 Ta có thể chọn M a b c , chẳng hạn. √ ,√ , √ 3 3 3 Ví dụ 3.0.4. Bên trong một hình cầu bán kính R trong không gian Euclid 3 chiều có một khối đa diện lồi. Đánh số mỗi cạnh khối đa diện lồi qua các số 1, 2, . . . , n. Giả sử cạnh thứ i có độ dài li và góc nhị diện của khối đa diện thuộc cạnh này là α được đo bằng radian. Chứng minh rằng n (π − α )8π.R. ∑ l i i=1 i i Bài giải. Gọi khối đa diện lồi là L. Ký hiệu Kr là tập tất cả các điểm bên ngoài khối đa diện và cách khối đa diện với khoảng cách không quá r. Ký hiệu H1 là một phần của Kr bao gồm tất cả các điểm bên ngoài khối đa diện và nó gần đỉnh khối đa diện nhất; Ký hiệu H2 là một phần của Kr bao gồm tất cả các điểm mà điểm gần nó nhất là một điểm trong thuộc cạnh khối đa diện. Như vậy H1 ∩ H2 = 0/. Tất cả các điểm thuộc H1 ∪ H2 là những điểm trong thuộc mặt cầu bán kính R + r. Do vậy, tổng thể tích của H1 và H2 không vượt quá 4 π (R + r) 3 . 3 Tính thể tích tập H2 : Dựng các hình trụ cạnh thứ i với độ dài cạnh li và góc phẳng nhị diện là 2π − αi và thể tích phần giao bằng 1 (π − αi)r 2 li. 2 Vậy thể tích H2 đúng bằng 1r2 ∑ (π α )l . Thể tích H1 bằng 4π r 3 . Khi đó ta có bất đẳng thức 2 i=1 − i i 3 4 1 4 π r 3 + r 2 ∑(π − αi)li π (R + r) 3 . 3 2 3 i=1
  • 56. Tải tài liệu tại sividoc.com Viết đề tài giá sinh viên – ZALO:0973.287.149-TEAMLUANVAN.COM 49 Vậy ∑(π − αi)li 8π 3R 2 R3 3R + + . 3 r r2 i=1 Cho r → ∞ ta có ∑(π − αi)li 8π R. i=1 Ta có điều phải chứng minh. Ví dụ 3.0.5. Cho hình chóp SABC. Một mặt phẳng cắt ba cạnh bên SA, SB, SC tại A1, B1,C1. Chứng minh rằng, với bất kỳ điểm trong O của tam giác A1B1C1 ta có 7 4 4 √ 2 V SABC 4 VAOB C +4 VBOCA + 4 VCOA B . 4 1 1 1 1 1 1 Bài giải. Đã biết V AOB1C1 = V AOB1C1 . V AA1B1C1 . V ASB1C1 = s(OB1C1) .AA1 .SB1 .SC1 . V V V s(A1B1C1) V AS SB SC SABC AA1B1C1 ASB1C1 SABC Từ đây suy ra s(OB1C1) 4 4 2 V AOB1C1 + 2AA1 +SB1 +SC1 . s(A1B1C1) V SABC ASSBSC Tương tự, ta còn có 4 4 2 V BOC1A1 V SABC 4 4 2 V COA1B1 V SABC Cộng bất đẳng thức được 7 4 V 4 √ 2 SABC 4 Ta có điều phải chứng minh. s(OC1A1) + 2 BB1 + SC1 + SA1 s(A1B1C1) BS SC SA s(OA1B1) + 2CC1 +SA1 +SB1 . s(A1B1C1) CS SA SB 4 V AOB1C1 + 4 V BOC1A1 + 4 V COA1B1 . Ví dụ 3.0.6. Cho tam giác ABC với độ dài cạnh BC = a,CA = b, AB = c. Dựng ra phía ngoài ba tam giác đều BCA1,CAB1, ABC1. Dựng tiếp ra phía ngoài tam giác A1B1C1 ba tam giác đều B1C1A2, C1A1B2, A1B1C2. Chứng minh rằng: (1) AA1 = BB1 = CC1 và AA1, BB1,CC1 đồng quy tại điểm V.
  • 57. Tải tài liệu tại sividoc.com Viết đề tài giá sinh viên – ZALO:0973.287.149-TEAMLUANVAN.COM 50 (2) Với O1, O2, O3 là tâm các tam giác đều BCA1, CAB1, ABC1, ba đường thẳng AO1, BO2,CO3 đồng quy tại W. Hơn nữa, ta còn có đồng nhất thức AO 2 1 + 2 3 a 2 = BO 2 2 + 2 3 b 2 = CO 2 3 + 2 3 c 2 . (3) Tiếp tục quá trình như trên để có ΔAnBnCn. Khi đó A, A1, . . . , An thẳng hàng và tìm giá trị lớn nhất của AnAn+1 khi b, c cố định, còn a thay đổi. (4) Các bộ (BC, O2O3, B1C1), (CA, O3O1,C1A1), (AB, O1O2, A1B1) gồm ba đường thẳng hoặc đồng quy hoặc song song. (5) Với tâm đường tròn ngoại tiếp O của ΔABC, ba điểm O,V,W thẳng hàng. Bài giải. (1) Tương ứng A, B,C ba số phức zA, zB, zC . Ta dễ dàng xác định được zA1 = zC + e iu (zB − zC ) = z iu (z z ) zB A + e 1 C A − iu z = z + e (z − z ) C1 B iu A B π z A2 = z + e (z B1 z ) với u = . C1 C1 3 − Từ
  • 58. Tải tài liệu tại sividoc.com Viết đề tài giá sinh viên – ZALO:0973.287.149-TEAMLUANVAN.COM zC + e iu (zB1 − zC ) = zC + e iu (zA + e iu (zC − zA) − zC )
  • 59. Tải tài liệu tại sividoc.com Viết đề tài giá sinh viên – ZALO:0973.287.149-TEAMLUANVAN.COM 51 ta suy ra hệ thức zC + e iu (zB1 − zC ) = zC (1 − e iu + e 2iu ) + zA(e iu − e 2iu ) = zA vì e iu = −1, e 3iu + 1 = 0 nên 1 − eiu + e2iu = 0, eiu − e2iu = 1. Như vậy ta có zA1 − zA = e iu (zB − zB1 ) và suy ra AA1 = BB1. Tương tự BB1 = CC1. Do góc giữa AA1, BB1,CC1 đều bằng π /3 nên AA1, BB1,CC1 đồng quy tại một điểm (Ta đặt giao điểm M = AA1 × BC, N = BB1 ×CA và P = CC1 × AB. Biến đổi tỷ số sau: T = MB .NC .PA = S ABA1 . S BCB1 . S CAC1 MC NA PBSACA1 S BAB1 S CBC1 c SIN(B + 60 0 ) a SIN(C + 60 0 ) b SIN(A + 60 0 ) = b SIN(C + 600 ) . c SIN(A + 600 ) . a SIN(B + 600 ) = 1. Như vậy, ba đường thẳng AA1, BB1,CC1 đồng quy theo Định lý Ceva). (2) Dễ dàng kiểm tra 3 zO2 + e iu (zO3 − zO2 ) = 3zO1 . Vậy ΔO1O2O3 đều. Ta đặt giao điểm M = AO1 × BC, N = BO2 × CA và P = CO3 × AB. Biến đổi tỷ số qua diện tích T = MB .NC .PA = S ABO1 . S BCO2 . S CAO3 MC NA PBSACO1 S BAO2 S CBO3 c SIN(B + 30 0 ) a SIN(C + 30 0 ) b SIN(A + 30 0 ) = b SIN(C + 300 ) . c SIN(A + 300 ) . a SIN(B + 300 ) = 1. Như vậy, ba đường thẳng AO1, BO2,CO3 đồng quy theo Định lý Ceva. (3) Dễ dàng chỉ ra z A1 + z A2 = 2z A . Vậy A là trung điểm của A1A2. Từ A1A2 = 2AA1 = 2BB1 (b + c). Vậy A1A2 lớn nhất bằng 2(b + c) khi A = 2 3 π . Từ đó có a để A1A2 lớn nhất.
  • 60. Tải tài liệu tại sividoc.com Viết đề tài giá sinh viên – ZALO:0973.287.149-TEAMLUANVAN.COM 52 (4) Giả sử đường thẳng AB cắt đường thẳng A1B1 tại S. Áp dụng Định lý hàm sin cho ΔSBA1 và ΔSAB1 có SA1 = a và SB1 = b . SIN(B + 60 0 ) SIN ∠ BSA1 SIN(A + 60 0 ) SIN ∠ ASB1 Vậy SA1 a SIN(B + 600 ) = . SB1 b SIN(A + 60 0 ) Giả sử đường thẳng O1O2 cắt đường thẳng A1B1 tại S ′ . Áp dụng Định lý Menelaus có S′A 1 . O 2 B 1 . O 1 O = 1 S ′ B1 O2O O1A1 và S′A 1 = O 2 O . O 1 A 1 = aO 2 O . S′ B1 O1O O2B1 bO1O Do SIN(B + 60 0 ) O2O = OC SIN 60 0 nên S′ A1 =aO2O =a SIN(B + 600 ) =SA1 . S ′ B1 b SIN(A + 60 0 ) bO1O SB1 Từ đây suy ra S ≡ S′ ≡ Sc hay ba đường thẳng AB, O1O2, A1B1 đồng quy.
  • 61. Tải tài liệu tại sividoc.com Viết đề tài giá sinh viên – ZALO:0973.287.149-TEAMLUANVAN.COM 53 (5) Xét hai tam giác A1O1A và O2B1B Ba giao điểm A1O1 × BO2 = O, AA1 × BB1 = V và AO1 × BO2 = W thẳng hàng theo Định lý 1.4.1, Định lý Desargues. Nhận xét 3.0.1. Nếu gọi các điểm A ′ , B ′ ,C ′ là trung điểm cạnh BC, CA, AB thì A′ O1 = A ′ A1 , B′ O2 = B ′ B1 , C′ O3 = C ′ C1 ΔA ′ BO1 ∼ ΔA ′ A1B. 3 3 3 Ví dụ 3.0.7. Cho tam giác ABC. Dựng ra phía ngoài hai tam giác đều BCA1, ABC1. Gọi M, N, P là trung điểm CA, BC1, BA1 , tương ứng. Chứng minh tam giác MN P đều. Bài giải. Đặt tọa vị A, B,C, M, N, P là sáu số phức a, b, c, m, n, p, tương ứng. Khi đó zA1 = c + ei u( b − c ), zC 1 = b + ei u(a − b ), u = 3 π a + c b + z C1 b + z A1 iu , α = e m = , n = , p = . 2 2 2 Vì α 2 − α + 1 = 0 nên (m − n)α = (a + c − 2b)α − (a − b)α 2 =(a − b) − (b − c)α . 2 2 Như vậy (m − n)α = m − p và suy ra tam giác MN P đều.
  • 62. Tải tài liệu tại sividoc.com Viết đề tài giá sinh viên – ZALO:0973.287.149-TEAMLUANVAN.COM 54 Ví dụ 3.0.8. Cho tam giác ABC với độ dài cạnh BC = a,CA = b, AB = c. Dựng ra phía ngoài ba tam giác đều BCA1,CAB1, ABC1. Dựng tiếp ra phía ngoài tam giác A1B1C1 ba tam giác đều sau đây: B1C1A2,C1A1B2, A1B1C2. Gọi M, N, P là trung điểm BC,CA, AB; M1, N1, P1 là trung điểm B1C1, C1A1, A1B1, và M2, N2 , P2 là trung điểm B2C2,C2A2, A2B2, tương ứng. Chứng minh rằng (1) AA1 2MM1, BB1 2N N1 ,CC1 2PP1; MM1 , N N1, PP1 đồng quy. = = = (2) A2M2 AM1 MM0, trong đó M0 là trung điểm của AA1. = = (4) Hạ MM2 ⊥ B1C1, N N2 ⊥ C1A1, PP2 ⊥ A1B1. Khi đó MM2, N N2 và PP2 đồng quy. Bài giải. (1) Tương ứng A, B,C ba số phức zA, zB, zC . Ta có 2z M = z B + z C , 2z M1 = z B1 + z C1 . −→ Viết IJ = zJ − zI . Khi đó −→ −→ iu 2MM1 = 2zM1 − 2zM = zB1 + zC1 − zB − zC = zA − zC + e (zC − zB) = A1A.
  • 63. Tải tài liệu tại sividoc.com Viết đề tài giá sinh viên – ZALO:0973.287.149-TEAMLUANVAN.COM 55 Vậy AA1 MM1, AA1 = 2MM1. Ba đoạn MM1, N N1 , PP1 đồng quy được suy ra từ việc xét ΔMN P và phép quay eiu . (2) Từ AA1 2MM1 suy ra AM1 = = suy ra A2M2 AM1 MM0. MM0. Ta lại có AM1 = A2M2 và cũng dễ dàng = = (3) Sử dụng kết quả AA1 = BB1 = CC1 = d và công thức đường trung tuyến ta có 2MB1 2 = d 2 + b 2 − a 2 a 2 , 2MC1 2 = d 2 + c 2 − . 2 2 Vậy MB 2 − MC 2 =b 2 − c 2 1 1 2 hay M2B 2 − M2C 2 =b 2 − c 2 . 1 1 2 Tương tự, có N2C 2 − N2A 2 =c 2 − a 2 và P2A 2 P2B 2 = a 2 − b 2 . 1 1 2 1 − 1 2 Khi đó M2B2 1 − M2C1 2 + N2C1 2 − N2A2 1 + P2A2 1 − P2B2 1 = 0. Do đó MM2 , N N2, PP2 đồng quy tại một điểm. Ví dụ 3.0.9. Cho tam giác ABC với độ dài cạnh BC = a,CA = b, AB = c. Xác định điểm M để MA + MB + MC là nhỏ nhất khi ∠A 2π . 3 Bài giải. Dựng tam giác đều BCA1 ra phía ngoài tam giác ABC. Khi M là điểm Torricelli của tam giác ABC ta có MA + MB + MC = AA1. Khi N = M thì theo Bất đẳng thức Ptolemy ta có N B.CA1 + NC.BA1 NA1 .BC hay N B + NC NA1 và suy ra NA + N B + NC NA + NA1 AA1. Do vậy MA + MB + MC nhỏ nhất bằng AA1 khi M là điểm Torricelli của ΔABC.
  • 64. Tải tài liệu tại sividoc.com Viết đề tài giá sinh viên – ZALO:0973.287.149-TEAMLUANVAN.COM 56 Kết luận 1 Những kết quả đã đạt được Luận văn “Phương pháp diện tích và thể tích trong hình học sơ cấp” đã đạt được các kết quả sau: 1. Trình bày về phương pháp diện tích trong hình học. Những vấn đề đã trình bày là liên quan đến Định lý Pythagore, Định lý Stewart, Ceva, Menelaus và Bất đẳng thức Erdos¨-Mordell cho đa giác. 2. Trình bày về phương pháp thể tích, các khía cạnh của thể tích thông qua định thức và mối quan hệ liên quan đến bán kính của mặt cầu nội và ngoại tiếp. 3. Trình bày chi tiết những vận dụng các kết quả đạt được để giải các bài thi học sinh giỏi. 2 Đề xuất một số hướng nghiên cứu tiếp theo Sau những kết quả đã đạt được trong luận văn, tác giả đặt phương hướng nghiên cứu tiếp theo là làm rõ và khai thác sâu sắc hơn những phương pháp đã trình bày trong luận văn thông qua các bài thi học sinh giỏi, các bài toán nâng cao. Tác giả sẽ chọn ra một số bài tập điển hình sau đó sẽ khai thác và phát triển. Đồng thời, tác giả cũng cố gắng nghiên cứu thêm các phương pháp giải toán điển hình và truyền thống khác trong hình học vào giải toán, chẳng hạn như phương pháp tọa độ và ứng dụng.
  • 65. Tải tài liệu tại sividoc.com Viết đề tài giá sinh viên – ZALO:0973.287.149-TEAMLUANVAN.COM 57 Tài liệu tham khảo Tiếng Việt [1] Nguyễn Văn Mậu, Đàm Văn Nhỉ (2015), Đồng nhất thức và phương pháp tọa độ trong hình học, Nhà xuất bản ĐHQG Hà Nội. [2] Trần Tuấn Nam, Đàm Văn Nhỉ, Trần Trung Tình, Nguyễn Anh Tuấn (2016), Giáo trình Hình học sơ cấp, Nhà xuất bản ĐHSP Thành phố Hồ Chí Minh. Tiếng Anh [3] H.S.M. Coxeter, S. L. Greitzer (1967), Geometry Revisited, The Mathemati-cal Association America. [4] A. Pogorelov (1987), Geometry, Mir Moscow.